• Shuffle
    Toggle On
    Toggle Off
  • Alphabetize
    Toggle On
    Toggle Off
  • Front First
    Toggle On
    Toggle Off
  • Both Sides
    Toggle On
    Toggle Off
  • Read
    Toggle On
    Toggle Off
Reading...
Front

Card Range To Study

through

image

Play button

image

Play button

image

Progress

1/258

Click to flip

Use LEFT and RIGHT arrow keys to navigate between flashcards;

Use UP and DOWN arrow keys to flip the card;

H to show hint;

A reads text to speech;

258 Cards in this Set

  • Front
  • Back
A deficiency of choline in the diet may cause abnormalities in metabolism of
1. lipids.
2. proteins.
3. minerals.
4. carbohydrates.
5. nucleoproteins.
LIPIDS

The correct answer is choice 1. Choline is a structural component of phosphatidylcholine or lecithin, a phosphoglyceride. Phosphoglycerides are major components of cell membranes. The raw material for the synthesis of phosphatidylcholine is dietary choline. In the absence of choline, phosphatidylcholine cannot be produced and a major phospholipid component of wall membranes will be unavailable. So the correct answer to question is choice 1. Choline is not directly involved in the metabolism of the other choices: proteins, minerals, carbohydrates or nucleoproteins.
Which of the following characterize the links between monomeric units of nucleic acids?
1. Ionic bonds
2. Glycosidic bonds
3. Phosphodiester bonds
4. Phosphotriester bonds
PHOSPHODIESTER BONDS

The correct answer is choice 3. Monomeric units of DNA consist of a phosphate group, deoxyribose, sugar, and nitrogen base of adenine, guanine, cytosine or thymine. These monomeric units are linked together to form the DNA strand by phosphodiester bonds. In these bonds, the 3-prime carbon of one deoxyribose links with one oxygen of the phosphate group, while a 5-prime carbon of another deoxyribose links with another oxygen of the phosphate group. This is known as a phosphodiester bond, indicating two bonds to a phosphate group. So the correct answer to question is choice 3.
In anaerobic glycolysis, glucose is broken down into pyruvic acid, which is then primarily converted to
A. Acetyl-CoA
B. Citric acid
C. Glycogen
D. Lactic Acid
E. Creatinine phosphate
LACTIC ACID

The correct answer is D. The anaerobic process of glycolysis enables the cell to create ATP when the mitochondria are unable to meet the "current" energy demands in the absence of oxygen. When the local area is oxygen deficient, glucose is still converted into pyruvic acid, which is subsequently converted to lactic acid. Lactic acid is an organic acid that in the body dissociates into a lactate ion and a hydrogen ion. This process is a temporary solution to maintain the energy demands of the body; however, when the muscle is continually subjected to this anaerobic state, it will cease to contract secondary to the low pH. In the presence of oxygen (aerobic glycolysis) glucose is broken down into pyruvic acid, which is then primarily converted to Acetyl-CoA (choice A), which then enters the TCA cycle. The formation of citric acid (choice B) is the first step in the TCA cycle. Glycogen (choice C) is the storage form of glucose. In skeletal muscle, some of the ATP generated is transferred and stored in another high-energy compound, creatinine phosphate (choice E). The energy stored in this compound is used to "recharge" ADP to ATP during muscle contractions.
In males, growth and development of secondary sex organs are under direct control of
1. FSH and LH.
2. aldosterone.
3. progesterone.
4. testosterone
TESTOSTERONE

The correct choice is #4, testosterone. Testosterone is the primary sex steroid produced by the Leydig cells of the testes. Its effects are as follows: stimulates sex drive, maintains secondary sexual characteristics, stimulates growth and secretory activities of the accessory sex glands, maintains spermatogenesis. It is also responsible for embryogenesis of male ducts, systems and external genitalia and subsequent maturation at the time of puberty. Descent of the testes into the scrotum from the abdominal cavity at birth is under control of testosterone. It also promotes the growth of many bodily tissues through anabolic effects. The action of testosterone on androgen-sensitive tissues appears to be mediated by conversion of testosterone to dihydrotestosterone by the target cell. Dihydrotestosterone is the active hormone in the prostate, epididymis and on the tissues to promote growth. The action is mediated by induction of RNA and protein synthesis via androgen receptors, similar to the effects of other steroids on other steroid receptors. Testosterone can also be convened to estradiol in the testes and peripherally. Again, the correct choice is #4, testosterone
In oxidative phosphorylation, the energy for the synthesis of ATP is provided by the
A. breaking of covalent bonds in glucose molecules
B. creation of carbon dioxide and a metabolic waste product
C. formation of GTP from GPD
D. movement of hydrogen ions through channels in the respiratory enzymes
E. splitting of water molecules into hydrogen and oxygen atoms
movement of hydrogen ions through channels in the respiratory enzymes

The correct answer is D. Oxidative phosphorylation is the process that produces greater than 90% of the ATP used by the cells in our body. The major steps involved in this process occur within the "electron transport system (ETS)" or "respiratory chain" of the mitochondria. The steps at the end of the electron transport system where ATP is generated are as follows: Along the ETS, the respirafory enzymes continually pump hydrogen ions from the matrix of the mitochondria to the intermembrane space, which creates a large concentration gradient. At the end of the ETS, hydrogen ions pass through channels in the respiratory enzymes along the concentration gradient. As the hydrogen ions pass through these enzymes, the energy created is used to convert ADP to ATP. The breaking of covalent bonds in glucose molecules (choice A), creation of carbon dioxide and a metabolic waste product in the TCA cycle (choice B) and splitting of water molecules into hydrogen and oxygen atoms (choice E) all require energy or do not create energy. The formation of GTP from GPD (choice C) is a process that creates ATP in the TCA cycle (Kreb's cycle).
In response to excess water in the body, one would expect to see which of the following physiological changes?
A. Decreased glomerular filtration rate
B. Decreased permeability of the distal convoluted tubules and collecting duct
C. Increased antidiuretic hormone levels
D. Increased permeability of the ascending loop of Henle
E. Increased urine osmolality
Decreased permeability of the distal convoluted tubules and collecting duct

The correct answer is B. When the blood volume increases due to an accumulation of water in the body, the kidneys are responsible for removing the excess fluid. Although both the distal convoluted tubule and collecting tubules are primarily involved in the reabsorption of sodium ions, they can impact the osmolality of urine by increasing or decreasing the reabsorption of water. When there is excess fluid in the body, these two portions of the nephron will decrease their permeability to water; hence, the elimination of fluid in the urine will increase. Deareasing the glomerular filtration rate (choice A) will decrease the amount of water excreted by the kidneys. Antidiuretic hormone (choice C) primarily functions to decrease the elimination of water, through greater reabsorption through the more permeable walls of the collecting tubules. Since the ascending loop of Henle is primarily involved with the reabsorption of ions, increasing its permeability will not increase the elimination of water (choice D). As a general rule, as the urine osmolality increases (urine is more salty), the amount of water in the urine decreases. Therefore, an increased urine osmolality is associated with a decreased excretion of fluid.
In response to respiratory alkalosis, which of the following physiological effects will most likely occur?
A. The body will retain carbon dioxide
B. The kidneys will conserve bicarbonate
C. The kidneys will increase the secretion of hydrogen ions
D. The respiratory rate will increase
E. The tidal volume will increase
The body will retain carbon dioxide

The correct answer is A. Respiratory alkalosis typically develops when the plasma PC02 levels drop below normal levels. Therefore, when this condition occurs, the body will work to retain carbon dioxide to increase the PC02 levels and correct the alkalosis. If the kidneys were to conserve bicarbonate (choice B) or increase the secretion of hydrogen ions (choice C), the severity of the respiratory alkalosis would increase. Since the primary cause of respiratory alkalosis is hyperventilation and loss of C02, increasing the respiratory rate (choice D) would only worsen the situation. Increasing the tidal volume (choice E) would not affect an individuals respiratory alkalosis.
In the catabolism of an amino acid, all of the following are true statements EXCEPT
A. Amino acid catabolism requires the coenzyme derivative of vitamin B6 (pyridoxine).
B. Deamination results in the generation of ammonia.
C. The first step in the catabolism of an amino acid is the removal of the central carbon.
D. The liver is the primary site of deamination
E. Transamination attaches the amino group of an amino acid to a keto acid.
The first step in the catabolism of an amino acid is the removal of the central carbon

The correct answer is C. The first step in the catabolism of an amino acid is the removal of the amino group, not the central carbon. This process of amino acid catabolism requires the coenzyme derivative of vitamin B6 (pyridoxine) (choice A). The amino group can be removed either by deamination or transamination. Deamination results in the generation of ammonia (choice B), which is then quickly detoxified into urea, which is subsequently excreted in the urine. The liver is the primary site of deamination reactions (choice D). Transamination is a process that attaches the amino group of an amino acid to a keto acid (choice E).
In the coagulation phase, which of the following is involved in the common pathway?
A. Christmas factor
B. Fibrinogen
C. Hageman factor
D. Proaccelerin
E. Tissue factor
Fibrinogen

The correct answer is B.
The process of homeostasis aids in the prevention of blood loss through the wall of damaged vessels and establishes a framework for tissue repairs. The common pathway begins when the thromboplastin from both the intrinsic and extrinsic pathways appears in the plasma. After a series of reactions, prompted by these compounds, thrombin completes the coagulation process by converting fibrinogen to fibrin. Christmas factor (choice A), or factor IX, is involved in the intrinsic pathway. Hageman factor (choice C), or factor XII, is involved in the intrinsic pathway and inactivates plasmin. Proaccelerin (choice D), or factor V, and tissue factor (choice E), or factor Ill, are involved with both the extrinsic and intrinsic pathways.
In the glycolytic pathway, the substance formed after the enzyme phosphoglucose isomerase exerts its action is
A. Fructose-6-phosphate
B. Fructose-1,6-biphosphate
C. Glucose-6-phosphate
D. Glyceraldehyde-3-phosphate
E. Pyruvate
Fructose-6-phosphate

The correct answer is A. Glycolysis, in summary, is a series of reactions that converts glucose into pyruvate with the concomitant production of ATP. Glycolysis begins when glucose reacts with hexokinase to form the compound glucose- 6-phosphate (choice C). When this compound interacts with the enzyme phosphoglucose isomerase, the fructose-6-phosphate (choice A) is formed. Following the formation of this compound, the enzyme phosphofructokinase will interact with the fructose-6- phosphate to form fructose I ,biphosphate (choice B). When fructose-l,6-biphosphate interacts with aldolase, glyceraldehyde 3-phosphate (choice D) is formed. After a number of enzymatic reactions, the compound phosphoenolpyruvate is formed. When it interacts with pyruvate kinase, pyruvate (choice E) is formed. In the presence of oxygen the pyruvate is converted to acetyl-CoA.
In the process known as cotransport all of the following are true EXCEPT
A. A carrier protein is required
B. ATP is NOT required
C. Molecules can be moved against a concentration gradient
D. Only one molecule or ion can be moved through the cell membrane at one time
E. The saturation phenomenon may be observed
Only one molecule or ion can be moved through the cell membrane at one time

The correct answer is D. In secondary active transport or cotransport, the carrier protein (choice A) does not require energy or ATP (choice B). In this process, a secondary active transport protein is able to move one or more ions along a concentration gradient. As one material enters a cell, another is removed, hence the term "co-transport" (thus Choice D is not true and is our answer). However, unlike facilitated diffusion, the carrier proteins in this process are able to move molecules against a concentration gradient (choice C). Since there are a limited number of carrier proteins, when large quantities of the ions or molecules present to the carrier proteins, these proteins may become saturated (choice E).
In which of the following conditions might the systolic blood pressure be abnormally high?
A. Cardiac shock
B. Heart failure
C. Anaphylactic shock
D. Decreased arterial compliance
E. Ventricular fibrillation
Decreased arterial compliance

The correct answer is Choice D. Systole is the period of contraction following diastole, a period of relaxation. Systolic and diastolic pressures are often measured by the ausculatory method. In a young adult, the systolic pressure is about 120 mm Hg and about 80 mm Hg for diastolic pressure. Systolic blood pressure may be abnormally high when the arteries become hardened with arteriosclerosis and have decreased compliance (Choice D). Conditions in Choices A, B, C and E will not cause an abnormally high systolic pressure. In these circumstances, the systolic pressure may be lower than normal because the contractility of the heart is compromised.
In which of the following parts of the circulatory system is blood pressure the lowest?
1. Aorta
2. Veins
3. Arterioles
4. Capillaries
5. Large arteries
Veins

The correct choice is #2 veins. Pressure in the circulation is the force applied by the blood against the unit of area of the wall of a blood vessel. It is by convention measured in millimeters of mercury. Blood pressure of 120 mmHg, for example, means that blood exerts the force sufficient to support a vertical column of mercury 120 mm in height. The systolic pressure is the peak of the entire cardiac cycle, occurring early in ventricular systole. The diastolic pressure is the lowest pressure of the entire cardiac cycle, occurring in ventricular diastole. The pulse pressure is the difference between the systolic and diastolic pressure. The venous portion of the circulation returns blood to the heart. They serve as capacitance vessels and at rest store up to 75% of systemic blood volume. Compared to the arterial part of the circulation the venous portion is characterized by much lower pressures and much higher compliances. The normal pressure ranges are as follows:
The right atrium has a pressure between 2-5 mmHg.
The right ventricle has a pressure between 15-20 mmHg.
Pulmonary artery 15-25 mmHg.
Left atrium 5- 12 mmHg.
Left ventricle 100- 140 mmHg.
Aorta 100-140 mmHg.
So again, the correct choice to question is #2, veins.
In which of the following situations is an osmotic diuresis observed?
1. During diabetes mellitus
2. During water deprivation
3. Following ingestion of starch
4. Following injection of Pitressin
During diabetes mellitus

The correct choice is #1, during diabetes mellitus. Before we answer this question it is important you understand the mechanism of insulin action. Insulin increases the maximum velocity of carrier-mediated glucose transport increased energy-dependent transport of amino acid, increased movement of potassium, leads to hyperpolarization of cells. In adipose tissue it increases the affinity of carriers for glucose. In the liver it may increase amino acid transport, but it is important that you understand that it has no effect on glucose transport in the liver. Insulin causes a decrease in the cyclic AMP levels, which affects enzymes that catalyze reactions involving gluconeogenesis and glycogen breakdown. Insulin increases ribosomal RNA activity, therefore increasing protein synthesis. Glucose is the most important signal promoting insulin secretion by the beta cells of the pancreas. Arginine also stimulates the secretion of insulin. Diabetes mellitus type I, or absent insulinopenia, is a state of inadequate glucose utilization. Inadequate levels of insulin lead to an inability to remove excess glucose from the blood. There is a severe lack of ability to store foodstuffs as glycogen, fat and protein. The response is like starvation, so that there is an increase in gluconeogenesis and, therefore, an increase in the concentration of glucose. Loss of glucose in the urine osmotically brings with it the loss of water. With it comes dehydration. So the correct choice is #l.
Ketone bodies are produced as a result of
A. Gluconeogenesis
B. Glycogen breakdown
C. Lipid and amino acid breakdown
D. Protein breakdown
E. Triglyceride breakdown
Lipid and amino acid breakdown

The correct answer is C. Ketone bodies are produced as a result of lipid and amino acid breakdown. When these substances break down, acetyl-CoA is generated. As acetyl-CoA levels increase, ketone bodies begin to form. When excessive amounts of ketone bodies are formed, the pathological state known as ketosis can occur. Gluconeogenesis (choice A) and glycogenolysis (glycogen breakdown) occur when glucose is needed for the production of ATP for various cells in the body. Glycogen breakdown (choice B) results in the production of glucose; lactic acid is also produced in this process. When protein is broken down (choice D), amino acids are formed. Triglyceride breakdown (choice E) results in the production of lipids.
Lipids are most characteristic of which of the following cellular components?
1. Cytoplasm
2. Ribosomes
3. Membranes
4. Chromosomes
Membranes

The correct choice is #3, the membranes. The cell membrane is selectively permeable and has many transmembrane proteins whose functions are still being investigated. Some transmembrane proteins may function as channels for transport of materials into and out of the phospho-lipid bilayer. All of the other choices listed here are subcellular structures and not composed primarily of lipid. The cytoplasm contains these subcellular structures and is confined by the membrane. So again, the correct choice is #3.
Low Density Lipoprotein (LDL) particles get into cells by
A. simple diffusion.
B. cell-cell fusion.
C. active transport.
D. facilitated diffusion.
E. receptor-mediated endocytosis.
receptor-mediated endocytosis.

The correct answer is Choice E. Low density lipoproteins (LDL) are large spherical particles made up of a core of esterified cholesterol surrounded by a lipid bilayer containing protein. It is the primary source of cholesterol for cells outside of the liver and intestine. LDL are transported into cells by receptor-mediated endocytosis (Choice E). When cholesterol is needed, a cell synthesizes receptor proteins for LDL and inserts them into its plasma membrane. LDL binds to these receptors and is taken up by the cell during endocytosis. When too much free cholesterol is accumulated, the cell shuts off LDL receptor synthesis and thus stops the flow of cholesterol into the cell.
Choices A through D do not describe the pathway in which LDL is transported into a cell.
MOST fluid reabsorption by the kidney occurs in which of the following?
A. Distal tubule
B. Proximal tubule
C. Collecting duct
D. Ascending loop of Henle
E. Descending loop of Henle
Proximal tubule

The correct answer is choice B. The nephron is the functional unit of the kidney. It consists of two main parts. The glomerulus portion through which large amounts of fluid are filtered from the blood, and the tubule portion in which the filtered fluid from the glomerulus is converted into urine. Fluid absorption by the kidney occurs in the tubule portion of the nephron. This section is further divided into segments each with a special function. The proximal tubule (Choice B) has the highest capacity for fluid reabsorption. In fact, most of the fluid reabsorbed by the kidney occurs here.
The distal tubule (Choice A) is mainly responsible for reabsorption of ions. It is virtually impermeable to water and urea.
The collecting duct (Choice C) is the terminal segment of the nephron tubule system. It reabsorbs less than 10% of filtered water.
The ascending loop of Henle (Choice D) is also virtually impermeable to water. It is chiefly concerned with reabsorption of solute.
The descending loop of Henle (Choice E) is highly permeable to water and moderately permeable to most solutes. It, however, only reabsorbs about 20% of the filtered water, far less than the proximal tubules.
Most heat is lost from the body by which of the following processes?
A. Conduction
B. Convection
C. Evaporation
D. Insensible perspiration
E. Radiation
Radiation

The correct answer is E. Heat exchange with the environment involves four basic processes: Radiation, conduction, convection, and evaporation. Warm objects, such as the sun or our bodies, will lose heat energy as infrared radiation. Our bodies lose heat energy in the same manner as the sun, except in much smaller amounts. Greater than half the heat lost from our body can be attributed to the process of radiation. Conduction (choice A) is the process where energy is directly transferred through physical contact with another substance. Convection (choice B) is the result of "conductive" heat loss to the air that overlies the body. As the body conducts heat to the air next to the skin, the air warms and rises and is replaced by cool air. Approximately 10-15% of the body's heat is lost through this process. Evaporation (choice C) is a process that changes liquid to a vapor. Insensible perspiration (choice D) is a form of evaporation where heat is lost at a constant rate throughout the day. This form of perspiration accounts for approximately 20% of the body's heat loss.
Most muscles in the body can be classified as
A. Circular muscles
B. Convergent muscles
C. Parallel muscles
D. Pennate muscles
E. Smooth muscles
Parallel muscles

The correct answer is C. Skeletal muscle is the most common type of muscle found within the human body. Muscle fibers within a skeletal muscle form bundles called fascicles. The muscle fibers within a fascicle organization can be classified as being parallel muscles, circular muscles, convergent muscles and pennate muscles. Since most skeletal muscles are composed of parallel muscles, the majority of the muscles in the body can be classified as being parallel muscles. In a parallel muscle the fascicles are parallel to the long axis of the muscle. In a circular muscle (choice A) or sphincter, the muscle fibers are concentrically arranged around an opening or recess. Circular muscles guard the entrances and exits of internal passageways, such as the digestive and urinary tracts. In a convergent muscle (choice B), the muscle fibers are based over a broad area; however, all the fibers come together at a common attachment site. For example, they may pull on a tendon, a tendinous sheet or a slender band of collagen fibers. In a pennate muscle (choice D), the fascicles form a common angle with the tendon. Smooth muscles (choice E) are not considered to be skeletal muscles; these muscles, for example, line the digestive tract.
Neutral fats contain mixtures of one or more fatty acids esterified with
1. sterol.
2. glycerol.
3. lecithin.
4. sphingosine.
5. alcohols of high molecular weight.
glycerol

The correct answer is choice 2. Neutral fats are formed when a molecule of glycerol combines with three fatty acids through a dehydration synthesis. Choice 1, sterols, including cholesterol and lanosterol, are steroids and are not chemically closely related to neutral fats. Choice 3, lecithin, is another name for phosphatidylcholine, a phosphoglyceride found in cell membranes. Choice 4, sphingosine, is a molecule which, together with a fatty acid and a polar head group, makes up a sphingolipid which is another lipidtype molecule. Choice 5 is incorrect. The glycerol, which combines with fatty acids, is an alcohol with three OH groups, but it is a low molecular weight substance. So the correct answer to question is choice 2.
Neutralization of acids by saliva results mainly from which of the following salivary contents?
1. Mucin
2. Ammonia
3. Carbonate
4. Bicarbonate
5. Amino Acids
Bicarbonate

The correct choice is #4, bicarbonate. Saliva is secreted from three pairs of salivary glands: the parotid, submaxillary and sublingual. The average adult secretes 1-2 liters of saliva a day. Saliva is alkaline, which means it has a pH of >7, due to its high concentration of HC03. This helps prevent dental caries by neutralizing any acid produced by bacteria in the oral cavity. Saliva also has elevated concentrations of potassium, amylase, lysozyme, IgA and lactoferrin. The stimulation of parasympathetics and the release of acetylcholine increases the volume of saliva and the concentration of the bicarbonate ion. Stimuli for salivation includes conditioned reflexes, the taste of acid and other tastes, smell, nausea and mechanical stimulation of the oral cavity. So again, the correct choice is #4, bicarbonate.
One would expect to see edema in all of the following situations EXCEPT
A. The blood hydrostatic pressure in the capillaries increases
B. The heart becomes an insufficient pump
C. The kidneys are unable to produce urine
D. The lymphatic vessels become blocked
E. The plasma protein concentration in the blood increases
The plasma protein concentration in the blood increases

The correct answer is E. Edema is defined as an abnormal accumulation of interstitial fluid. The underlying problem associated with all types of edema is the disturbance of the normal balance between hydrostatic and osmotic forces at the capillary level. Plasma proteins affect osmotic forces at the capillary level in the following manner: When plasma protein levels increase, fluid is drawn from the interstitial tissue into the capillaries. Conversely, when plasma protein levels decrease, fluid will flow from the capillaries into the interstitial tissue. As the blood hydrostatic pressure in the capillaries increases (choice A) fluids will be forced out of the capillaries. Note: all of the other answer choices either directly or indirectly cause the hydrostatic pressure in the capillaries (or lymphatic vessels) to increase. When the heart becomes an insufficient pump (choice B), as commonly seen in congestive heart failure, blood will begin to "pool" in the capillary system leading to an increased hydrostatic pressure. If the kidneys are unable to produce urine (choice C) the blood volume will increase, leading to an increased hydrostatic pressure. When the lymphatic vessels become blocked (choice D), fluid will move out of the lymph vessels into the interstitial tissue.
Over a period of time, a man uses 4 liters of oxygen and produces 3 liters of CO2. What is his RQ for that period?
1. 0.75
2. 1.00
3. 1.33
4. 7.00
0.75

The correct choice is #1,0.75. The R.Q. or respiratory quotient is defined as the volume of carbon dioxide produced/volume of oxygen consumed. This is a fairly straightforward problem, since it is not even necessary to convert the units of volume into the same units, since both oxygen and carbon dioxide are in liters. So in this case we have a man producing 3 liters of carbon dioxide and consuming 4 liters of oxygen. Our R.Q. is equal to C02 produced/02 consumed, or 3 liters/4 liters. Liters canceling out, we are left with 314 or 0.75 for the correct choice of #1.
Ovulation is triggered by a marked increase in which of the following?
A. Estrogen
B. Estradiol
C. Progesterone
D. Luteinizing hormone
E. Follicle-stimulating hormone
Luteinizing hormone

The correct answer is Choice D. Ovulation is the first step in the female reproductive cycle when a single ovum is released from an ovarian follicle. It is triggered by a marked increase of luteinizing hormone (Choice D) secreted by the anterior pituitary gland. The luteinizing hormone in turn causes a cascade of effects until the follicle ruptures with the discharge of the ovum. Estrogen, choice A, is incorrect as it has a strong feedback effect in the anterior pituitary gland to maintain low secretory rates of both FSH and LH. Estrogen is in fact used in oral contraceptives to suppress ovulation.
Estradiol, Choice B, is one of the three estrogens presents in the human female. It is also an incorrect choice as explained above.
Progesterone, Choice C, is also incorrect as it promotes secretory changes of the uterine endometrium in preparation for implantation during the latter part of the reproductive cycle.
Follicle stimulating hormone, Choice E, causes growth of the follicles before ovulation, but will not trigger ovulation.
Oxygen tension is GREATEST in which of the following blood vessels?
A. Aorta
B. Pulmonary vein
C. Pulmonary artery
D. Coronary artery
E. Coronary vein
Pulmonary vein

The correct answer is Choice B. Oxygen tension is highest in the pulmonary vein (Choice B). Pulmonary veins contains blood that is richly oxygenated by the lungs on its way to the left atrium of the heart. The vein is unusual as veins are normally oxygen poor.
The aorta (Choice A) receives oxygenated blood from the left ventricle and distributes it to various parts of the body. Though the oxygen tension in the aorta is high, it is not as oxygen rich as the pulmonary vein.
Pulmonary artery (Choice C) receives oxygen poor blood from the right ventricle on its way to the lungs.
Coronary artery and vein (Choices D and E) provide local circulation to the heart muscle itself. They also do not have the greatest oxygen concentrations in the body.
Pantothenic acid is an integral part of
1. NAD.
2. cobalamin.
3. folic acid.
4. coenzyme A.
5. pyridoxine phosphate.
coenzyme A.

The correct answer is choice 4. Co-enzyme A, involved in fatty acid oxidations, fatty acid syntheses, pyruvate oxidation and other reactions, contains a pantothenic acid group as well as an adenine, a ribose phosphate, and a beta-mercaptoethylamine. Choice 1 , NAD, is involved as a co-enzyme in many oxidation reduction reactions, and is known as nicotinamide-adenine dinucleotide. It contains nicotinamide, adenines, phosphate groups and pyridine. Choice 2, cobalamin, is incorrect. Cobalamin, combined with a cyanide group, forms Vitamin B 12. Cobalamin contains a corrin ring system, a cobalt atom, and a ribonucleotide. Choice 3, folic acid, is incorrect. It contains pteridine, para-aminobenzoic acid and glutamic acid. Folic acid is involved in purine and pyrimidine synthesis and transfers one-carbon groups. Choice 3 is incorrect. Pyridoxine phosphate is a pyridoxine co-enzyme closely related to vitamin B6 especially involved in transamination. It contains a pyridoxine ring and phosphate group. So only choice 4, co-enzyme A, contains a pantothenic acid group, and the correct answer to question is choice 4.
Phenylalanine is the precursor essential amino acid of which of the following?
A. Asparagine
B. Cysteine
C. Glutamine
D. Proline
E. Tyrosine
Tyrosine

The correct answer is E. The human body is able to synthesize roughly half of the amino acids necessary to build protein. The amino acids that must be supplied in the diet are termed essential, whereas the amino acids that the body is able to synthesize are termed non-essential. The essential amino acids are as follows: Histidine, Isoleucine, Leucine, Lysine, Methionine, Phenylalanine, Threonine, Tryptophan, and Valine. The non-essential amino acids are Alanine, Arginine, Asparagine, Aspartate, Cysteine, Glutamate, Glutamine, Glycine, Proline, Serine, and Tyrosine. As noted before, phenylalanine is an essential amino acid in mammals. This amino acid undergoes hydroxylation to one of the non-essential amino acids, tyrosine. This reaction is catalyzed by the enzyme phenylalanine hydroxylase. It is noteworthy that tyrosine is an essential amino acid in individuals lacking this enzyme. Asparagine (choice A) is a member of the "oxaloacetate family"; its immediate precursor is aspartate. The immediate precursor of cysteine (choice B) is serine. Serine is also the precursor of the non-essential amino acid glycine. Glutamine (choice C) proline (choice D) and arginine are produced from glutamate. The synthesis of glutamate occurs by the reductive amination of alpha-ketoglutarate.
Platelets play an important role in hemostasis. Which of the following describes this role?
1. They convert fibrinogen to fibrin.
2. They agglutinate and plug small, ruptured vessels.
3. They supply fibrin stabilizing factor.
4. They supply proconvertin for thromboplastin activation.
They agglutinate and plug small, ruptured vessels

The correct answer is choice 2, they agglutinate and plug small ruptured vessels. Within seconds after vascular injury platelets, small anuclear circulating blood cell fragments adhere to the site of damage and pile up on one another to provide a mechanical plug that effectively stops bleeding from minor injuries. Clot formation. however, involves a network of fibers of the blood known as fibrin. Choice 1 is incorrect. Prothrombin converts fibrinogen to fibrin. Choice 3 is incorrect. Plasmin released by damaged cells activates fibrinolysis. Choice 4 is incorrect. Fibrin stabilizing factor is part of the fibrinolytic system. Choice 5 is incorrect. Proconvertin Factor VII is a factor in the extrinsic pathway. So the correct answer to question is choice 2.
Preganglionic autonomic nerve fibers are exclusively
1. somatic.
2. adrenergic.
3. sympatetic.
4. cholinergic.
5. parasympathetic.
cholinergic

The correct answer is choice 4, cholinergic. Acetylcholine is the transmitter for a) all autonomic preganglionic neurons either sympathetic or parasympathetic b) parasympathetic postganglionic neurons and c) motor neurons of the spinal cord at the neuromuscular junction. Postganglionic sympathetic neurons use norepinephrine as the neurotransmitter. Choice 1, somatic, is incorrect. Somatic refers to the body or peripheral. However, preganglionic autonomic nerve fibers arise in the central nervous system. Choice 2, adrenergic, is incorrect. Only postganglionic sympathetic fibers are adrenergic. Choice 3, sympathetic, is incorrect, as there are both sympathetic and parasympathetic preganglionic fibers. Choice 5, parasympathetic, is incorrect for the same reason as choice 3. There are both sympathetic and parasympathetic preganglionic fibers. so the correct answer to question is choice 4.
Prostaglandins are made within cells
1. on an RNA template.
2. on rough endoplasmic reticulum.
3. from methionine.
4. from progesterone.
5. from polyunsaturated fatty acids.
from polyunsaturated fatty acids.

The correct choice is #5, from polyunsaturated fatty acids. Prostaglandin synthesis occurs in cell membranes, utilizing polyunsaturated fatty acids. A prostaglandin is a 20-carbon fatty acid which has contained in its structure a 5-carbon ring. The prostaglandins are hormone fine tuners. They are, therefore, not hormones. Some prostaglandins are PGEI, which functions in breakdown of fat; PGE2, which induces delivery, playing a key role in parturition. PGF2-alpha is another prostaglandin. This functions by decreasing the secretion of progesterone. Prostaglandin synthetase is a key enzyme in prostaglandin synthesis. Another name for this enzyme is cyclooxygenase, since it is involved in the formation of the five-membered ring of this 20-carbon compound. The reason I mention cyclooxygenase is that it is important to know that aspirin inhibits the formation of prostaglandins by inhibiting the cyclooxygenase. Prostaglandins exacerbate the inflammation reaction. So again, the correct choice is #5, from polyunsaturated fatty acids.
Protein kinase regulate the activities of key enzymes through which of the following?
A. Oxidation
B. Hydrolysis
C. Acetylation
D. Phosphorylation
E. Dephosphorylation
Phosphorylation

The correct answer is Choice D. Protein kinase when activated by cyclic AMP modulates key enzymes in different cells by phosphorylation (Choice D). In this way, cyclic AMP can influence many cellular processes. For example, protein kinase phosphorylate glycogen synthetase rendering it inactive and phosphorylate phosphorylase kinase rendering it active. So cyclic AMP is a indirectly responsible for stimulation of glycogen breakdown and inhibition of glycogen synthesis. Choices A, B, C and E do not describe the process in which protein kinase exerts its influence
Purine bases taken in the human diet in the form of DNA or RNA are mostly
1. excreted as urea.
2. excreted in the form of uric acid.
3. reused and converted to ATP needed as a source of energy.
4. broken down to give NH3, and either malonic acid or methylmalonic acid.
excreted in the form of uric acid.

The correct answer is choice 2. The purine bases are adenine and guanine. When digested, their final breakdown product is uric acid. Nucleic acids are acted on by various nucleases which will split the nucleic acid into either guanine nucleotides or adenine nucleotides. Further degradation leads to guanosine or adenosine, and then eventually xanthine and finally to uric acid. So the correct answer to question is choice 2.
Riboflavin is a vitamin that is a constituent of which of the following?
A. coenzyme for amino acid metabolism
B. coenzyme FAD
C. coenzyme NAD
D. decarboxylation reactions
E. prothrombin
coenzyme FAD

The correct answer is B. Riboflavin is a vitamin that is a constituent of the coenzymes FAD and FMN; deficiencies of this vitamin result in epithelial and mucosal deterioration. Pyridoxine acts as a coenzyme in amino acid and nucleic acid metabolism (choice A). Deficiencies of pyridoxine result in retarded growth, anemia, and various GI disorders. Niacin (nicotinic acid) is a vitamin constituent of the coenzyme NAD (choice C). Deficiencies of niacin result in the development of CNS, GI, epithelial and mucosal deterioration; this condition is known as pellagra. Thiamine is a vitamin that acts as a coenzyme in decarboxylation reactions (choice D). Thiamine deficiency can lead to muscle weakness as well as CNS and cardiovascular problems. Vitamin K or phytonadione is essential for the synthesis of prothrombin (choice E) and other clotting factors in the liver. Specifically the vitamin K dependent clotting factors are Factor II, VII, IX and X. A deficiency of vitamin K results in a variety of bleeding disorders.
Secretin functions in digestion of proteins by increasing
1. flow of bile.
2. secretion of pepsin.
3. flow of pancreatic juice.
4. secretion of carboxypeptidase.
flow of pancreatic juice

The correct answer is choice 3. Secretin is secreted by the duodenum as food from the stomach enters the duodenum. When secretin reaches the pancreas, it causes the pancreas to secrete pancreatic juice into the duodenum. Pancreatic juice contains enzymes and also raises the pH of the acidic contents leaving the stomach. Choice 1, flow of bile, would be controlled by cholecystokinin, also from the intestine. Choice 2, secretion of pepsin, is controlled by gastrin from the stomach. Secretion of carboxypeptidase is accomplished by the walls of the small intestine. Carboxypeptidase, choice 4, is a protease and its secretion is not controlled by secretin. So the correct answer to question is choice 3.
Severe thiamin deficiency will result in the development of
A. beriberi
B. pellagra
C. pernicious anemia
D. retarded growth
E. scurvy
beriberi

The correct answer is A. Beriberi is a condition that occurs with thiamin deficiency; this condition is associated with the development of muscle weakness, and CNS and cardiovascular problems, including heart disease. Niacin is a water soluble vitamin. When severe niacin deficiency occurs, there is a good chance that patients will develop CNS, GI, epithelial and mucosal deterioration, otherwise known as pellagra (choice B). Pernicious anemia (choice C) is a type of anemia that occurs with cyanocobalamin deficiency. Retarded growth can occur with deficiencies of pantothenic acid, pyridoxine, and folic acid. Scurvy (choice E) is associated with the development of epithelial and mucosal deterioration; this condition is seen in patients that are vitamin C deficient.
Sounds heard during systole in the antecubital space are produced by
1. closure of AV valves.
2. closure of the aortic valve.
3. turbulent blood flow through the artery.
4. laminar blood flow through the occluded artery.
turbulent blood flow through the artery.

The correct answer is choice 3. Let's review the heart sounds briefly. The first sound or lub occurs when the mitral and tricuspid valves, the A-V valves, close just before systole or ventricular pumping. This rules out choice 1 because A-V closure occurs before systole. Closing of the aortic and pulmonic arterial valves constitute the second sound or dub. This occurs just after systole and so it is also incorrect. Sometimes a low third sound can be heard after diastole during ventricular filling. Choice 3 is correct. During systole, blood flows out of the ventricles into the aorta and pulmonary artery. If this flow is turbulent, a sound can be heard. If it is laminar or smooth flow, it is quiet and therefore choice 4 is incorrect. So the correct answer to question is choice 3.
Strong stimulation of spindles in the masseter muscle causes
1. relaxation of the masseter muscle.
2. contraction of the masseter muscle.
3. contraction of the digastric muscle.
4. no change in mandibular position
contraction of the masseter muscle.

The correct answer is choice 2. Spindles are muscle stretch receptors. They set up a reflex arc which tends to keep muscles at a physiologic length. If the spindle fibers of the masseter are stimulated by stretching, a reflex contraction will be set up to return the masseter to normal length. Choice 1 is incorrect and opposite. Likewise, choice 4 is incorrect as we do expect a reflex contraction. Choice 3 is incorrect. The masseter is primarily a closing muscle while the digastric is primarily an opening muscle. The reflex contraction caused in the masseter, which is a closing muscle, might induce relaxation or stretching of the opposing digastric, but would not cause the digastric to contract. So the correct answer to question is choice 2.
Sympathetic stimulation of the intestinal tract results in all of the following EXCEPT
A. Increased intestinal secretions
B. Hypomotility
C. Decreased gastric enzyme production
D. Inhibition of smooth muscle contractions
E. Vasoconstriction of blood vessels
Increased intestinal secretions

The correct answer is A. The sympathetic nervous system, when stimulated, leads to the following: Hypomotility, decreased gastric enzyme production, inhibition of smooth muscle contractions, vasoconstriction of blood vessels, and decreased intestinal secretions, not increased intestinal secretions. Remember the sympathetic nervous system prepares the body for "Fight or Flight" by increasing the activity of the organs essential for the response; the intestinal tract is not needed in the "Fight or Flight" response. Conversely, parasympathetic stimulation results in increased motility, blood supply to the gut through vasodilatation of blood vessels and increased gut secretions.
Tension on the vocal cords is regulated by which of the following?
A. Extrinsic laryngeal muscles
B. Extrinsic ligaments
C. Intrinsic laryngeal muscles
D. Intrinsic ligaments
E. Vocal folds
Intrinsic laryngeal muscles

The correct answer is C. The larynx is composed of two sets of muscles, the intrinsic laryngeal muscles and the extrinsic laryngeal muscles. The intrinsic laryngeal muscles have two primary functions: regulation of the tension in the vocal folds and opening and closing the glottis. The extrinsic laryngeal muscles (choice A) are responsible for positioning and stabilizing the larynx. During swallowing, both sets of extrinsic and intrinsic laryngeal muscles cooperate to prevent food from entering the glottis. The extrinsic ligaments (choice B) attach the thyroid cartilage to the hyoid bone and the cricoid cartilage to the trachea. The intrinsic ligaments (choice D) bind all nine laryngeal cartilages together. The vocal folds (choice E) protect the entrance to the glottis.
The air remaining in alveoli at the end of a forced maximal expiration is the
1. tidal volume.
2. vital capacity.
3. residual volume.
4. expiratory reserve volume.
5. inspiratory reserve volume.
residual volume

The correct choice is #3, residual volume. Let's briefly review the different lung volumes. The tidal volume is the amount of air inspired and expired with every normal breath, which is approximately 500 ml. The inspiratory reserve volume is the extra volume of air that would be inspired over and above the normal tidal volume. The expiratory reserve volume is the extra volume of air that could still be expired beyond the normal tidal volume. The residual volume is the volume of air remaining in the lungs after maximal expiration. Let's discuss lung capacities. The total lung capacity is the maximum volume to which the lungs can be expanded with the greatest possible inspiratory effect. The vital capacity is the maximal expiration following a maximal inspiration, which is the tidal volume plus inspiratory reserve volume plus expiratory reserve volume. The functional residual capacity is the amount of air remaining in the lungs after normal expiration, and the inspiratory capacity is the total amount of air that can be inhaled after a normal expiration. So again, the correct choice to question is #3, residual volume.
The cerebral nuclei of the extrapyramidal system are specifically responsible for
A. Control of skeletal muscle tone
B. Coordination of limb movements
C. Integration of sensory feedback
D. Processing of equilibrium information
E. Processing of visual information
Coordination of limb movements

The correct answer is B. The axons of the pyramidal cells of the motor cortex descend to synapse on lower motor neurons in the brain stem and the spinal cord. Since there are no intervening synapses, the pyramidal system is able to provide rapid commands to the skeletal muscles and various other organs. There are several other centers that can issue somatic motor commands as a result of processing performed at the unconscious, involuntary level. These centers and their associated tracts comprise the extrapyramidal system. The cerebral nuclei primarily control coordination and preparation of limb movements. The red nucleus, located in the mesencephalon, is the component of the extrapyramidal system primarily responsible for controlling skeletal muscle tone (choice A). The cerebellar nuclei, located in the cerebellum, primarily control the "general" coordination of body movements as well as integrating sensory feedback (choice C). The vestibular nuclei, located in the pons and medulla oblongata, process equilibrium sensations (choice D). The superior colliculi, located in the mesencephalon, process visual information (choice E).
The destruction of a clot is a normal physiological process that begins with the
A. Activation of calcium to produce tissue plasmin
B. Activation of prothrombin, which initiates the production of thrombin
C. Activation of the proenzyme fibrinogen, which initiates the production of fibrin
D. Activation of the proenzyme plasminogen, which initiates the production of plasmin
E. Production of tissue thromboplastin
Activation of the proenzyme plasminogen, which initiates the production of plasmin

The correct answer is D. Following a traumatic injury, a blood "clot" will form to prevent excessive blood loss and promote healing. As this repair process proceeds, the clot will gradually dissolve. The dissolution of a clot, known as fibrinolysis, begins with the activation of the proenzyme plasminogen, which initiates the production of plasmin. Plasmin is a substance that will begin to "digest" the fibrin strands and erode the foundation of the clot. Activation of plasminogen can occur with tissue plasminogen activator. When introduced into systemic circulation, at therapeutic concentrations, this agent will bind to fibrin in a thrombus and convert the entrapped plasminogen to plasmin, which initiates local fibrinolysis, through the degradation of fibrin matrix of the thrombus. Activation of calcium to produce tissue plasmin (choice A) occurs after the proenzyme plasminogen is activated. Both choices B and C are involved in the common clotting pathway; the activation of prothrombin initiates the production of thrombin. Thrombin activates the proenzyme fibrinogen, which in turn initiates the production of fibrin. The production of tissue thromboplastin (choice E) or Factor III is involved in the extrinsic clotting pathway.
The distribution of carbonate within dental enamel follows the same surface to dentinoenamel junction patterns as which of the following?
1. Lead
2. Calcium
3. Fluoride
4. Strontium
5. None of the above
None of the above

The correct choice is #5, none of the above. To understand this question you must understand about the development of the enamel prisms. The enamel prism is laid down via the ameloblasts, specifically via the Tomes' processes on the ameloblasts. When the enamel matrix is formed hydroxyapatite crystallites are deposited in the enamel matrix along the long axis oriented perpendicular to the Tomes' process. Maturation of the enamel matrix involves calcification of the enamel matrix, so distribution of calcium, not carbonate, follows the pattern of the enamel matrix which was laid down via the ameloblasts. The enamel prisms are curved nearest to the dento-enamel junction, and therefore calcification will follow this pattern. So again, the correct choice is #5, none of the above.
The dominant factor controlling absorption of iron from the gastrointestinal tract is
1. excretion of iron in the urine.
2. excretion of iron in the stools.
3. saturation of mucosal cells with iron.
4. concentration of the ferrous iron in bone marrow, spleen and liver.
saturation of mucosal cells with iron.

The correct choice is #3, saturation of the mucosal cells with iron. Most iron absorption occurs at the proximal small intestine. Iron first binds to a receptor on the brush border membrane and then gets transported into the cell. The iron may then be transported to the blood, complexing with transferrin, while some binds to apoferritin in the cell to form ferritin. With time some of the iron in ferritin is converted to free iron and transported to the blood. However, much is lost upon sloughing of the cells. Other major electrolytes absorbed from the small intestine are sodium, potassium and chloride. Sodium is absorbed via passive diffusion, cotransport, or secondary active transport and cotransport with chloride ion. Again, the correct choice is #3, the mucosal cells must be saturated with iron.
The fluid-mosaic model for membrane structure proposes that
A. the outer and inner faces of the membrane are identical.
B. peripheral proteins are situated only on the outer face of the plasma membrane.
C. integral proteins are associated with the hydrophobic phase of the bilayer.
D. both polar and nonpolar ends of membrane phospholipids are within the hydrophobic phase of the bilayer.
integral proteins are associated with the hydrophobic phase of the bilayer.

The correct answer is Choice C. The fluid-mosaic model for embryological membranes was proposed in 1972. This model suggests that membranes are made up of a bilayer of phospholipid molecules with their hydrophobic tails pointing toward the center of the structure where they interact with the hydrophobic tails of the opposing phospholipid layer. Integral proteins are found scattered in the hydrophobic phase of the bilayer (Choice C). These proteins may extend through the entire thickness of the membrane and they may also move freely within the plane of the phospholipid bilayer.
Choice A is incorrect as the outer and inner faces of the membrane are not identical. The inner face as made up of the hydrophobic portion of the phospholipid molecules while the outer face is made up of the hydrophilic portion.
Choice B is incorrect since membrane proteins may extend through to the outer face of the membrane but are not situated only on the outer surface.
Choice D is also incorrect since the model contends that only the non-polar end of the phospholipid molecule stays within the hydrophobic phase of the bilayer.
The gallbladder is caused to contract by the hormone
1. gastrin.
2. secretin.
3. pancreozymin.
4. enterogastrone.
5. cholecystokinin.
cholecystokinin

The correct choice is #5, cholecystokinin. The primary role of the gallbladder is bile storage for release during meals when the gallbladder is relaxed. The sphincter of Oddi has low resistance. Approximately 50% of bile is stored, while the remainder enters the duodenum. Contraction begins approximately a half an hour after meals. Contraction is stimulated by acetylcholine (that's the vagocephalic phase of digestion). Hormones which stimulate contraction include cholecystokinin, or CCK, secretin and gastrin. Contraction, however, is seldom complete, being from 51 % to 99% emptying. The gallbladder wall absorbs water and also it absorbs electrolytes. It concentrates cholesterol and absorbs sodium by the electrogenic pump. It also allows for the passive movement of chloride and the passive movement of water. The secretory product, bile, contains water, phospholipid, cholesterol, bile acids and bilirubin.
The hormone responsible for decreasing gluconeogenesis in the liver is synthesized in which of the following?
A. Adrenal cortex
B. Adrenal medulla
C. Alpha cells in the pancreas
D. Anterior lobe of the pituitary
E. Beta cells in the pancreas
Beta cells in the pancreas

The correct answer is E.
In humans, insulin is the hormone that promotes fuel storage by decreasing gluconeogenesis in the liver, promoting glycogen deposition in the liver, and decreasing blood glucose levels. Insulin is synthesized in the beta cells in the islets of Langerhans in the pancreas. The release of insulin from the beta cells is stimulated by an increase in the concentration of glucose in the blood. The action of insulin is antagonized by the hormones produced by the other answer choices. The adrenal cortex (choice A) is responsible for the production of cortisol. Both epinephrine and norepinephrine are produced in the adrenal medulla (choice B). Glucagon is synthesized in the alpha cells in the pancreas (choice C). The anterior lobe of the pituitary (choice D) is responsible for the production of growth hormone.
The lack of which of the following substances during tooth formation most likely induces enamel hypoplasia?
1. Vitamins A and D
2. Vitamins C and D
3. Vitamins C and K
4. Phosphorus and iron
5. Calcium and fluoride
Vitamins A and D

The correct answer is choice 1, vitamins A and D. Vitamin A is known to be required for the production and maintenance af healthy epithelial tissue and enamel is an epithelial tissue. Vitamin D is produced in the skin by ultraviolet irradiation of 7 dehydrocholesterol. Vitamin D induces synthesis of a protein required for calcium transport and since calcium transport is involved in enamel formation, Vitamin D would be required. In the formation of enamel, many organic and inorganic substances, for example calcium and phosphate, are required. However, even if there are abundant substrate materials, if there is not an adequate amount of vitamin A and D, the formation of enamel will not be adequate. So the correct answer to question is choice 1.
The largest amount of body water can be found in which of the following?
A. Urine
B. Blood plasma
C. lntracellular fluid
D. Interstitial fluid
E. Stomach and intestines
lntracellular fluid

The correct answer is Choice C. The human body is made up of 50 to 60% of water by total body weight. Most of this water is found in intracellular fluid (Choice C) which constitutes approximately 33% of total body weight.
The amount of water in urine (Choice A) is variable. It depends on a number of factors, including the amount of fluid intake, and the amount that is reabsorbed. It does not account, however, for the largest amount of body water.
Choice B body plasma contains only 4.5% of water by total body weight.
Choice D interstitial fluid accounts for approximately 27% of water by weight. It is the source of the second largest quantity of water found in the body.
Choice E stomach and intestines also contain a substantial amount of fluid but do not have the largest quantity.
The main control over body temperature, located in a portion of the central nervous system, is the
1. pons.
2. cerebellum.
3. hypothalamus.
4. medulla oblongata
hypothalamus

The correct choice is #3, hypothalamus. The hypothalamus has a multitude of functions. It centrally maintains the homeostasis of the body by integrating autonomic, somatic and endocrine systems. Temperature regulation involves both posterior hypothalamus and anterior hypothalamus. The posterior hypothalamus responds to cold by producing shivering, vasoconstriction of skin vessels, piloerection and discharge of catecholamines. The anterior hypothalamus responds to excessive heat through sweating, panting and vasodilatation of skin vessels. The receptors monitoring body temperature seem to be located primarily in the anterior or preoptic area of the hypothalamus. Other functions of the hypothalamus include the release of corticotropin, thyrotropin, gonadotropin, somatastatin, prolactin and growth hormone releasing factors. These releasing factors in turn regulate the release of the above hormones from the anterior lobe of the pituitary. So again, the correct choice is #3, hypothalamus.
The major center in the brain for autonomic nervous system regulation is the
1. hypothalamus.
2. pituitary.
3. cerebral cortex.
4. vestibular nuclei.
5. floor of the fourth ventricle.
hypothalamus

The correct choice is #1, hypothalamus. The autonomic nervous system is defined as the efferent or motor neurons to the viscera, cardiac and smooth muscle and glands. It functions to adjust the amplitude of visceral activity and coordinate activities of different viscera. The central nervous system centers are the hypothalamus and brainstem, especially the reticular formation. They contain the controlling neurons, cell bodies which are the sites of autonomic regulation.
The major driving force for formation of a lipid micelle is
1. protein-lipid interaction.
2. hydrophobic interaction between hydrocarbon tails.
3. hydrogen bonding between water molecules of the micellar core.
4. electrostatic interaction between the micellar core and polar heads.
hydrophobic interaction between hydrocarbon tails.

The correct answer is choice 2. Micelles are formed by bile salts or bile acids that are involved in lipid absorption. Bile salts have polar heads and non-polar tails. Polar heads remain outside in contact with the polar water molecules, while the non-polar tails remain on the interior and attach to fatty acid and phospholipid molecules. Thus, the entire micelle is made miscible with water. The driving forces for the formation of micelles are actually two. The first is the interaction of the nonpolar bile salt tail with hydrophobic fatty acids and phospholipids and removal of these hydrophobic groups from the water. The second is the interaction of the polar heads with water. Choice 2 refers to one of these driving forces. Choice 1 is incorrect as proteins are not involved in micelle formation. Choice 3 is incorrect because the micelle core is lipid and hydrophobic, not water. Choice 4 is incorrect as the micelle core is hydrophobic and does not interact with polar heads. So the correct answer to question is choice 2.
The major function of serum LDL is to transport which of the following?
A. Bile salts from the intestine
B. Triglycerides from the intestine
C. Free fatty acids from adipose tissue
D. cholesterol and phospholipids from peripheral tissues
E. Cholesterol, cholesterol esters, and phospholipids from the liver
Cholesterol, cholesterol esters, and phospholipids from the liver

The correct answer is Choice E. Lipoproteins are compounds containing both lipids and proteins. Serum LDL or low density lipoproteins are intermediate-density lipoproteins with a high concentration of cholesterol, moderately high concentration of phospholipids, and minimal amount of triglycerides. Their primary function is to transport their lipid components, cholesterol, cholesterol esters, and phospholipids from the liver to the peripheral tissues (Choice E). Choice A through D are all incorrect since serum LDL does not perform functions described in those choices.
The majority of the hydroxyapatite produced in bone occurs when calcium phosphate interacts with
A. calcium carbonate
B. calcium hydroxide
C. fluoride
D. magnesium
E. sodium
calcium hydroxide

The correct answer is B. Calcium phosphate, Ca3(P04)2. which accounts for approximately two-thirds of the weight of bone. interacts with the calcium hydroxide, Ca(OH)2, to form crystals of hydroxyapatite, Ca10(P04)6(OH)2. As these crystals form, they will incorporate other calcium salts, such as calcium carbonate (choice A), fluoride (choice B), magnesium (choice D) and sodium (choice E).
The maximal number of impulses that a nerve fiber can carry is determined by which of the following?
1. The intensity of the stimulus
2. The diameter of the nerve fiber
3. The duration of the absolute refractory period
4. The duration of the relative refractory period
The duration of the absolute refractory period

The correct answer is choice 3, the duration of the absolute refractory period. The absolute refractory period is a period during which a second stimulus no matter how strong cannot initiate another impulse and it therefore determines the maximum number of impulses that a nerve fiber can cany. Choice 1, the intensity of the stimulus is incorrect. Stimulus is a current such that the membrane is depolarized to the firing level. Any stimulus at or above the threshold intensity would result in the same action potential with constant amplitude. This is known as the all or none response. Choice 2 is incorrect, the diameter of the nerve fiber. The greater the diameter of a given nerve fiber, the greater its speed of conduction. Choice 4, the duration of the relative refractory period is incorrect. The relative refractory period which immediately follows the absolute refractory period is the phase at which sodium inactivation is ending. At this point, a stronger than normal stimulus can initiate another action potential. So the correct answer to question is choice 3.
The nitrogenous product that is excreted in high amounts after prolonged acidosis is
1. urea.
2. ammonia.
3. glutamine.
4. creatinine.
5. aspartic acid.
ammonia

The correct answer is choice 2. In prolonged acidosis, higher than usual amounts of ammonia are excreted. As ammonia is excreted, hydrogen ions are excreted as well in the combined NH4+ form. This helps to reduce the level of hydrogen ions in the body. Choice 1 is incorrect. Urea is normally the primary nitrogenous waste, but after prolonged acidosis, ammonia excretion is greater than urea excretion. Choice 3 is incorrect. Glutamine is changed to glutamate in tubular cells, forming NH4+ or ammonium ions. However, glutamine itself is not excreted. Creatinine is excreted to a small degree in normal urine and to a greater degree in children and in pregnant women, but is never the chief nitrogenous waste product. So choice 4 is incorrect. Choice 5 is incorrect. Aspartic acid and amino acid is not a major nitrogenous waste. So the correct answer to question is choice 2.
The normal delay in sexual development until puberty is attributed to
1. lack of effect of sex hormones on somatic tissue.
2. failure of the gonads to respond to gonadotrophins.
3. inability of the hypophysis to synthesize gonadotrophins.
4. lack of hypothalamic stimulation of gonadotrophin release.
5. inability of the hypophysis to respond to hypothalamic releasing hormones.
lack of hypothalamic stimulation of gonadotrophin release

The correct choice is #4, lack of hypothalamic stimulation of gonadotropin release. In explaining this question let's use the female as an example. The hypothalamic pituitary cycle initiates the menstrual cycle of the female. Luteinizing hormone and follicular stimulating hormone initiate development of the ovary. Gonadotropic releasing hormone regulates the release of LH and FSH. Without hypothalamic stimulation of the gonadotropin release the female will remain prepubescent. In males the production of androgenic steroid hormones by Leydig cells and spermatogenesis is regulated by the anterior pituitary secretions, luteinizing hormone and follicle stimulating hormone. The anterior pituitary also is regulated by the hypothalamic secretion of gonadotropic releasing hormone. In prepubertal boys gonadotropin occurs at low levels. By age 10 there are surges in gonadotropin in the sleep and this initiates puberty. Again, the correct choice is #4.
The oxygen atoms in a molecule of oxygen are held together by
A. a single covalent bond
B. a double covalent bond
C. a triple covalent bond
D. a polar covalent bond
E. an ionic bond
a double covalent bond

The correct answer is B. Oxygen, with an atomic number of 8, has two electrons in its first energy level and six in its second energy shell. Two oxygen atoms in a molecule of oxygen are stable and held together by a double covalent bond; with the configuration 0=0 or 02. A single covalent bond (choice A) can be found between two hydrogen atoms; with the configuration H-H or H2. A triple covalent bond (choice C) is rarely found; it would be represented with the configuration HCEN. A polar covalent bond (choice D) exists when two elements share electrons and one of the elements has a much stronger affinity for the electrons than the other. For example, in molecules of water, an oxygen atom forms covalent bonds with hydrogen atoms. The oxygen has a much stronger attraction for the shared electrons than the hydrogen atoms. An ionic bond (choice E) is formed by the electrical attraction between atoms that have gained or lost electrons and thus carry an electrical charge. Salts are formed by ionic bonds.
The partial pressure of oxygen in arterial blood is lower than that in alveoli. The condition suggested is
1. hyperventilation.
2. slow blood flow.
3. high blood pressure.
4. inadequate ventilation.
5. thickening of the alveolar membrane.
thickening of the alveolar membrane.

The correct choice is #5, thickening of the alveolar membrane. The assumption that alveolar partial pressure, or PA02 equals end capillary partial pressure is not strictly true. That is, equilibrium in the alveoli is not complete. The result is an alveolar atrial oxygen gradient. If there is a diffusion disturbance such as thickened alveolar capillary barriers or when we have pulmonary blood flow which is faster, or a low oxygen mixture is inspired, the alveolar arterial oxygen gradient is exaggerated. Equilibrium is approached less and less, and p02 falls. So again, the correct choice is #5, thickening of the alveolar membrane.
The postganglionic neurons of the sympathetic nervous system release which of the following neurotransmitters?
A. Acetylcholine
B. Dopamine
C. Epinephrine
D. Norepinephrine
E. Serotonin
Norepinephrine

The correct answer is D. The sympathetic and parasympathetic nerve fibers all secrete one of two synaptic neurotransmitters: acetylcholine and/or norepinephrine. The nerve fibers that secrete acetylcholine are termed cholinergic and those that secrete norepinephrine are adrenergic nerve fibers. All the preganglionic neurons in both the parasympathetic and sympathetic nervous systems are cholinergic and release the neurotransmitter acetylcholine. The postganglionic neurons of the sympathetic nervous system release norepinephrine. The postganglionic neurons of the parasympathetic nervous system are also cholinergic; hence, release acetylcholine (choice A). Both dopamine (choice B) and serotonin (choice E) are neurotransmitters primarily located in the brain. Epinephrine (choice C) is a catecholamine that is a potent sympathomimetic stimulator of the alpha and beta receptors throughout the body.
The PR interval in an electrocardiogram is an index of the
1. length of auricular filling.
2. period of isometric relaxation.
3. length of auricular contraction.
4. conduction time between atria and ventricles.
5. period of maximal ejection of ventricular systole.
conduction time between atria and ventricles

The correct answer is choice 4. Let's review the P, Q, R, S, T aspects of the electrocardiogram. The upward P wave is produced by atrial depolarization, the QRS complex by ventricular depolarization, and the ST segment and T wave by ventricular repolarization. So in this case the PR interval is the time from the beginning of depolarization of the atria to the beginning of depolarization of the ventricles. Therefore, it represents the time for the depolarization wave to move from the atria to the ventricles. So choice 4 is our correct answer to question.
The preganglionic neurons of both the parasympathetic and sympathetic nervous systems release which of the following?
A. Acetylcholine
B. Dopamine
C. Epinephrine
D. Norepinephrine
E. Serotonin
Acetylcholine

The correct answer is A.
The sympathetic and parasympathetic nerve fibers all secrete one of two synaptic neurotransmitters: acetylcholine and/or norepinephrine. The nerve fibers that secrete acetylcholine are termed cholinergic; those that secrete norepinephrine are adrenergic nerve fibers. All the preganglionic neurons in both the parasympathetic and sympathetic nervous systems are cholinergic. The postganglionic neurons of the parasympathetic nervous system are also cholinergic; hence, they release acetylcholine. The postganglionic neurons of the sympathetic nervous system mostly release norepinephrine (choice D). Both dopamine (choice B) and serotonin (choice E) are neurotransmitters primarily located in the brain. Epinephrine (choice C) is a catecholamine; it is a potent sympathomimetic stimulator of the alpha and beta receptors throughout the body.
The presence of glucose in the urine proves that the patient has
1. diabetes mellitus.
2. negative nitrogen balance.
3. exceeded his renal threshold for glucose.
4. excessive damage to a glomerulus of the kidney.
exceeded his renal threshold for glucose

The correct choice is #3, exceeded his renal threshold for glucose. Most solutes appearing in the glomerular filtrate are reabsorbed. The maximum rate of reabsorption for a given solute is defined as its maximum tubular resorptive capacity, or Tm Glucose has a given value for Tm. Normally, all filtered glucose is resorbed. Thus, normally it does not show up in the urine. The filtration load of glucose is directly proportional to the plasma glucose concentration. When T, or maximum tubular resorptive capacity is exceeded glucose is excreted. At this point the tubules are absorbing to their maximum, and any extra glucose spills into the urine. The presence of glucose in the urine does not prove the patient has diabetes mellitus, since other states may induce glucose in the urine. Negative nitrogen balance indicates starvation and may show ketonuria, and the appearance of glucose in the urine does not damage the glomerulus, so again, the correct choice is #3, exceeded his renal threshold for glucose.
The primary determinant of the resting membrane potential in a neuron is
A. intracellular negatively charged chloride ions
B. extracellular negatively charged chloride ions
C. membrane permeability to calcium
D. membrane permeability to potassium
E. membrane permeability to sodium
membrane permeability to potassium

The correct answer is D. With respect to the resting membrane in a neuron, the primary determinant of this value is related to the electrochemical gradient of the neuron. This gradient is described as the sum of the chemical and electrical forces across the cell membrane. This resting membrane potential for a given neuron is primarily a reflection of the membrane permeability to the potassium ions. In.other words, the primary determinant of the resting membrane potential in a neuron is membrane permeability to potassium. Although the electrochemical gradient for sodium ions is very large, sodium permeability is small; therefore, sodium ions have a small effect on the normal resting membrane potential (choice E). Sodium ions are primarily attracted by excess negative charges inside of the cell, such as intracellular negatively charged chlorine ions (choice A) and intracellular negatively charged chloride ions (choice B). Hence, these two factors will have a minimal effect on the resting membrane potential. Calcium (choice C) has no effect on the resting membrane potential.
The primary link between the nervous and endocrine system is the
A. Cerebellum
B. Cerebrum
C. Hypothalamus
D. Mesencephalon
E. Pons
Hypothalamus

The correct answer is C. The walls of the diencephalon are composed of the left and right thalamus. A narrow stalk, the infundibulum, connects the floor of the diencephalon, or hypothalamus, to the pituitary gland, which is a component of the endocrine system. The hypothalamus contains various centers involved with emotions, autonomic function and hormone production. The hypothalamus is the primary link between the nervous and endocrine system. The cerebellum (choice A) adjusts voluntary and involuntary motor activities as well as comparing incoming sensory information with anticipated sensations. Conscious thought processes, sensations, intellectual functions, memory storage and retrieval and complex motor actions originate in the cerebrum (choice B). The mesencephalon (choice D) is the center responsible for processing visual and auditory information. The pons (choice E) connects the cerebellum to the brain stem. It also contains nuclei involved with somatic and visceral motor control.
The process of chewing food is associated with which of the following movements?
A. Abduction
B. Elevation
C. Flexion
D. Opposition
E. Protraction
Elevation

By definition, elevation occurs when a structure moves in a superior direction. Conversely, depression occurs when a structure moves in a inferior direction. For example, one depresses the mandible when the mouth opens and elevates the mandible upon closing the mouth. In other words, the process of chewing is a repetitive cycle of depressing and elevating the mandible. Abduction (choice A) is the movement away from the longitudinal axis of the body. For example, swinging the right arm to the side. Flexion (choice C) is described as a movement in the anteriorlposterior plane that decreases the angle between the articulating extremities. For example, flexion of the biceps. Opposition (choice D) is the placement of the thumb opposite to the fingers that allows the hand to grasp and hold various objects. Protraction (choice E) is described as the movement of a given part of the body anteriorly in a horizontal plane; the grasping of ones upper lip with their lower teeth is a protracting movement.
The process of deamination produces which of the following?
A. Acetyl-CoA
B. Ammonia
C. Keto acids
D. Urea
E. Uric acid
Ammonia

The correct answer is B. Amino acids can be catabolized in one of two processes: Deamination or transamination. Deamination is a process that is performed in the preparation of an amino acid for breakdown in the TCA cycle. In deamination, an amino group is removed and ammonia is generated. Since the ammonia produced is a toxic substance, the liver immediately metabolizes the ammonia molecule to urea (choice D), which is a relatively non-toxic, water-soluble compound that is excreted in the urine. Acetyl-CoA (choice B) is produced by the breakdown of pyruvic acid; an acetyl-group is added to these by-products to produce acetyl-CoA. In the process of transamination, the amino group of a given amino acid is transformed to a keto acid (choice C). A keto acid resembles an amino acid except that the second carbon is bound to an oxygen rather than an amino group. Uric acid (choice E) is another non-toxic waste product that is produced when either DNA or RNA is catabolized during the "breakdown" of a cell.
The second heart sound is related to
1. excitation of atria.
2. opening of AV valves.
3. closure of AV valves.
4. opening of aortic valves.
5. closure of aortic valves.
closure of aortic valves

The correct choice is #5, closure of the aortic valves. Let's discuss the heart sounds. S1, or the first heart sound, is synchronous with mitral and tricuspid valve closure. Its intensity is a clinically useful sign. S1 intensity varies inversely with the PR interval on the ECG. The shorter the P-R interval the louder S1. In mitral stenosis the absence of a loud S1 is an important sign suggesting a rigid, heavily-calcified and immobile mitral valve. The second heart sound is called S2. It is synchronous with aortic and pulmonary valve closure. Its intensity also is a clinically useful sign. The third heart sound, S3, or ventricular gallop, occurs in early diastole. It represents vibrations of the left ventricular structures and blood mass as the rapid inflow of blood is limited to the left ventricular diastolic expansion. S3 is a normal sound in children and young adults. In adults, however, over the age of 30 or 35 years, it represents significant global ventricular dysfunction and is frequently associated with ventricular dilatation. It can also be present in severe anemia, thyrotoxicosis and mitral regurgitation. The fourth heart sound is S4. This is atrial gallop. It occurs after atrial contraction and before S1. Again, the correct choice is #5, closure of the aortic valves.
The second step in glycolysis involves the formation of
A. Dihdyroxyacetone phosphate
B. Fructose-l,6-bisphosphate
C. Glucose-6-phsophate
D. Phosphoenolpyruvic acid
E. Pyruvic aci
Fructose-l,6-bisphosphate

The correct answer is B. In summary, glycolysis breaks down a six carbon glucose molecule into two three carbon molecules of Pyruvic acid. This process involves a series of enzymatic steps. The second step in glycolysis involves the formation of Fructose-l,6-bisphosphate. The entire glycolysis pathway is as follows: Glucose Glucose-6-phsophate -> Fructose-1.6-bisphosphate -> Dihdyroxyacetone phosphate -> Glyceraldehyde 3- phosphate -> 1,3 Biphosphoglyceric acid -> 3- Phosphoglyceric acid -> Phosphoenolpyruvic acid -> Pyruvic acid.
The sensory organs concerned with maintenance of skeletal muscle tonus are
1. muscle spindles.
2. gamma efferents.
3. pacinian corpuscles.
4. golgi tendon organs
muscle spindles

The correct choice is # 1 , muscle spindles. Within muscles there are specialized receptors, the muscle spindles and Golgi tendon organs. The muscle spindles function when the muscles are stretched. A good example of this is tapping on the patellar tendon. This stretching causes a volley of impulses in the primary endings which synapse directly on the alpha motor neurons, innervating the extrafusal fibers of the muscle in which the spindle is imbedded. Thus, we elicit a contraction of the quadriceps muscles when we tap on the patellar tendon, and obtain the familiar knee jerk reflex. So again, the correct choice to question is #1, muscle spindles.
The smooth endoplasmic reticulum is primarily responsible for which of the following?
A. Control of metabolism
B. Distribution of materials by diffusion
C. Lipid and carbohydrate synthesis
D. Neutralization of toxic compounds
E. Secretory protein synthesis
Lipid and carbohydrate synthesis

The correct answer is C. With respect to basic cell physiology, the synthesis of both lipids and carbohydrates occurs in the smooth endoplasmic reticulum. Control of metabolism (choice A) is the responsibility of the nucleus of the cell. In addition, the nucleus is also responsible for storage and processing genetic information and controlling protein synthesis. The cytosol of the cell is responsible for the distribution of materials by diffusion (choice B). Peroxisomes are intracellular compounds that contain degradative enzymes and are primarily responsible for the neutralization of toxic compounds. The synthesis of secretory proteins (choice E) occurs in the rough endoplasmic reticulum. Note, protein synthesis, in general, is the responsibility of the ribosomes.
The spinal cord is the only structure in the central nervous system necessary for which of the following body functions?
1. Respiration
2. Simple reflex
3. Temperature regulation
4. Coordinated muscle mavement
Simple reflex

The correct answer is choice 2. Simple reflex arcs involve a receptor and afferent neurons which take impulses to the spinal cord, possibly an intervening connecting neuron within the spinal cord, and an efferent neuron to the effector muscle or gland. Thus, in a reflex, the only CNS or central nervous system section involved is the spinal cord. In other words, the brain is not involved in the reflex arc. The brain is involved in choice 1, respiration, as the respiratory control center is located in the medulla. The brain is also involved in choice 3, as the hypothalamus primarily controls the temperature regulation. Coordinated muscle movement, choice 4, is controlled by the cerebellum of the brain. So the correct answer to question is choice 2.
The tone of the masseter muscle is maintained by the
1. flexor reflex.
2. stretch reflex.
3. crossed-extensor reflex.
4. continuous production of metabolic products.
stretch reflex

The correct choice is #2, stretch reflex. The stretch reflex, or myotactic or tendon jerk reflex, is a fast reflex. It involves the stretch of the annulospiral endings to initiate the muscle contraction. The best example of this is the knee jerk reflex. In the masseter, however, this reflex involves muscle tone, which involves a continuous stretch reflex. This tone helps prepare the muscle for movement. Without the stretch reflex the muscle would not be optimally effective. It would be flaccid and inoperable. The annulospiral endings are muscle spindle, primary endings, and 1A fibers come from them. They have a diameter of 12-20 microns and a conduction velocity of 75-125 m/sec. So again, the correct choice to question is #2, stretch reflex.
The two most important skeletal elements generally used to determine sex and age are
A. Bone thickness and vertebrae curvature
B. Bone weight and bone markings
C. Condition of teeth and muscular mass
D. Presence and percentage of muscular and fatty tissue
E. Teeth and healed fractures
Bone weight and bone markings

The correct answer is B. A study of a human skeleton can reveal important information about an individual. For example, the two most important skeletal elements generally used to determine sex and age are bone weight and bone markings. With respect to the sex of an individual, one can examine the skull. The skull of a male, as compared to a female, will have the following characteristics: The general appearance will be rougher and heavier, the forehead tends to slope, the sinuses are larger, the cranium is around 10% larger, the mandible is larger and robust and the teeth are typically larger. The actual weight of all male bones is often larger to support a bigger mass. Fuhthermore, bone markings are more prominent on all male bones. As years pass, there is a reduction in the size and roughness of bone markings. Furthermore, the weight of a given bone will also decrease. Therefore, one can see that the two most important skeletal elements generally used to determine sex and age are bone weight and bone markings. Regarding choice A, the thickness of "similar" bones in males and females can be closely related and vertebrae curvature increases with age only. The condition of teeth and muscular mass (choice C) are good indicators of a persons general health and exercise habits. The presence and percentage of muscular and fatty tissue (choice C) will depend on an individuals exercise habits, not primarily their sex or age. Looking at a person's teeth and healed fractures (choice E) will show information about an individuals medical history.
The type of articulation found between the temporal bone and the mandible can best be described as a (an)
A. Ellipsoidal diarthrosis
B. Gliding diarthrosis
C. Hinge diarthrosis
D. Pivot diarthrosis
E. Synarthrosis (gomphosis)
Hinge diarthrosis

The correct answer is C. The type of articulation found between the temporal bone and the mandible, otherwise known as the temporomandibular joint, can best be described as a hinge diarthrosis since it allows for elevation and depression of the jaw. An ellipsoidal diarthrosis (choice A) is a type of articulation that has an oval articular face that rests within a depression on the opposing surface, which allows angular movement in two planes, along or across the length of the oval. An example of an ellipsoidal diarthrosis would be the articulation found between the occipital bone and the atlas. A gliding diarthrosis (choice B) is a type of articulation that has flattened or slightly curved faces that slide across one another. Examples of this type of diarthrosis would be the articulations between the thoracic vertebrae and ribs. An example of a pivot diarthrosis (choice D) would be the articulation between the axis and the atlas, since a pivot diarthrosis allows for rotation. An example of a synarthrosis (gomphosis) (choice E) would be the articulation found between the maxilla and the upper teeth as well as the mandible and the lower teeth. Movement does not occur with this type of articulation.
Thyroid secretion is stimulated by
1. hypoxia.
2. propylthiouracil.
3. exogenous thyroid hormone.
4. high serum iodide concentration.
5. prolonged exposure to a cold environment.
6. prolonged exposure to a warm environment.
prolonged exposure to a cold environment.

The correct choice is #5, prolonged exposure to a cold environment. There are two forms of thyroid hormone, T3, triiodothyronine and T4, thyroxin. The thyroid hormones travel in the blood, largely bound to plasma proteins in equilibrium with free hormones. Most of the T3 and T4 is bound to thyroxine binding globulin. T3 is more active, but present at much lower plasma levels than T4. Under thyroid stimulating hormone stimulation the apical membrane of the thyroid follicles undergo marked development of the microvilli projecting into the colloid. The epithelial cell microvilli engulf peripheral colloid from the periphery of the follicles. Phagocytic vacuoles containing colloids fuse with lysosomes, and lysosomal proteases break down the thyroglobulin into small peptides, which include T3 and T4. Thyroid hormones are essential to survival in the cold. Normal animals quickly adapt to increase their oxygen consumption, catecholamines and increase muscular activity. These are the direct agents of the adaptation, and T3 and T4 are permissive to the actions of catecholamines. So again, the correct choice to question is 5, prolonged exposure to a cold environment.
Tropic hormone releasing factors produced in the hypothalamus are transported into the anterior pituitary through
1. nerve impulses.
2. the ultrashort feedback mechanism.
3. the peripheral circulatory system.
4. the microcirculatory portal system.
5. long nerve fibers or duas that also transport neurophysins.
the microcirculatory portal system.

The correct answer is choice 4. Tropic hormone releasing factors are chemicals made in the hypothalamus which act on the anterior pituitary to cause release of tropic hormones. For example, thyroid stimulating hormone releasing factor will be secreted by the hypothalamus, travel in a portal circulation, enter the anterior pituitary and cause that gland to secrete thyroid-stimulating hormone or TSH which will then stimulate the thyroid to release thyroxin. So our correct answer is choice 4. In a portal circulation, one bed of capillaries is connected to another bed of capillaries by a vein. The other main portal system in the body is the hepatic portal systenl in which a capillary bed in the intestine is connected to another capillary bed in the liver by the hepatic portal vein. In the other choices, we should note that neurophysins in choice 5 are sections of precursor molecules of oxytocin and vasopressin which are posterior pituitary hormones produced in the hypothalamus. They are not related to releasing factors. The neurophysins are removed and the active hormone remains. So the correct answer to question is choice 4.
Twiddling your thumbs during a dental physiology lecture demonstrates the action that occurs at
A. Ball-and-socket joints
B. Ellipsoidal joints
C. Gliding joints
D. Hinge joints
E. Saddle joints
Saddle joints

The correct answer is E. Saddle joints have articular faces that resemble "saddles". Each face is concave at one axis and convex on the other axis. This arrangement permits angular motion, including circumduction; however, rotation is prevented. The carpometacarpal joint at the base of the thumb is a prime example of a saddle joint; twiddling your thumbs during a dental physiology lecture demonstrates the action that can occur at this joint. Ball-and-socket joints (choice A) have a round head at one end that rests within a cup-shaped depression in another. All combinations of movements can be performed at this joint. Prime examples of ball-and- socket joints would be at the hip and shoulder. Ellipsoidal joints (choice B) have an oval articular face that rests within a depression on the opposing surface, which allows angular movement in two planes, along or across the length of the oval. These joints connect the fingers and toes with the metacarpals and metatarsals, respectively. Gliding joints (choice C) have flattened or slightly curved faces that slide across one another. Although rotation is theoretically possible, ligaments usually prevent or restrict such movements. These joints are at the ends of the clavicles, for example. Hinge joints (choice D) permit angular movement in a single plane, much in the same way as opening and closing a door on a hinge. Prime examples of hinge joints include the elbow and knee.
Two atoms are considered isotopic if
1. their nuclei contain the same number of neutrons.
2. their atomic numbers are the same, but their mass numbers differ.
3. their mass numbers are the same, but their atomic numbers differ.
4. one is a beta emitter, and the other an alpha emitter.
their atomic numbers are the same, but their mass numbers differ.

The correct answer is #2, their atomic numbers are the same but their mass numbers are different. This is a simple definition type question from general chemistry. The atomic number, as you may recall, is the number of protons in the nucleus of an atom. Isotopes have the same number of protons bur a different number of neutrons. The number of protons plus neutrons of an atom is referred to as the mass number of the atom. If two atoms are isotopes they therefore have different mass numbers. since mass number equals number of protons plus the number of neutrons. If the number of protons is the same and the number of neutrons is different the number of protons plus the number of neutrons will be different, or the mass number will be different. So the correct choice is #2, their atomic numbers are the same but their mass numbers are different.
Vascular resistance is directly related to all of the following EXCEPT
A. The amount of friction in the blood vessels
B. The diameter of the blood vessel
C. The length of the blood vessel
D. The viscosity of the blood
E. The volume of blood in the body
The volume of blood in the body

The correct answer is E. Resistance is defined as any force that opposes movement. The most important component in vascular resistance is the friction caused between the blood and the vessel walls, which is related to the viscosity of the blood as well as the length and diameter of the blood vessel itself. Although the volume of blood in the blood may impact vascular resistance, it does not directly oppose blood flow. As a general rule, vascular resistance and friction (choice A) will increase when the following occur; the diameter of the vessel decreases (choice B) and/or the length of the vessel increases (choice C). Furthermore, as the viscosity of the blood increases (choice D), the resistance will increase as well; remember, syrup flows slower than water.
Vascular smooth muscle relaxes in response to
A. hyperoxia.
B. adenosine.
C. vasopressin.
D. angiotensin.
D. norepinephrine
adenosine

The correct answer is Choice B. Of the following substances, only adenosine (Choice B) has a vasodilator effect on blood vessel smooth muscles. It is released from the tissue in response to oxygen deficiency causing increased blood flow and oxygen concentration.
Choice A hyperoxia, or an increased oxygen content in body tissues, will cause a local vasoconstriction in the blood vessels and not vasodilation.
Choice C vasopressin, or antidiuretic hormone, is mainly responsible for the retention of water in the body. In large concentrations, it will cause constriction of the blood vessels and elevate blood pressure.
Choice D angiotensin is a powerful vasoconstrictor. Its primary role is to increase the total peripheral resistance in regulation of arterial pressure.
Choice E norepinephrine is released by the sympathetic nerve endings during stress or exercise. Norepinephrine stimulates the veins and arterioles by constriction of these vessels.
Visual purple, rhodopsin, is usually formed in the
1. lens.
2. liver.
3. retina.
4. adrenals.
5. vitreous humor.
retina

The correct choice is #3, the retina. Rhodopsin is a light-sensitive pigment found in the retina. Rhodopsin is formed by the combination of opsin, a protein, with vitamin A aldehyde. Vitamin A aldehyde is also referred to as retinal. Retinal is formed by the oxidation of vitamin A, or retinol. Retinol is preformed vitamin A found in liver, whole milk, eggs and fish oils. Humans intake pro-vitamin A from plants and preformed vitamin A from animals. Beta carotene is the principal dietary source of pro-vitamin A. It is found in yellow and orange vegetables. One molecule of beta carotene is cleaved in the intestinal mucosa into two molecules of retinol, or vitamin A. Again, the correct choice is #3, retina.
Vitamin A functions to
1. prevent pellagra.
2. promote absorption of calcium.
3. promote differentiation of epithelial cells.
4. maintain the integrity of connective tissues.
promote differentiation of epithelial cells

The correct answer is choice 3. Vitamin A is a constituent of visual pigments and maintains epithelial tissues by promoting differentiation. Deficiency of vitamin A is signalled by night blindness and dry skin. Choice 1 is incorrect. Pellagra is caused by niacin deficiency. Choice 2 is incorrect. Vitamin D increases intestinal absorption of calcium and phosphorous. Choice 4 is incorrect. Vitamin C is associated with maintaining normal intercellular materials such as collagen in bone, cartilage and dentin. So our correct answer to question is choice 3.
What type of muscle contraction occurs when both ends of the muscle are fixed and no change in length occurs during the contraction, but the tension increases?
A. Concentric contraction
B. Dynamic contraction
C. Eccentric contraction
D. Isometric contraction
E. Isotonic contraction
Isometric contraction

The correct answer is D. An isometric contraction occurs when both ends of the muscle are fixed and no change in length occurs during the contraction, but the tension increases. A concentric contraction (choice A) is a type of dynamic contraction where the muscle fibers shorten and the tension on the muscle increases. A dynamic contraction (choice B) includes both concentric and eccentric types of contractions. In general, a dynamic contraction results in the change in length of the muscle with a corresponding change in tension on that muscle. An eccentric contraction (choice C) is a type of dynamic contraction where the muscle fiber lengthens and the tension on the muscle increases. An isotonic contraction (choice E) occurs when a muscle shortens against a fixed load while the tension on that muscle remains constant.
Voluntary control of the muscles involved in the process of mastication is provided by which of the following?
A. Afferent nervous system
B. Autonomic nervous system
C. Parasympathetic nervous system
D. Somatic nervous system
E. Sympathetic nervous system
Somatic nervous system

The correct answer is D. The somatic nervous system provides voluntary control over all skeletal muscle contractions. Therefore, voluntary control of the muscles involved in the process of mastication would be provided by the somatic nervous system. The afferent nervous system (choice A) is primarily involved with bringing sensory information to the CNS from peripheral organs and tissues. The autonomic nervous system (choice B) is responsible for providing autonomic, involuntary regulation of cardiac muscle, smooth muscle and glandular activity. The autonomic nervous system contains both the parasympathetic (choice C) and the sympathetic (choice E) nervous systems. Both of these divisions of the autonomic nervous are antagonistic to one another.
When a muscle is producing peak tension during rapid cycles of contraction and relaxation, it is said to be in
A. Complete tetanus
B. Incomplete tetanus
C. Treppe
D. Wave summation
Incomplete tetanus

The correct answer is B. Since there is no mechanism to regulate the amount of tension produced in a given muscle contraction, the muscle fiber is either producing tension or is relaxed. This feature is known as the all- or-none principle. Therefore, the amount of tension produced by a skeletal muscle as a whole is determined by both the frequency of stimulation and the number of muscle fibers stimulated. With these principles in mind, the following definitions can be applied: When a muscle is producing peak tension during rapid cycles of contraction and relaxation, it is said to be in incomplete tetanus. Complete tetanus (choice A) occurs when a muscle is stimulated so frequently that the relaxation phase is completely eliminated. Recruitment (choice C) occurs when there is a smooth but steady increase in the muscle tension that is produced by increasing the number of active motor units. Treppe, (choice D) is a situation where a muscle is stimulated repeatedly for several seconds with a constant stimulus; the amount of tension produced then gradually increases to a maximum. Wave summation (choice E) occurs when a second stimulus arrives before the relaxation phase has ended and a second, more powerful, contraction occurs.
When an individual is administered large amounts of glucocorticoids, such as prednisone, a condition similar to one of the following may result. The patients clinical appearance would be most similar to
A. Addison's disease
B. Adrenogenital syndrome
C. Cushing's syndrome
D. Myxedema
E. Primary Aldosteronism
Cushing's syndrome

The correct answer is C. The Cushing's syndrome is caused by a hypersecretion of cortisol by the adrenal cortex. Cortisol is glucocorticoid similar to prednisone. Therefore, if one were administered large amounts of prednisone for an extended period of time, a Cushing's-like syndrome could result. Cushing's syndrome is characterized by an increased fat deposition in the thoracic region, an edematous appearance of the face, as well as the development of acne and hirsutism. Addison's disease (choice A) results from the failure of the adrenal cortex to produce adrenocortical hormones, resulting in glucocorticoid and mineralocorticoid deficiency. Loss of cortisol (a glucocorticoid) secretion results in the depression of many metabolic functions and decreases one's ability to tolerate various stressors. Adrenogenital syndrome (choice B) is the result of excessive production of androgens, leading to premature masculinizing effects in young males. Myxedema (choice D) is caused by a lack of thyroid function. Patients typically present with swelling under the eyes as well as swelling of the face. Primary Aldosteronism,(choice E), is caused by a hypersecretion of aldosterone leading to hypokalemia, which can eventually lead to periods of muscle paralysis.
When an individual quickly rises from a prone position, one would expect to see
A. Heart rate decrease
B. The cardiostimulatory center in the medulla become less excited
C. The carotid baroreceptors become inhibited
D. Vasoconstriction of the peripheral blood vessels
E. Venous return increase
Vasoconstriction of the peripheral blood vessels

The correct answer is D. When a person rises from a prone position, there will typically be a "pooling" of blood in the venous system. In an effort to maintain uniform blood pressures, the carotid baroreceptors, when stimulated by a decrease in blood pressure, will initiate the following: There will be widespread vasoconstriction of the peripheral blood vessels. The net effect will be a shunting of blood to the internal organs and a decreased venous return (choice E). Also, when the carotid baroreceptors are stimulated (choice B), the heart rate will increase (choice A) due to stimulation of the cardiostimulatory center in the adrenal medulla (choice C).
When are nerve fibers hypoexcitable?
1. During resting potential
2. At the firing level
3. During local depolarization
4. During negative after-potential
5. During positive after-potential
During positive after-potential

The correct answer is choice 5. At certain times or under certain conditions, nerve cells may become hypoexcitable; that is, the nerve cell becomes more difficult to stimulate in order to initiate an action potential. Let's look at our choices one at a time. Choice 1, during resting potential, the neuron is at its normal state of excitability, being neither hyper nor hypoexcitable. Choice 2, when nerve fibers are at the firing level or at the point of initiating an action potential, any further stimulation will cause the action potential to occur, so they would not be hypoexcitable at this point. Choice 3 is incorrect. When a neuron experiences a local depolarization response, a depolarization occurs which is strong enough to cause changes in the membrane, but not strong enough to cause an action potential. However, further depolarization may cause the action potential to occur. In this way, the neuron can be viewed as perhaps hyperexcitable, but certainly not hypoexcitable. Choice 4 is incorrect. Following an action potential, a refractory period occurs during which the neuron cannot be stimulated again. Following this period, a brief negative after-potential will occur, during which the neuron is hyperexcitable. Following this period, a positive after-potential will occur, during which the neuron is slightly hypoexcitable. Later, the neuron will return to its normal level of excitability. So the correct answer to question is choice 5.
When the blood supply to the dermis decreases and the sweat glands become less active, as seen in the elderly, which of the following factors would be affected the most?
A. The ability to fight infection
B. The ability to produce vitamin D
C. The ability to repair itself
D. The ability to thermoregulate
E. The skin will begin to wrinkle
The ability to thermoregulate

The correct answer is D. The skin is a highly complex organ that has many different physiological functions. As individuals age, the skin begins to lose its ability to perform each of these specialized functions. In the elderly, the blood flow to the dermis begins to decrease. Furthermore, glandular activity decreases. When the blood supply to the dermis decreases and the sweat glands become less active, the individual is less able to lose body heat. Therefore, increased physical exertion can lead to extremely high body temperatures. The decreased ability of the skin to fight infection in the elderly (choice A) is related to the decreased number of Langerhans cells and the thinning of the germinative cell layer. A decreased ability to produce vitamin D (choice B) is a normal part of the aging process. This decreased production contributes to a reduction of calcium levels eventually leading to muscle weakness and decreased bone density. The thinning of the epidermis and the loss of elasticity of epidermis contribute to the decreased ability of the skin to repair itself (choice C) as in increased "wrinkling" (choice E) of the skin.
When the pH of the extracellular fluid decreases, the kidney will compensate by
A. Excreting more bicarbonate ions
B. Excreting more sodium ions
C. Increasing the urine pH
D. Reabsorbing less water
E. Retaining more potassium ions
Retaining more potassium ions

The correct answer is E. Although the buffer system in the body is able to regulate the blood pH during normal circumstances, the kidneys may need to aid in the regulation of the blood pH when this system becomes overwhelmed. When the pH of the extracellular fluid declines, the following will occur with respect to potassium: Hydrogen ions will move into the intracellular fluid in exchange for potassium. Furthermore, the kidneys will decrease the elimination of potassium because hydrogen ions are being eliminated in the urine instead of potassium. It is important to note that both of the previously mentioned factors can lead to severe hyperkalemia. In addition to the removal of hydrogen ions, the kidneys will decrease the elimination of bicarbonate ions (choice A), sodium ions (choice B) and water (choice D). In a situation where the extracellular fluid pH is acidic, the kidneys will compensate by acidifying the urine by excreting more hydrogen ions (choice C).
When the ventricle of the heart contracts, the stimulus for the contraction comes directly from which of the following?
A. Atrial muscle
B. AV node
C. His-bundles
D. Purkinje fibers
E. SA node
Purkinje fibers

The correct answer is D.
With respect to normal cardiac conduction, the electrical impulse that depolarizes the heart normally originates in the sinoatrial (SA) node (choice E). The depolarizing impulse travels from the SA node concentrically through the atrial myocardium (choice A), eventually reaching the atrioventricular (AV) node (choice B). From the AV node, the depolarizing impulse enters the His-bundles (choice C), where conduction becomes rapid again. The His-bundle penetrates the interarterial septum, where it divides into the right bundle branch (RBB) and left bundle branch (LBB). The RBB and LBB arborize into a fine network of Purkinje fibers (choice D). The Purkinje fibers penetrate the ventricular myocardium and function to depolarize the ventricular muscle mass, leading to ventricular contraction.
Where in the autonomic nervous system is norepinephrine stored?
A. Preganglionic sympathetic nerve endings
B. Postganglionic sympathetic nerve endings
C. Preganglionic parasympathetic nerve endings
D. Postganglionic parasympathetic nerve endings
Postganglionic sympathetic nerve endings

The correct answer is Choice B. Norepinephrine is synthesized and stored in vesicles located in the postganglionic sympathetic nerve endings (Choice B).
Preganglionic sympathetic nerve endings (Choice A), preganglionic parasympathetic nerve endings (Choice C), and postganglionic parasympathetic nerve endings (Choice D) are all cholinergic because the neurosecretory transmitter for these neurons to acetylcholine.
Where is norepinephrine stored?
1. At preganglionic sympathetic nerve endings
2. At postganglionic sympathetic nerve endings
3. At preganglionic parasympathetic nerve endings
4. At postganglionic parasympathetic nerve endings
At postganglionic sympathetic nerve endings

The correct answer is #2, at the postsynaptic ganglionic sympathetic nerve endings. Norepinephrine is the transmitter in C postganglionic neurons of the autonomic nervous system. In the central nervous system norepinephrine-containing neurons are prominent in the locus ceruleus, which projects diffusely to the cortex, cerebellum and spinal cord and is involved in alerting mechanisms. Sympathetic postganglionic fibers usually store norepinephrine as their neurotransmitter. However, there are always exceptions to a rule. Sympathetic postganglionic neurons to sweat glands and to some of those causing dilatation of blood vessels in skeletal muscles are often cholinergic neurons. Again, the correct answer is #2, at postganglionic sympathetic nerve endings.
Which of following vitamins prevents the destruction of fatty acids and vitamin A?
A. Vitamin B6
B. Vitamin C
C. Vitamin D
D. Vitamin E
E. Vitamin K
Vitamin E

The correct answer is D. Vitamins are typically classified as being fat-soluble or water-soluble. The fat soluble vitamins are vitamins A, D, E and K. All of the "B" vitamins and vitamin C are classified as being water-soluble vitamins. Vitamin E is responsible for preventing the breakdown of vitamin A and fatty acids. Vitamin E is typically found in meats, milk and vegetables. When Vitamin E is deficient, anemia can develop. Vitamin B6 (choice A) acts as a coenzyme in amino acid and lipid metabolism. Vitamin C (choice B) acts as a coenzyme; it delivers hydrogen ions and acts as an antioxidant. It is essential for connective tissue formation. Vitamin D (choice C) is required for normal bone growth as well as facilitating both calcium and phosphorus absorption from the intestinal tract. Vitamin K (choice E) is essential for liver synthesis of prothrombin and other clotting factors.
Which of the following age-related skeletal changes will occur first in an individual's life?
A. Appearance of major vertebral curves
B. Appearance of secondary dentition
C. Decrease in the bones mineral content
D. Fusion of epiphyseal plates
E. Remodeling of the bone
Appearance of major vertebral curves

The correct answer is A. A review of the human skeleton can reveal important information about an individual's age and/or sex. In sequential order of one's life-span the following age- related events will occur: The appearance of major vertebral curves occur begin at the age of two fetal months. The fetal spine is "bow" shaped and the adult spine is more "S" shaped. The appearance of secondary dentition (choice B) occurs when the primary teeth are lost and the permanent teeth take their place. The fusion of epiphyseal plates (choice D) typically occur near the beginning of adulthood. The mineral content of bone (choice C) typically decreases after the age of 65. The bone remodeling process (choice E) is one that occurs throughout a persons life at a relatively constant rate and is not related to age. In the bone remodeling process, the matrix of the bone is replaced while leaving the bone as a whole unchanged.
Which of the following amino acids can most easily be converted to tyrosine in the human body?
1. Glycine
2. Arginine
3. Methionine
4. Tryptophan
5. Phenylalanine
Phenylalanine

The correct answer is choice 5, phenylalanine. In the category of aromatic amino acids are phenylalanine, tyrosine and tryptophan. In phenylalanine, the R group contains a benzene ring and in tyrosine, it contains a phenol group. Therefore, forming tyrosine from phenylalanine only requires hydroxylation of the benzene ring at carbon 3. So the correct answer to question is choice 5.
Which of the following causes activation of the pyloric pump, relaxation of the pylorus, and contraction of the lower esophageal sphincter?
A. Gastrin
B. Secretin
C. Pepsinogen
D. Acetylcholine
E. Cholecystokinin
Gastrin

The correct answer is Choice A. Gastrin (Choice A) causes activation of the pyloric pump along with relaxation of the pylorus, and contraction of esophageal sphincter. It is a hormone released by the antral mucosa of the stomach.
Secretin (Choice B) is also a hormone of the digestive tract. It is formed by the epithelial cells of the duodendum and its primary function is to stimulate pancreatic secretion of sodium bicarbonate.
Pepsinogen (Choice C) is an inactive proenzyme formed and secreted by the chief cells of the gastric mucosa. When it comes into contact with hydrochloric acid, it becomes active enzyme pepsin.
Acetylcholine (Choice D) is a neurotransmitter synthesized in the cytoplasm of synaptic terminals.
Cholecystokinin (Choice E) secreted by the duodenal and upper jejunal mucosa is chiefly responsible for the stimulation of gallbladder contraction.
Which of the following causes some degree of temporary alkalosis?
1. Hyperventilation
2. High fluid intake
3. Excessive smoking
4. Severe muscular effort
5. Ingestion of ammonium chloride
Hyperventilation

The correct choice is #1, hyperventilation. Alveolar ventilation is adequate for oxygenation when it matches the oxygen with its supply. Hypoventilation leads to a buildup of pC02 and a decrease in p02. The buildup of pC02 leads to an acidotic state in the blood, and since it is caused by a respiratory abnormality; that is, hypoventilation, it is referred to as respiratory acidosis. Likewise, hyperventilation leads to a decrease in pC02 and an increase in p02 for an alkaline blood pH. This is called respiratory alkalosis. Drugs which depress the respiratory center in the medulla, such as barbiturates, will cause an acidosis. Also, some drugs may induce a hyperventilatory state, thus may induce a respiratory alkalosis. So again, the correct choice to question #62 is #1, hyperventilation.
Which of the following cells are responsible for producing antibodies against the bacteria found in a abscessed tooth?
A. Fibroblasts
B. Mast cells
C. Macrophages
D. Mesenchymal cells
E. Plasma cells
Plasma cells

The correct answer is E. Plasma cells are derived from lymphocytes. These cells are responsible for producing antibodies, including the antibodies produced against the bacteria found in a abscessed tooth. When an antibody binds to an antigen, an antibody-antigen complex is formed. The specificity of the binding depends on the type of antibody employed to the site of the infection. There are five major classes of antibodies: IgG, IgM, IgE, IgA and IgD. Fibroblasts (choice A) are the most common cells in the connective tissue proper. These cells are responsible for the production and maintenance of connective tissue fibers. Mast cells (choice B) are mobile connective tissue cells commonly found near blood vessels. The cytoplasm of these cells contain histamine which stimulates local inflammation when an infection or injury to the area is present. Macrophages (choice C) are large cells that primarily function to engulf damaged cells or pathogens in the body. Mesenchymal cells (choice D) are the stem cells present in many connective tissues. These cells respond to a local infection or injury by dividing and producing "daughter" cells that eventually differentiate into macrophages, fibroblasts or other connective tissue cells.
A deficiency of choline in the diet may cause abnormalities in metabolism of
1. lipids.
2. proteins.
3. minerals.
4. carbohydrates.
5. nucleoproteins.
Choline
Choline is a structural component of phosphatidylcholine or lecithin, a phosphoglyceride. Phosphoglycerides are major components of cell membranes. The raw material for the synthesis of phosphatidylcholine is dietary choline. In the absence of choline, phosphatidylcholine cannot be produced and a major phospholipid component of wall membranes will be unavailable. So the correct answer to question is choice 1. Choline is not directly involved in the metabolism of the other choices: proteins, minerals, carbohydrates or nucleoproteins.
A deficiency of rhodopsin is most likely caused by decreased dietary intake of
1. glucose.
2. adenine.
3. vitamin A.
4. tryptophan.
5. riboflavin.
vitamin A.
Rhodopsin is a photosensitive molecule in rods. It is made up of opsin and 1-cis-retinal. The precursor of 11-cis-retinal is II-trans-retinol, or vitamin A. Vitamin A is an essential vitamin; that is, it cannot be synthesized in mammals. The all-trans-retinol is converted to 11-cis-retinal by retinol dehydrogenase, followed by an isomerization between the 11th and 12th carbon double bonds, from trans to cis configuration by retinal isomerase. If vitamin A is deficient in the human it will lead to night blindness and eventual degeneration of the rods. So the correct choice is #3, vitamin A.
A deficiency of vitamin A in a developing tooth most likely affects the
1. pulp.
2. enamel.
3. dentin.
4. cementum.
Enamel
To answer this question it is important to think back to the development of the tooth. In looking at our choices, pulp enamel, dentine and cementum, we see that dentine, cementum and pulp are classified as connective tissues, while enamel may be classified as a product of epithelial cells. The tooth, therefore, is from mesenchymal and ectodermal origin. The mesenchyme is derived from neural crest, and the epithelium, as I mentioned before, is derived from ectoderm. Vitamin A plays a key role in protein synthesis of epithelial cells. The exact role has not been elucidated. However, you can see here that you can answer this question by just looking at your choices. So again, the correct choice is #2. (*enamel hypoplasia)
A derivative of vitamin K is the coenzyme for which of the following?
A. Production of menadiol
B. Esteilfication of retinol
C. Hydrolysis of peptide bonds
D. Cross-linking of fibrinogen
E. Carboxylation of glutamate side chains
Carboxylation of glutamate side chains

Vitamin K is essential for the normal synthesis of prothrombrin and several other clotting factors in the liver. It acts as a coenzyme for the carboxylation of glutamate side chains (Choice E) into 6 - carboxyglutamate. Without this vitamin K-independent carboxylation reaction, the prothrombin formed does not bind ca2+ and may result in hemorrhage and death. Choice A through D are incorrect as vitamin K is not the coenzyme for any of the reactions listed.
A disease that results in the destruction of a large number of cells and their nuclei would be expected to cause an increased blood level of which of the following?
A. Ammonia
B. DNA
C. Ketones
D. Mitochondria
E. Uric acid
Uric Acid

When a cell is broken down, the majority of the cells constituents, such as proteins and sugars, are catabolized into non-toxic waste products. However, when DNA and RNA are catabolized only the sugars and pyrimidines are able to be used for energy production. The purines (adenine and guanine) cannot be catabolized; instead they are deaminated and excreted as uric acid. Uric acid is a relatively non-toxic nitrogenous waste product that is non-soluble in blood. When large amounts of uric acid are produced, this waste product will begin to accumulate due to an inability of the kidneys to excrete this substance. Hyperuricemia is a clinical condition where the uric acid levels in the blood are greater than 7.5 mg/dL. This condition often develop into clinical state called gout. Since ammonia (choice A) is not found in the interior of a cell, blood levels of ammonia would not be expected to increase. DNA (choice B) blood levels would not increase in a disease that results in the destruction of a large number of cells and their nuclei because this compound is catabolized into waste products and uric acid. Ketones (choice C) are produced when lipids and amino acids are catabolized. Although mitochondria (choice D) pre found in the inside of a cell, they would also be catabolized when the cell is destroyed.
A lack of exercise has which of the following physiological effects on bone?
A. Bones are able to store more calcium
B. Bones become thin and brittle
C. Bones increase in length
D. Exercise or lack thereof has no effect on bone
E. The thickness-of the bone increases
Bones become thin and brittle.

Since bones are adaptable, their shapes often reflect the forces applied to them on a regular basis. For example, the various bumps and ridges on the surface of bone often indicate the sites where tendons attach to the bone. As the muscle becomes stronger and the force applied to the bone becomes larger, these bumps and ridges will become larger to withstand the increased force applied to them. Conversely, during periods of inactivity the size of the bone will decrease and the bone will subsequently become thin and brittle. In general, bones are able to store more calcium (choice A), increase in length (choice C), and the thickness of the bone will increase in periods of activity (choice E). Based on this information, it is easy to conclude that exercise or lack thereof does have an effect on bone (choice D).
A man is being chased by a vicious dog. Secondary to stimulation of the sympathetic nervous system, which of the following would most likely occur?
A. Decreased heart rate
B. Dilation of the peripheral blood vessels
C. Dilation of the pupils
D. Increased peristalsis in the small intestine
E. lncreased secretions of the salivary glands
Dilation of the Pupils

The sympathetic nervous system promotes the "fight or flight" reaction. During periods of stress, the sympathetic nervous system will help the body handle the stressor by promoting a series of physiologic changes. For example, one would expect to see dilation of the pupils, secretion of the sweat glands, increased heart rate, constriction of the blood vessels to nonessential organs, bronchodilation, and decreased peristalsis. All the other answer choice decreased heart rate (choice A), dilation of the peripheral blood vessels (choice B), increased peristalsis in the small intestine (choice D), and increased secretions of the salivary glands (choice E) would be expected to occur with the stimulation of the parasympathetic nervous system.
A muscle devoid of tonus is
1. atonic.
2. spastic.
3. hypotonic.
4. hypertonic.
atonic.

This is a simple definition-type problem. An atonic muscle will be flaccid. The muscle will be freely movable, and lies without tone. This occurs with lower motor neuron lesions; that is, the destruction of the alpha motor neurons. Spasticity is a hypertonus state of the affected muscles. This is due to a release of the tonic inhibition of the brainstem, facilitory information which leads to gamma motor neuron excitation. Thus, hyperactivity of the gamma fibers causes exaggerated response to stretch of muscle spindles. Hypotonia is simply a decrease in alpha motor neuron stimulation; however, not a complete destruction. So therefore we have partial tone loss. So again, the correct choice to question is #1, atonic.
A patient diagnosed with cystinuria will have difficulties reabsorbing all of the following in the renal tubules EXCEPT
A. arginine
B. cysteine
C. lysine
D. methionine
E. ornithine
Methionine

The transport systems for amino acids in the intestinal and luminal cells are similar to those for glucose, where transport across the luminal membrane is sodium dependent and transport across the contraluminal membrane is sodium independent. Genetic deficiencies in a variety of these carrier systems have been identified. For example, cystinuria is a clinical condition arising from faulty transport mechanisms. It is an autosomal-recessive disease caused by a defect in the transport of cysteine, ornithine, arginine, and lysine, resulting in these amino acids being excreted in the urine. A mnemonic: cystinuria results from an inability to reabsorb COAL (cysteine, ornithine, arginine, and lysine) in the renal tubules.
A patient presents with dry skin, cold intolerance, constipation, muscle weakness, recent weight gain, chronic lethargy, easy fatigue, loss of ambition, and periorbital puffiness. This patient's condition is most likely caused by which of the following?
A. Aldosterone hypersecretion
B. Androgen hypersecretion
C. Hypercortisolism
D. Hypocorticoidism
E. Hypothyroidism
Hypothyroidism

Hypothyroidism, or myxedema, is caused by a lack of thyroid function. Patients with this condition often present with dry skin, cold intolerance, constipation, muscle weakness, recent weight gain, chronic lethargy, easy fatigue, loss of ambition, and periorbital puffiness. Aldosteronism is caused by a hypersecretion of aldosterone (choice A), leading to hypokalemia, which can eventually lead to periods of muscle paralysis. Adrenogenital syndrome is the result of excessive production of androgens (choice B), leading to premature masculinizing effects in young males. Cushing syndrome, which is caused by a hypersecretion of cortisol (choice C) by the adrenal cortex, is characterized by an increased fat deposition in the thoracic region and an edematous appearance of the face; it may also cause acne and hirsutism. Addison disease results from the failure of the adrenal cortex to produce adrenocortical hormones, resulting in glucocorticoid (choice D) and mineralocorticoid deficiency. Loss of cortisol (a glucocorticoid) secretion results in the depression of many metabolic functions and decreases one's ability to tolerate various stressors.
A person who is severely deficient in folic acid would most likely experience which of the following?
A. Acidosis
B. Anemia
C. Hemorrhage
D. Seizure
E. Thrombocytopenia
Anemia

Anemia is a condition marked by a reduction in the hemoglobin and/or hematocrit content of the blood; this condition can be caused by a number of factors, one of which includes folic acid deficiency. Folic acid is the generic term for pteroylmonoglutamic acid, which serves as an important mediator of many reactions involving one-carbon transfers. This substance is found in many fruits and vegetables, especially citrus fruits and green leafy vegetables. The most common cause of folate deficiency is inadequate dietary intake and individuals most commonly seen with folate deficiency are alcoholics, the elderly and anorexic patients. The most common signs and symptoms of folic acid deficiency are megaloblastic anemia, with changes in the mucosa. The diagnosis is made upon finding macrocytic anemia with macroovalocytes and hypersegmented neutrophils on peripheral blood smear. Acidosis (choice A) is a condition where the blood pH is less than 7.4 and is not related to folic acid content in the body. Hemorrhage (choice C) can be caused by a number of factors, including vitamin K deficiency. Adequate amounts of vitamin K must be present for the liver to be able to synthesize four clotting factors, which include prothrombin. Without vitamin K, the entire clotting system will be inactivated. Therefore, if an individual were vitamin K deficient, they would most likely experience some degree of hemorrhage because the body would be unable to stop bleeding in a traumatic event. A seizure (choice D) can be defined as a condition characterized by a sudden onset of convulsions or other epileptic symptoms; its incidence is not increased in folic acid deficiency. Thrombocytopenia (choice E) is characterized by an abnormally low platelet count; its incidence is also not increased in folic acid deficiency.
A protein in solution is at its isoelectric point when the
A. pH is the same as that of the blood.
B. logarithm of the concentration is zero.
C. pH allows for maximum solubility.
D. pH is such that no migration occurs during electrophoresis.
E. pH is produced by a 1:1 mixture of the protein solution with isotonic saline.
pH is such that no migration occurs during electrophoresis.

The isoelectric point is the pH reached when there is no net electric charge on a protein. In other words, no migration of a protein in an electric field occurs during electrophoresis (Choice D) Choices A, B, C, and E do not describe isoelectric point
A reduction in the population of which of the following lymphocytes would impair all aspects of an immune response?
A. B cells
B. Cytotoxic T cells
C. Helper T cells
D. Memory T cells
E. Suppressor T cells
Helper T cells

Specific immunity is provided by the coordinated activities of both the T cells and B cells when antigens invade the human body. T cells play a crucial role in the initiation, maintenance and control of the immune response. Helper T cells stimulate the responses of both the T cells and B cells. They are absolutely essential to the immune response since B cells and other T cells must be activated by the helper T cells. Without helper T cells, an immune response to a given pathogen will not be elicited. The immunodeficiency caused by the AIDS virus occurs secondary to a reduction of helper T cells. B cells (choice A) are responsible for launching a chemical attack on antigens through the production of antibodies; without B cells an immu.ne response can still be elicited by T cells. Cytotoxic T cells (choice C) are responsible for cell mediated immiinity; these cells provide a direct physical and chemical attack on antigens. Memory T cells (choice D) remain long after an antigen enters the body. These cells ensure that an immediate immune response is elicited when the "same" antigen enters the body again. The suppressor T cells (choice E) inhibit both T cell and B cell activities; they moderate the immune response.
Acidosis is most likely to occur in patients severely deficient in which of the following hormones?
A. Antidiuretic hormone
B. Calcitonin
C. Insulin
D. Prolactin
E. Thyroxine
Insulin

Ketoacidosis, a specific form of acidosis, is most commonly seen in diabetic patients deficient in insulin. When an individual is insulin deficient, the cells of the body are unable to properly utilize the glucose in the blood. Therefore, the body breaks down fatty acids in an effort to maintain normal physiological functions. When these fatty acids are broken down, large numbers of ketone bodies are formed and the blood pH decreases, leading to systemic acidosis. Antidiuretic hormone (choice A) prevents excessive water loss. In the absence of the hormone dehydration can occur. Calcitonin (choice B) is primarily responsible for the regulation of calcium ion concentrations in the blood. When blood levels calcium ions become elevated, calcitonin is secreted to reduce calcium ion levels to normal limit. Deficiencies of calcitonin would result in the development of hypercalcemia. Prolactin (choice D) is responsible for the production of milk in the mammary glands of women after pregnancy. If the hormone were deficient in a "new" mother, milk production would be substantially decreased. The thyroid hormones, such as thyroxine (choice E), are responsible for the elevation of oxygen consumption and the rate of energy consumption; increased heart rate; stimulation of activity of other endocrine tissue; maintenance of sensitivity of respiratory centers as well as various other functions. Individuals deficient in thyroid hormones are typically lethargic, unable to adjust to cold temperatures, have dry skin and some degree of hair loss, muscle weakness and slowed reflexes
ADH receptors in the nephron are located on the tubular membrane of which of the following?
A. Distal tubule
B. Proximal tubule
C. Ascending loop of Henle
D. Descending loop of Henle
Distal Tubule

The nephron is the functional unit of the kidney. It is organized into different components each with specialized functions. ADH receptors are located on the distal tubule (Choice A) portion of the nephron. The distal tubule is responsible for the reabsorption if ions such as sodium, potassium, and chloride. Under the influence of ADH, the distal tubule can also dilute or concentrate the urine by regulating the amount of water entering into its lumen. Choice B, proximal tubule, functions mainly in the reabsorption of glucose. No ADH receptors are present in this segment of the nephron. Choice C, ascending loop of Henle, is virtually impermeable to water and it's also responsible for reabsorption of ions. Choice D the descending loop of Henle is highly permeable to water and functions mainly to allow simple diffusion of substances through its walls. Like the absorption part of the loop, it does not posess ADH receptors
Which of the following characterize the links between monomeric units of nucleic acids?
1. Ionic bonds
2. Glycosidic bonds
3. Phosphodiester bonds
4. Phosphotriester bonds
The correct answer is choice 3. Monomeric units of DNA consist of a phosphate group, deoxyribose, sugar, and nitrogen base of adenine, guanine, cytosine or thymine. These monomeric units are linked together to form the DNA strand by phosphodiester bonds. In these bonds, the 3-prime carbon of one deoxyribose links with one oxygen of the phosphate group, while a 5-prime carbon of another deoxyribose links with another oxygen of the phosphate group. This is known as a phosphodiester bond, indicating two bonds to a phosphate group. So the correct answer to question is choice 3.
Albinism is a genetic disease that results in incomplete metabolism of
1. alanine.
2. tyrosine.
3. cysteine.
4. histidine.
5. tryptophan.
tyrosine

Albinism is a condition in which melanin is not produced and pigment is absent from skin, hair and the iris of the eyes. It is due to the absence of the enzyme tyrosinase. Tyrosinase is a copper-containing enzyme which acts on tyrosine to form dopaquinone which cyclizes and eventually forms melanin. Melanin granules are generally both dark and insoluble and are composed of high molecular weight polymers. Anyway, we need tyrosine for tyrosinase to act on to form melanin and the correct answer to question is choice 2.
Aldosterone is normally associated with partial regulation of which of the following processes?
1. Sodium balance
2. Gluconeogenesis
3. Lipid digestion
4. Protein degradation
5. Carbohydrate metabolism
sodium balance.

Aldosterone is a mineralocorticoid which is synthesized by the zona glomerulosa of the adrenal cortex. Aldosterone increases the resorption of sodium from gastric secretions, sweat, urine and saliva. Aldosterone acts on the epithelium of the distal tubule and collecting duct in the kidney. It does three important things: (1) it increases the resorption of sodium (2) promotes secretion of potassium and (3) promotes the secretion of hydrogen. Sodium retention occurs with excessive secretion of aldosterone. Along with increased sodium retention goes expansion of extracellular fluid volume, depletion of potassium and consequently metabolism alkalosis. Deficiency of aldosterone results in loss of sodium, hyperkalemia and acidosis. The renin angiotensin system regulates aldosterone. Angiotensin II stimulates secretion of aldosterone. Aldosterone secretion also increases with increasing plasma potassium concentrations and decreases with decreasing plasma potassium concentrations. Again, the correct choice is #1.
`All of the following are associated with the action of postganglionic sympathetic fibers EXCEPT
A. Constriction of pupils
B. Increased heart rate
C. Increased sweat secretion
D. Reduction of blood flow to the skin
E. Release of stored lipids from subcutaneous adipose tissue
Pupil Constriction

The major actions of the postganglionic sympathetic fibers are as follows: dilation of the pupils (NOT constriction of the pupils) and focusing on distant objects; acceleration of the heart rate (choice B) and increased force of contraction; increased sweat secretion from the sweat glands (choice C); constriction of cutaneous blood vessels leading to a reduction of blood flow to the skin (choice D); and release of stored lipids from subcutaneous adipose tissue (choice E).
All of the following are derivatives of the amino acid tyrosine EXCEPT
A. Dopamine
B. Epinephrine
C. Melatonin
D. Norepinephrine
E. Thyroxine
Melatonin

Some hormones are small molecules that are structurally related to amino acids. For example, the following are derivatives of the amino acid tyrosine: dopamine (choice A), epinephrine (choice B), norepinephrine (choice D) and the thyroid hormones, thyroxine (choice E) and triiodothyronine. With respect to the synthesis of norepinephrine, the following steps apply. The synthesis of norepinephrine begins in the axoplasm of the terminal nerve endings of adrenergic fibers and is completed inside the vesicles of these fibers. The basic steps in the synthesis of norepinephrine are as follows: Tyrosine is converted to DOPA through the process of hydroxylation and then DOPA undergoes decarboxylation to become dopamine. Dopamine is then transported into the vesicles of the adrenergic fibers where it undergoes hydroxylation to become norepinephrine. In the adrenal medulla, norepinepherine is transformed into epinephrine through the process of methylation. Choline is combined with Acetyl-CoA to become acetylcholine. Melatonin (choice C) is a derivative of the amino acid tryptophan.
All of the following are essential amino acids in adults EXCEPT
A. arginine
B. isoleucine
C. leucine
D. methionine
E. tryptophan
Arginine

An essential amino acid is one that cannot be synthesized in adequate amounts to meet the needs of the cell and therefore must be supplied by dietary protein. There are 10 essential amino acids: arginine, histidine, isoleucine, leucine, lysine, methionine, phenylalanine, threonine, tryptophan, and valine. Two of these (arginine and histidine) are considered "essential" only in infants and children. A mnemonic to help remember these amino acids is as follows: "Any help in learning these little molecules proves valuable."
OR
Essential Amino Acids
PriVaTe TIM HALL
Phe, Val, Thr, Trp, Ile, Met, His, Arg, Leu, Lys
Nitric oxide:What is the amino acid precursor?

(Extra)
Arginine

When the dentist works on your teeth, you say, "AAArg! (Arginine)" before he administers Nitric Oxide (NO) to take the pain away.

Other players necessary for NO synthesis: NO synthase, Ca++, NADPH.
All of the following are the result of parasympathetic stimulation EXCEPT
A. Contraction of the urinary bladder during urination
B. Constriction of the pupils
C. Increased heart rate
D. Increased salivary gland secretion
E. Increased smooth muscle activity of the intestinal tract
Increased HR

The parasympathetic and sympathetic nervous systems, for the most part, oppose each other. The major actions of the parasympathetic nervous system are as follows: Decreased heart rate (NOT increased heart rate); Contraction of the urinary bladder during urination (choice A), constriction of the pupils (choice B) increased salivary gland secretion (choice D) and increased smooth muscle activity of the intestinal tract (choice E). Other parasympathetic actions include secretion of hormones that promote the absorption and utilization of nutrients by peripheral cells, stimulation and coordination of defecation, constriction of the respiratory pathways and sexual arousal and stimulation of sexual glands in both sexes.
All of the following changes in the skeleton are considered age-related EXCEPT
A. Appearance of major vertebral curves
B. Appearance of secondary dentition
C. Decrease in mineral content of the bones
D. Fusion of epiphyseal plates
E. Remodeling of the bone
Remodeling of bone

A review of the human skeleton can reveal important information about an individual's age and/or sex. However, there are certain characteristics of the human skeleton that are independent of an individuals age or sex. For example, bone remodeling is a process that occurs throughout a persons life at a relatively constant rate. In the "remodeling" process, the matrix of the bone is replaced while leaving the bone as whole unchanged. With respect to individual variations in the skeletal system, the following principles apply: The appearance of major vertebral curves occur between the age of 2 fetal months and adulthood (choice A); The fetal spine is "bow" shaped and the adult spine is more "S" shaped. The appearance of secondary dentition (choice B) , occurs when the primary teeth are lost and the permanent teeth take their place. The mineral content of bone (choice C) typically decreases after the age of 65. The fusion of epiphyseal plates (choice D) typically occurs near the beginning of adulthood.
All of the following hormones involved in the regulation of bone growth stimulate osteoblast activity and the synthesis of bone matrix EXCEPT
A. Estrogen
B. Growth hormone
C. Parathyroid hormone
D. Testosterone
E. Thyroxine
Parathyroid Hormone
Normal bone growth and maintenance depend on a combination of hormonal and nutritional factors. With respect to the hormonal factors, the only hormone involved in the regulation of bone growth, listed as an answer choice, that does not stimulate osteoblast activity and the synthesis of bone matrix is parathyroid hormone. Parathyroid hormone is responsible for stimulation of osteoclast activity and elevation of calcium ion concentrations in body fluids. Estrogen (choice A) secreted in the ovaries, Growth hormone (choice B) secreted from the pituitary gland, Testosterone (choice D) secreted from the testes, and Thyroxine (choice E) secreted from the follicle cells of the thyroid gland, are all responsible for stimulating osteoblast activity and synthesis of bone matrix to either a small or large degree.
All of the following hormones promote protein synthesis EXCEPT
A. Cortisone
B. Estrogen
C. Insulin
D. Growth hormone
E. Testosterone
Cortisone

A summary of protein synthesis is as follows: An mRNA strand contains codons that are complementary to the DNA strand. The molecules of tRNA contain the anticodons that are complementary to the mRNA codons. Different amino acids are then delivered by the tRNA's. The sequence of amino acids in the completed peptide will reflect the sequence of the tRNA arrivals. The human body then uses these amino acids to build proteins. Cortisone is a glucocorticoid hormone that has the following effects on the peripheral tissues: It decreases the use of glucose and releases amino acids from skeletal muscles and lipids from adipose tissue. Cortisone also has anti-inflammatory effects. Estrogens (choice B) and androgens, such as testosterone, (choice E) are "sex hormones" that increase the rate of amino acid usage in protein synthesis. Insulin (choice C) is primarily responsible for increasing glucose uptake and utilization in the liver, adipose tissue and skeletal muscle. Insulin also promotes increased amino acid uptake and protein synthesis. This is also one of the primary functions of growth hormone (choice D).
All of the following statements are true with respect to blood pressure and resistance to blood flow EXCEPT
A. blood pressure increases with a decrease in arterial elasticity
B. resistance to blood flow is directly related to the blood viscosity
C. resistance to blood flow is directly related to vessel diameter
D. stroke volume is directly related to blood pressure
E. vascular resistance is directly proportional to the length of the blood vessel
resistance to blood flow is directly related to vessel diameter

With respect to the vascular system, resistance represents the opposition to the flow of blood. A number of factors can affect blood flow, such as vessel diameter, arterial elasticity, blood viscosity, and vessel length. As the vessel diameter increases, resistance to blood flow will decrease. In other words, resistance to blood flow is inversely, not directly, related to vessel diameter.
All of the following statements are true with respect to deglutition EXCEPT
A. approximately one-twelfth of all "swallows" that occur daily occur during sleep
B. food is moved to the esophagus under the direction of cranial nerve XII
C. involuntary contraction of the pharynx advances the bolus into the esophagus
D. the nucleus of the solitary tract coordinates the swallowing mechanism
E. swallowing is triggered by cranial nerves VIII and IX
swallowing is triggered by cranial nerves VIII and IX (is incorrect)

Swallowing is a reflex action with its afferent impulses carried primarily by the glossopharyngeal (CN IX) and vagus (CN X) nerves. The act of swallowing is triggered by cranial nerves IX and X, not VIII and IX. All the other statements about deglutition are true. Approximately one twelfth of all daily "swallows" occur during sleep (choice A), and one-third occur during eating. Food is moved to the esophagus under the direction of cranial nerve XII (choice B). Involuntary contraction of the pharynx advances the bolus into the esophagus (choice C). The nucleus of the solitary tract coordinates the swallowing mechanism (choice D) via the nucleus ambigus and hypoglossal nucleus.
All of the following statements are true with respect to proteins EXCEPT:
A. Proteins are the structural components of androgens
B. Proteins are the structural components of antibodies
C. Proteins consist of small organic molecules called amino acids
D. Proteins help to create the threedimensional framework of the body
E. Protein structures transport materials around the body
Proteins are the structural components of androgens

Proteins perform a variety of essential functions in the body; these functions include the following: The proteins of the immune system are known as antibodies. In other words, proteins are the structural components of antibodies (choice B). Proteins consist of small organic molecules called amino acids (choice C). The human body contains significant quantities of about 20 different amino acids. In fact, these proteins help to create the three-dimensional framework of the body (choice D). Insoluble lipids, respiratory gasses, special minerals, such as iron and several hormones are carried throughout the body attached to special transport proteins (choice E). Cholesterol is the structural component of androgens (choice A).
All of the following would lead to an increase in the glomerular filtration rate EXCEPT
A. Constricting the efferent arterioles
B. Decreasing plasma protein concentrations
C. Decreasing renin levels
D. Dilation of the afferent arteriole
E. Inhibition of sympathetic stimulation of the kidney
Decreasing renin levels

The glomerular filtration rate is defined as the amount of filtrate produced in the kidneys each minute, which averages around 125 mL of filtrate per minute. There are many different factors that can alter (increase or decrease) the glomerular filtration rate. For example, when renin levels are decreased (choice C), there is a decreased conversion of angiotensin I to angiotensin II. Since angiotensin II is a potent constrictor of the efferent arteriole, decreasing renin will decrease the GFR. Constricting the efferent arterioles choice A) and dilation of the afferent arteriole (choice D) will directly increase the glomerular filtration rate. Conversely, constriction of the afferent arteriole often occurs when there is a fall in blood pressure, which will lead to a decrease in the glomerular filtration rate. The constriction of the afferent arteriole occurs when there is sympathetic stimulation of the kidney; therefore, inhibition of sympathetic stimulation of the kidney (choice E) will cause glomerular filtration rate to increase. When plasma protein concentrations are decreased (choice B), the kidneys will increase the glomerular filtration rate to increase the production of urine.
An alcoholic man is diagnosed with Wernicke triad and Korsakoff syndrome. Which of the following would be the most appropriate treatment for this patient?
A. Biotin
B. Niacin
C. Pyridoxine
D. Riboflavin
E. Thiamine
Thiamine

The diagnosis of classic Wernicke triad (ophthalmoplegia, ataxia, and global confusion) and Korsakoff psychosis is considered to be a medical emergency that necessitates the immediate administration of thiamine, or vitamin B1. Biotin (choice A) acts as a carrier of "activated carboxyl" groups for three key enzymes that catalyze carboxylation reactions. Symptoms of biotin deficiency are alopecia, skin and bowel inflammation, and muscle pain. Niacin (choice B), or vitamin B3, is converted to nicotinamide, which is then incorporated into the coenzymes NAD+ and NADP+. Pellagra is a condition associated with a deficiency of niacin, which can result in the development of diarrhea, dermatitis, and dementia. Pyridoxine (choice C), or vitamin B6, is associated with the coenzyme pyridoxal phosphate. A deficiency in pyridoxine can lead to the development of peripheral neuropathy and dermatitis. Riboflavin (choice D), or vitamin BP, is responsible for maintaining proper levels of both flavin mononucleotide (FMN) and flavin adenine dinucleotide (FAD). Individuals deficient in riboflavin are likely to present with lesions of the lips, mouth, skin, and genitalia.
An electrocardiogram showing extra P waves before each QRS complex indicates
1. fibrillation.
2. atrial bradycardia.
3. partial heart block.
4. ventricular tachycardia.
5. left bundle branch block.
Partial Heart BLock


In order to understand this question you must be familiar with the normal ECG tracings. The P-wave is the first wave of the ECG tracing, and it represents the spread of depolarization through the atria. The P-R interval is measured from the beginning of the P-wave to the beginning of the QRS complex. It reflects the time it takes for the depolarization impulse to travel from the S-A node to the ventricular myocardium. The P-R segment is measured between the end of the P-wave and the beginning of the QRS complex. It reflects the spread of depolarization through the A-V node, the bundle of His and the Purkinje fibers. The QRS complex is the second and most important deflection on the ECG tracing. It represents the spread of depolarization through the ventricular muscle. The ST segment is measured from the end of the QRS to the beginning of the T wave. It represents the portion of the cardiac cycle in which all the ventricular myocardial fibers are depolarized and no further current flows. The T-wave is the third deflection of the QRS and represents the recovery period or repolarization of the ventricles. The Q-T interval is measured from the beginning of the QRS to the end of the T-wave and gives the total duration of ventricular systole. Partial heart block represents a delay in the transmission of the action potential from the atrial pacemaker to the ventricles. This delay may be due to abnormalities in the atria or the A-V node or the bundle of His, and is reflected by prolongation of the P-R interval on the ECG. It is also characterized by extra P-waves before each QRS complex. So again, the correct choice to question is #3, partial heart block.
An impulse can travel from one nerve to another in only one direction because the
1. synapse limits the direction of travel.
2. myelin sheath limits the direction of travel.
3. myoneural junction limits the direction of travel.
4. nerve fiber permits conduction in only one direction.
5. cell body must be stimulated before the nerve fiber will conduct.
synapse limits the direction of travel

Action potentials are propagated along the axon. The electrical conduction between neurons occurs by gap junctions. Chemical transmission occurs at the synapse. The neurotransmitter, upon arrival of the impulse, is released by the presynaptic neuron and reacts with receptors on the postsynaptic neuron. The conductance for ions is then increased, and an electrochemical current is propagated. Since the presynaptic neuron releases the neurotransmitter upon arrival of the impulse the impulse is unidirectional. Gue to the synapse the myelin sheath limits the direction of lateral conduction to adjacent nerves. Conduction along the nerve fiber itself can be bidirectional, and the nerve conducts an impulse due to the amval of the action potential. So again, the correct choice is #1, synapse limits the direction of travel.
An individual deficient in which of the following is most likely to develop pernicious anemia?
A. Cholecystokin
B. Gastrin
C. Hydrochloric acid
D. Intrinsic factor
E. Secretin
Intrinsic Factor

Intrinsic factor is the secretion of parietal cells that facilitates the absorption of vitamin B12 across the intestinal lining. Vitamin B12 is a member of the cobalamin family and serves as a cofactor for two important reactions in humans. As methylcobalamin, it serves as a cofactor for methionine synthetase in the Conversion of homocysteine to methionine, and as idenosylcobalamin, it serves as a cofactor for the snversion of methyl-malonyl-CoA to succinyl-CoA after being ingested, it becomes bound to intrinsic factor, which is a protein secreted by gastric parietal cells. Once absorbed, it is stored in the liver. The most common disorder seen with Vitamin B12 deficiency is pernicious anemia, which is associated with the development of a megaloblastic anemia. This megaloblastic state often leads to various mucosal changes, intestinal disturbances, such as anorexia and diarrhea, and a neurological syndrome where the peripheral nerves are often affected first, causing paresthesias and difficulty i n balance. Cholecystokinin (choice A) is produced and stored in the I cells of the duodenal and jejunal mucosa. The parietal cell secretes two substances: intrinsic factor and hydrochloric acid. The hormone gastrin (choice B), which is produced in the G cells of the duodenum, primarily functions to stimulate hydrochloric acid, histamine, and pepsinogen secretion as well as increase gastric blood flow. Hydrochloric acid (choice C) functions primarily to denature proteins and activate digestive enzymes. Secretin (choice E) is synthesized and stored in the S cells of the upper intestine. It stimulates the secretion of bicarbonate containing substances from the pancreas and inhibits gastric emptying and gastric acid production.
An individual that has anti-A agglutinins and Rh agglutinogens would have which of the following classifications of blood?
A. Type A negative
B. Type A positive
C. Type B negative
D. Type B positive
E. Type AB positive
Type B +

There are two antigens (type A and type E) that appear on the surfaces of red blood cells. Furthermore, strong antibodies that react specifically with either the type A or type B antigens, occur in individuals who do not have these particular antigens on their cells. Hence, type A and type B antigens are called agglutinogens and the antibodies that cause agglutination are called anti A/B agglutinins. Along the same lines, individuals who have the Rh agglutinogen will be Rh+, and those without the Rh agglutinogen will produce the anti-Rh agglutinin. Remember that agglutinogens are antigens, and agglutinins are antibodies. An individual that has anti-A agglutinins and Rh agglutinogens would have a Type B (+) blood classification. One would expect to see antiB agglutinin and no Rh agglutinogen in a person with type A (-) blood (choice A), conversely anti-B agglutinin and Rh agglutinogen (choice B) is seen in persons with A (+) blood. Anti-A agglutinin and no Rh agglutinogen (choice C) is seen in B (-) blood types. When there are no anti-A or anti-B agglutinins (the person has both agglutinogens) and there is Rh agglutinogen, the person has blood type of AB positive (choice E).
An individual who is chronically lethargic, unable to adjust to cold temperatures and has chronic dry skin is most likely deficient in of which of the following hormones?
A. Calcitonin
B. Growth hormone
C. Parathyroid hormone
D. Testosterone
E. Thyroid hormone
Thyroid Hormone

Adults with hypothyroidism are chronically lethargic, unable to adjust to cold temperatures and often have chronically dry skin as well as have some degree of hair loss, muscle weakness and slowed reflexes. In other words, the patient in this question is most likely deficient in their thyroid hormones, since the effects of thyroid hormones are as follows: Elevation of oxygen consumption and the rate of energy consumption; increased heart rate; stimulation of activity of other endocrine tissues; maintenance of sensitivity of respiratory centers as well as various other functions. Calcitonin (choice A) is the hormone primarily responsible for the regulation of calcium ion concentrations in the blood. When blood levels of calcium ions become elevated, calcitonin is secreted to reduce calcium ion levels to normal limits; Therefore, deficiencies of calcitonin would result in the development of hypercalcemia. Growth hormone (choice B) stimulates cell growth and replication by accelerating the rate of protein synthesis. Individuals deficient in growth hormone are often described as "pituitary dwarfs". Parathyroid hormone (choice C) is responsible for increasing calcium ion concentrations in the body fluids; therefore individuals deficient in this hormone would most likely be hypocalcemic. Testosterone (choice D) promotes the production of functional sperm, maintains the secretory glands of the male reproductive tract and stimulates growth. Deficiencies of testosterone are associated with a decreased development of male secondary sex characteristics.
An inflammatory response is triggered when which of the following occur?
A. Blood flow to an area increases
B. Clot formation occurs
C. Fibroblasts secrete beta-interferon
D. Mast cells release histamine
E. Neutrophils phagocytize bacteria
Mast cells release histamine

Inflammation is a localized tissue response to injury that produces various local sensations, such as swelling, heat, redness and pain. There are many different factors that can cause inflammation, including abrasion, impact, irritation, and infection. Mast cells play an important role in the inflammation process. These cells release histamine, prostaglandins, heparin and other chemicals into the immediate area. When histamine is released, capillary permeability is increased and blood flow to the area increases. Although blood flow to the area (choice A) is the most common cause of inflammation, the inflammatory response is triggered when mast cells release histamine. Clot formation (choice B) is one of the final states of tissue repair; it does not promote inflammation. When fibroblasts secrete beta-interferon, this substance slows the inflammation process in damaged tissue. When neutrophils phagocytize bacteria (choice E), inflammation is not promoted.
Antidiuretic hormone acts to
1. decrease renal filtration fraction.
2. increase storage capacity of the bladder.
3. decrease permeability of distoconvoluted tubules and/or collecting ducts to water.
4. increase permeability of distoconvoluted tubules and/or collecting ducts to water.
increase permeability of distal convoluted tubules and/or collecting ducts to water.

When plasma osmotic pressure changes the brain's osmoreceptors respond by stimulating or inhibiting antidiuretic hormone, abbreviated ADH. Axons of neurons in the supraoptic and paraventricular nuclei of the anterior hypothalamus extend down the pituitary stalk to the posterior lobe of the pituitary, where the axon terminals release secretory products into the blood. Vasopressin, or ADH, is one of the two hormones released. The other is oxytocin. ADH binds to specific cell receptors of the distal convoluted tubule and the collecting duct, activating adenylate cyclase; CAMP is produced, followed by activation of protein kinase. Water then flows across the cell, due to the phosphorylation of lumenal membrane components. Urine flow will decrease, and the osmolarity of bloodwill drop. In the presence of ADH the kidney will produce a small amount of very concentrated urine. Again, the correct choice is #4, increase permeability of the distal convoluted tubules and/or collecting ducts to water.
Which of the following enzymes converts trypsinogen to trypsin?
A. Enterokinase
B. Peptidase
C. Secretin
D. Pepsin
The correct answer is Choice A. Trypsin is a powerful proteolytic enzyme present in pancreatic secretions. It aids in the digestion of proteins. When synthesized in the pancreatic cells, the proteolytic enzyme is in its inactive form trypsinogen. It is important for the enzyme to remain inactive as it will digest the pancreas itself. Trypsinogen is activated to trypsin only after it is released into the intestinal tract by enterokinase (Choice A). Enterokinase is secreted by the intestinal mucosa when chyme comes into contact with the mucosa.
Peptidase (Choice B) is an enzyme present in the microvilli of the small intestine capable of splitting large polypeptides into their components.
Secretin (Choice C) is a hormone secreted by duodenal and jejunal mucosa in response to high acid content in the small intestine. This hormone in turn stimulates pancreatic secretion of sodium bicarbonate.
Pepsin (Choice D) is the major digestive enzyme of the gastric juices. It is active in an acidic environment to digest proteins and collagen.
Which of the following enzymes is involved in the synthesis of fatty acids?

A. Acetyl CoA carboxylase
B. Glucose 6-phosphate dehydogenase
C. Glycogen synthase
D. Phosphorylase
E. Thiolase
The correct answer is A. The synthesis of lipids is known as lipogenesis. Pathways of lipid synthesis begin with acetyl-CoA; these molecules can be stmng together in the cytosol yielding fatty acids. Acetyl CoA carboxylase is the key enzyme involved in fatty acid synthesis. Glucose 6- phosphate dehydogenase (choice B) is involved in the pentose phosphate pathway. The two enzymes involved in the synthesis and degradation of glycogen are phosphorylase and glycogen synthase. Furthermore, glycogen synthesis and degradation are coordinated by a hormone-triggered cascade, which ensures that when one enzyme is active, the other enzyme is inactive. The enzyme involved in glycogen degradation is phosphorylase (choice D) and the enzyme involved in glycogen synthesis is glycogen synthase (choice C), as the name implies. Thiolase (choice E) is responsible for converting acetoacetyl CoA into acetyl CoA
Which of the following functions as part of the extracellular matrix?

A. Mucin
B. Heparin
C. Collagenase
D. Chondroitin sulfate
E. Dolichol phosphate
The correct answer is Choice D. Of the following, only chondroitin sulfate (Choice D) functions as part of the extracellular matrix. It is a proteoglycan that acts as a filler between collagen fibers and cells. Chondroitin sulfate is also a principal component in the organic matrix of bone and cartilage.
Mucin (Choice A) is a mucus secretion produced by salivary glands that acts as a lubricant for surface protection.
Heparin (Choice B) is a powerful anticoagulant produced mainly by the basophilic mast cells located in the pericapillary connective tissues.
Collagenase (Choice C) is an enzyme that catalyzes the hydrolysis of collagen. It is not a functional part of normal extracellular matrix.
Dolichol phosphate (Choice E) is a very long chain lipid that acts as a carrier for oligosaccharide.
Which of the following groups includes only amino acids essential for humans?

A. Valine, serine, leucine
B. Leucine, lysine, glycine
C. Tyrosine, threonine, tryptophan
D. Phenylalanine, methionine, proline
E. Tryptophan, methionine, isoleucine
The correct answer is Choice E. Amino acids are the basic building blocks of proteins. They are classified into two separate groups, the essential and the nonessential amino acids. The nonessential group includes amino acids that the body can synthesize. The essential amino acid group include ones that the body either cannot synthesize or are synthesized in amounts too small to meet the needs of the body. These amino acids must be supplied in the diet. The following is a list of essential amino acids:
1. Threonine
2. Lysine
3. Methionine
4. Arginine
5. Valine
6. Phenylalanine
7. Leucine
8. Tryptophan
9. Isoleucine
10. Histidine
Of the choices listed, Choice E (tryptophan, methionine, isoleucine) contains only essential amino acids.
Which of the following hormones are produced by the neurohypophysis and affect contraction of smooth muscle?

1. Vasopressin (ADH) and oxytocin
2. Thyrotropin and intermedin
3. Adrenotropin and somatotropin
4. Aldosterone and luteinizing hormone
The correct answer is choice 1. The neurohypophysis is another term for the posterior pituitary. The two major hormones produced by the posterior pituitary are ADH, also known as vasopressin, and oxytocin. ADH raises blood pressure and helps retain fluid by increasing permeability of collecting tubules in the kidney. Oxytocin causes uterine contraction and milk secretion. In choice 2, thyrotropin or TSH comes from the anterior pituitary and stimulates the thyroid gland; while intermedin or melanotropin is an anterior pituitary hormone that stimulates melanin production. And choice 3, adrenotropin or ACTH is another anterior pituitary hormone that stimulates the adrenal cortex, while somatotropin or growth hormone is another anterior pituitary hormone that stimulates overall body growth. And choice 4, aldosterone is an adrenal cortical hormone primarily responsible for sodium retention, while luteinizing hormone is an anterior pituitary hormone involved in triggering ovulation and formation of the corpus luteum. So the correct answer to question is choice 1.
Avidin influences which of the following vitamins?
1. Niacin
2. Biotin
3. Thiamine
4. Tocopherol
5. Phylloquinone
Biotin

The correct answer is choice 2. Avidin, which is a glycoprotein found in egg white, can produce biotin deficiency as it binds biotin tightly, preventing its absorption from the intestine. It has no effect on any of the other choices. Choice 1, niacin, is involved in producing NAD and NADP and niacin deficiency leads to pellagra. Choice 3, thiamin, or vitamin B1 deficiency, causes beriberi. Thiamin acts as a co-enzyme in reactions involving aldehyde groups. Tocopherol, or vitamin E deficiency, is related to infertility, and vitamin E has antioxidant properties. Choice 5, phylloquinone, is a member of the quinone family. Quinones are involved in electron transport in the respiratory chain. So the correct answer to question is choice 2.

(A,D,E,K are the fat soluble vitamins)

(9 water soluble vitamins...8 B vitamins & Vit C)

Fat Soluble Vitamins are Stored in the liver while water soluble are not stored and get eliminated in urine leading to the need of a continous supply in our diet)
Bacteria in the intestinal tract are responsible for synthesis of all the following EXCEPT
A. ammonia
B. ascorbic acid
C. folic acid
D. small organic acids
E. vitamin K
Ascorbic Acid (Vitamin C)

In normal feces, 30% of the solid waste may be composed of bacteria. In the intestinal tract, bacteria split urea to form ammonia (choice A) and synthesize folic acid (choice C), small organic acids (choice D) from unabsorbed fat and carbohydrates, and vitamin K. The brown color of feces is mainly due to the action of bacteria on bile pigments. The odor of feces is due to sulfides and indolic compounds. Ascorbic acid (choice B), or vitamin C, is NOT produced by the bacteria in the intestinal tract.
Both control and regulation of sympathetic function primarily occur in centers in the
A. cerebellum
B. cerebrum
C. mesencephalon
D. posterior and lateral hypothalamus
E. spinal cord and adrenal medulla
E. spinal cord and adrenal medulla

The autonomic nervous system consists of two subdivisions, the sympathetic and parasympathet nervous systems. In summary, an increase sympathetic activity leads to stimulation of tissue metabolism, increased alertness and preparation of the body for an emergency. The sympathetic division consists of preganglionic neurons located between spinal segments TI and L2. Furthermore, the ganglionic neurons are also located in ganglia near the vertebral column. The center of each adrenal gland (adrenal medulla) is a modified sympathetic ganglion. In other words, both control and regulation of sympathetic function primarily occurs in centers in the spinal cord and adrenal medulla. The cerebellum (choice A) adjusts voluntary and involuntary motion activities as well as compares incoming sensory information with anticipated sensations. Consciousness, thought processes, sensations, intellectual function, memory storage and retrieval and complex motion actions originate in the cerebrum (choice B). The mesencephalon (choice C) is the center responsible for processing visual and auditory information. The posterior and lateral portion of the hypothalamus choice D) contain various centers involved with emotions, autonomic function and hormone production. Furthermore, the hypothalamus is the primary link between nervous and endocrine system.
Breathing a gas mixture with 5 percent CO2, ultimately leads to which of the following?
A. Hypoventilation
B. A decrease in heart rate
C. A stimulation of central chemoreceptors
D. An inhibition of peripheral chemoreceptors
E. A decrease in cerebrospinal fluid hydrogen-ion concentration
A stimulation of central chemoreceptors

C02 has a direct effect on the respiratory center of the brain in controlling respiration. Breathing a gas mixture with excess C02 will stimulate control chemoreceptors (Choice C) to cause increased strength of pulmonary ventilation, thus reducing C02 concentration by expiring greater amounts of C02 out of the body.
Choice A is incorrect as an excess amount of C02 will ultimately lead to hyperventilation as the lungs labor to rid the body of CO2.
Choice B is also incorrect as C02 concentration in the blood can stimulate peripheral chemoreceptors especially at the onset of exercise. It will not, however, inhibit peripheral chemoreceptors as stated in this choice. Choice E is incorrect as excess C02 can pass through the blood-cerebrospinal fluid barrier, react with water present to form hydrogen ions thus increasing the hydrogen ion concentration in the cerebrospinal fluid.
Calcium ions initiate contraction in skeletal muscle when they
A. bind to T tubules.
B. bind to troponin.
C. interact with actin.
D. interact with myosin.
E. bind to sarcoplasmic reticulum.
Bind to troponin

. Calcium ions bind strongly to troponin (Choice B) changing the condition between troponintropomyosin complex and actin in the relaxed muscle state to produce a new relationship that leads to muscle contraction.
T-tubule (Choice A) facilitates transmission of action potentials to all skeletal muscle fibers. It does not bind with calcium ions.
Calcium ions also do not bind with actin (choice C) or myosin (Choice D). Actin must attach to the heads of the myosin filaments to cause contraction.
Sarcoplasmic reticulum (Choice E) releases calcium ion in the vicinity of all myofibrils to bring about contraction.
Calcium ions trigger contraction of muscles by binding to
1. actin.
2. myosin.
3. troponin.
4. tropomyosin.
troponin

During muscular contractions, the heavy meromyosin subunit of myosin extends to reach and bind with the actin filament. This binding activates the ATPase activity of the myosin head and in the presence of ATP, a confirmational change may occur in the end causing a spring like shortening. This results in pulling the actin filament past the myosin filament. Tropomyosin on the actin molecule covers the binding site for myosin cross bridges in the absence of calcium ion. When calclum ion is present, it binds to troponin and exposes the binding site on the actin molecule for myosin to interact and allows the contraction cycle to occur. Choice 1 is incorrect. Actin is a major protein found in the thin filament of the myofibril. Choice 2 is incorrect. Myosin is large protein consisting of two sets of light chains and one set of heavy chain composing the thick filament. Choice 4 is incorrect. Tropomyosin is found in the thin filament. These rod shaped molecules are associated with actins to form the basic structural component of the thin filament. So the correct answer to question is choice 3.
Carbohydrate is stored in the body principally as
1. glucose.
2. maltose.
3. sucrose.
4. glycogen.
5. glycosaminoglycans.
glycogen.

Glycogen is stored in the liver. At times, when glucose is in demand glycogen will be converted into glucose via glycogenolysis, which is the lysis of glycogen, or stored carbohydrate. Epinephrine, or glucagon, stimulates the conversion of ATP to cyclic AMP via adenylate cyclase. Cyclic AMP then converts protein kinase inactive to protein kinase active. Protein kinase aids in the phosphorylation of phosphorylase kinase inactive to phosphorylase kinase active via ATP. The phosphorylase kinase active then converts to phosphorylase-B inactive to phosphorylase-A active. This then releases successive glucose-1-phosphate residues from the branch chains of glycogen. Thus, we have the production of glucose from our stored carbohydrate. So again, the correct choice is 4, glycogen.
Carbon monoxide decreases the amount of
1. bicarbonate in the blood plasma.
2. carbonic anhydrase available in the lungs.
3. oxygen that can be transported by hemoglobin.
4. alveolar surface available for gaseous exchange.
5. carbon dioxide that can be transported by the blood.
oxygen that can be transported by hemoglobin.
Carbon monoxide combines with ferromyoglobin and ferrohemoglobin, interfering with oxygen transport. Heme has a high affinity for carbon monoxide. Myoglobin and hemoglobin have less affinity than the isolated heme. However, they still bind. These proteins decrease the affinity of carbon monoxide to the heme-protein complex by sterically hindering the binding of carbon monoxide. More specifically, the myoglobin and hemoglobin change the geometry of the binding of carbon monoxide from that of the protein-oxygen binding and in doing so weakens the interaction of carbon monoxide with the heme, optimizing O2 binding. Nevertheless, carbon monoxide has a greater affinity for binding than oxygen, and this fact decreases the amount of oxygen that can be transported by hemoglobin. So again, the correct choice is #3, oxygen that can be transported by hemoglobin.
Cardiac muscle differs from the other types of muscle tissue in that cardiac muscle
A. Contains visible striations
B. Forms muscle fibers
C. Has both actin and myosin filaments
D. Has only one nucleus per cell
E. Is able to contract without neural stimulation
Is able to contract without neural stimulation

There are three types of muscle tissue in the human body: cardiac, skeletal and smooth muscle. All of these types of muscle tissue are specialized for contraction. Each of these types of muscle is designed to perform specific functions in the body. For example, cardiac muscle is found only in the heart and functions to circulate blood and maintain blood pressure. This type of muscle differs from skeletal and smooth muscle in that it is able to contract without neural stimulation. Within the cardiac muscle are specialized muscle cells called pacemaker cells, which establish a regular rate of contraction. Although the nervous system may alter the rate of pacemaker activity, it does not provide "control" over the cardiac muscle cells. Visible striations (choice A) are found in both skeletal and cardiac muscle. Both cardiac and skeletal muscle form muscle fibers (choice B). Actin and myosin filaments (choice C) are found in all three types of muscle. Although cardiac and smooth muscle contain one nucleus (choice D), skeletal muscle cells may contain hundreds of nuclei per cell.
Cell bodies of sympathetic preganglionic neurons are located in
1. dorsal root ganglia.
2. sympathetic chain ganglia.
3. the brain stem and the spinal cord.
4. thoracic and lumbar segments of the spinal cord.
5. none of the above.
thoracic and lumbar segments of the spinal cord.

Let's review the location of the cell bodies of pre- and post-ganglionic neurons of the sympathetic and parasympathetic nervous systems. In the sympathetic nervous system, pre-ganglionic cell bodies are located in thoracic spinal segments T1 through T12 and lumbar segments L1 through L3 or 4. Most of these neurons synapse in sympathetic ganglia in the sympathetic ganglion chain. Here, the post-ganglionic nerves originate and the post-ganglionic cell bodies are located in thc sympalhctic chain. These post-synaptic neurons travel a much longer distance than the pre-synaptic neurons, ending near the effector muscle or gland. In the parasympathetic system, the pre-ganglionic cell bodies are located in the mid-brain or medulla or the sacral regions of the spinal cord, sections S1 through S5. They synapse in ganglia far from the spinal cord, but near the effector musclc or gland. So choice 1 is incorrect. Dorsal root ganglia contain cell bodies of afferent neurons. Choice 2 is incorrect and is the location of cell bodies of the sympathetic post-synaptic neurons. Choice 3 is incorrect and is the location of pre-ganglionic parasympathetic cell bodies. So the correct answer to question is choice 4.
Cholesterol is primarily used by mammalian cells as
1. an energy storage material.
2. a precursor of ketone bodies.
3. a component of cell membranes.
4. a precursor of polyunsaturated fatty acids.
a component of cell membrane. Cholesterol, which can be derived from the diet or manufactured de novo in virtually all of the cells of the human body, plays a number of important roles. The first role of cholesterol is I that it is the major sterol in humans and a component of virtually all cell membranes and intracellular membranes. The second role of cholesterol is as the immediate precursor of the bile acid. The third physiological role of cholesterol is as the precursor of the various steroid hormones. So the correct answer to question is choice 3.
Chondroitin sulfate occurs primarily in
1. bile.
2. blood.
3. liver.
4. urine.
5. cartilage.
Cartilage

Chondroitin sulfate is in the chemical family of acid mucopolysaccharides which includes hyaluronic acid. As chondroitin 4-sulfate or chondroitin-A, and chondroitin 6-sulfate or chondroitin-C, chondroitin sulfates make up a large percentage of cartilage, bone and cornea structure. Of the other answer choices, it should be noted that a related acid mucopolysaccharide is heparin which prevents coagulation of blood and is found in arterial walls and in the lungs. So the correct answer to question is choice 5.
Clinical adult onset diabetes mellitus (Type II) results from
1. deficient secretion of insulin.
2. excess secretion of epinephrine.
3. failure of the kidneys to retain sugar.
4. secretion of oxytocin from the posterior pituitary.
5. loss of the insulin receptor function in the target tissues.
The correct answer is choice 5, loss of the insulin receptor function in the target tissues.

Non-insulin-dependent diabetes mellitus, which accounts for 80-90% of the diagnosed cases of diabetes, is also called adult onset diabetes to differentiate it from insulin-dependent juvenile diabetes. It usually occurs in middle aged, obese people. Insulin is present at near normal to elevated levels in this form of the disease. The defect in these patients lies in the loss of the insulin receptor function. Receptors are normally located on the plasma membranes of normally insulin- responsive cells, that is, hepatocytes, adipocytes and muscle cells. Insulin-dependent diabetes mellitus is also called juvenile onset diabetes or diabetes mellitus I, because it usually appears in childhood or the teens, but it is not limited to these patients. Insulin is either absent or nearly absent in this disease because of defective or absent beta cells in the pancreas. Choice 1 is incorrect. Deficient secretion of insulin would be associated with diabetes mellitus I. Choice 2 is incorrect as excess secretion of epinephrine does not lead to either form of diabetes. Choice 3 is incorrect. Sugar in the urine is due to excretion of the large quantity of plasma glucose present in diabetes. This is a normal function of the kidney. Choice 4 is incorrect. Oxytocin release is connected with uterine contraction and is not connected with either form of diabetes. So the correct answer to question is choice 5.
Corti's organ is responsible for perception of
1. light.
2. sound.
3. taste.
4. temperature.
5. proprioception.
Sound

. The organ of Corti is a spiral-shaped organ within the cochlea of the ear. It contains hair cells which are the auditory receptor cells. Sensation from the hair cells travels along the auditory or cochlear division of the vestibulocochlear acoustic nerve which terminates in the cochlear nuclei of the medulla. So the correct answer to question is choice 2.
Decreased arterial pressure upon standing is compensated by
A. decreased heart rate.
B. dilation of mesenteric vessels.
C. constriction of systemic arterioles.
D. dilation of venules.
Constriction of systemic arterioles

Immediately upon standing from a prone or supine position, a person's arterial blood pressure in the head and upper part of the body tends to fall. The baroreceptors located in the walls of large systemic arteries compensate by eliciting strong sympathetic discharge throughout the body resulting in constriction of systemic arterioles (Choice C) and thus raising blood pressure. Decreased heart rate (Choice A), dilation of mesenteric vessels (Choice B), and dilation of venules (Choice C) will all decrease blood pressure further which is undesirable in this case.
Cholesterol is primarily used by mammalian cells as
1. an energy storage material.
2. a precursor of ketone bodies.
3. a component of cell membranes.
4. a precursor of polyunsaturated fatty acids.
a component of cell membrane. Cholesterol, which can be derived from the diet or manufactured de novo in virtually all of the cells of the human body, plays a number of important roles. The first role of cholesterol is I that it is the major sterol in humans and a component of virtually all cell membranes and intracellular membranes. The second role of cholesterol is as the immediate precursor of the bile acid. The third physiological role of cholesterol is as the precursor of the various steroid hormones. So the correct answer to question is choice 3.
Chondroitin sulfate occurs primarily in
1. bile.
2. blood.
3. liver.
4. urine.
5. cartilage.
Cartilage

Chondroitin sulfate is in the chemical family of acid mucopolysaccharides which includes hyaluronic acid. As chondroitin 4-sulfate or chondroitin-A, and chondroitin 6-sulfate or chondroitin-C, chondroitin sulfates make up a large percentage of cartilage, bone and cornea structure. Of the other answer choices, it should be noted that a related acid mucopolysaccharide is heparin which prevents coagulation of blood and is found in arterial walls and in the lungs. So the correct answer to question is choice 5.
Clinical adult onset diabetes mellitus (Type II) results from
1. deficient secretion of insulin.
2. excess secretion of epinephrine.
3. failure of the kidneys to retain sugar.
4. secretion of oxytocin from the posterior pituitary.
5. loss of the insulin receptor function in the target tissues.
The correct answer is choice 5, loss of the insulin receptor function in the target tissues.

Non-insulin-dependent diabetes mellitus, which accounts for 80-90% of the diagnosed cases of diabetes, is also called adult onset diabetes to differentiate it from insulin-dependent juvenile diabetes. It usually occurs in middle aged, obese people. Insulin is present at near normal to elevated levels in this form of the disease. The defect in these patients lies in the loss of the insulin receptor function. Receptors are normally located on the plasma membranes of normally insulin- responsive cells, that is, hepatocytes, adipocytes and muscle cells. Insulin-dependent diabetes mellitus is also called juvenile onset diabetes or diabetes mellitus I, because it usually appears in childhood or the teens, but it is not limited to these patients. Insulin is either absent or nearly absent in this disease because of defective or absent beta cells in the pancreas. Choice 1 is incorrect. Deficient secretion of insulin would be associated with diabetes mellitus I. Choice 2 is incorrect as excess secretion of epinephrine does not lead to either form of diabetes. Choice 3 is incorrect. Sugar in the urine is due to excretion of the large quantity of plasma glucose present in diabetes. This is a normal function of the kidney. Choice 4 is incorrect. Oxytocin release is connected with uterine contraction and is not connected with either form of diabetes. So the correct answer to question is choice 5.
Corti's organ is responsible for perception of
1. light.
2. sound.
3. taste.
4. temperature.
5. proprioception.
Sound

. The organ of Corti is a spiral-shaped organ within the cochlea of the ear. It contains hair cells which are the auditory receptor cells. Sensation from the hair cells travels along the auditory or cochlear division of the vestibulocochlear acoustic nerve which terminates in the cochlear nuclei of the medulla. So the correct answer to question is choice 2.
Decreased arterial pressure upon standing is compensated by
A. decreased heart rate.
B. dilation of mesenteric vessels.
C. constriction of systemic arterioles.
D. dilation of venules.
Constriction of systemic arterioles

Immediately upon standing from a prone or supine position, a person's arterial blood pressure in the head and upper part of the body tends to fall. The baroreceptors located in the walls of large systemic arteries compensate by eliciting strong sympathetic discharge throughout the body resulting in constriction of systemic arterioles (Choice C) and thus raising blood pressure. Decreased heart rate (Choice A), dilation of mesenteric vessels (Choice B), and dilation of venules (Choice C) will all decrease blood pressure further which is undesirable in this case.
Decreased response of sense organs when exposed to a constant stimulus is called
A. occlusion.
B. summation.
C. adaptation.
D. facilitation.
E. sensory deprivation.
Adaptation

A common characteristic that applies to all sensory organs is that their response to a constani stimulus decreases after a period of time. This special trait is known as adaptation (Choice C). Adaptation occurs in all sensory receptors but to different extents and by different mechanisms.
Choice A occlusion as a general term refers to the act of closing or the state of being closed. It does not describe sensory organ response to stimuli.
Choice B summation often refers to the additive effect of frequent repetitive stimuli applied to nerve terminals to achieve excitation. Again, it does not involve sensory organs or their response.
Choice D facilitation is the enhancement or reinforcement of a nervous activity by the arrival of signals from other neurons.
Choice E sensory deprivation occurs when some or all usual external stimuli are absent. It will often result in psychological stress and abnormal behavior. It also does not apply to sensory receptors response to stimuli.
Dental plaque arises primarily as a result of enzymatic reactions using which of the following?
1. Sucrose and lipid
2. Sucrose and saliva
3. Glucose and saliva
4. Glucose and protein
Sucrose and Saliva

One of the few dental questions on this exam, you should recognize that the major carbohydrate substrate and energy source of plaque is sucrose, not glucose. In addition, the salivary proteins are often involved in aiding the plaque to adhere to tooth structure. In general, lipids are not highly involved in plaque formation, so choice 1 is incorrect. Choices 3 and 4 are incorrect because they include glucose and not sucrose. So the correct answer to question is choice 2.
Destruction of the cerebellum causes loss of
1. memory.
2. vision.
3. hearing.
4. muscular coordination.
5. both (2) and (3) above.
Muscular coordination

The cerebellum acts to coordinate voluntary muscular activity. Choice 1 is incorrect. Memory function is located within the cerebral cortex, especially in the temporal lobes. Choice 2 is incorrect. Visual function within the brain is located within the visual cortex of the cerebrum in the occipital lobes. Choice 3 is incorrect. Hearing function is located within the cerebral cortex in the area of the auditory cortex of the temporal lobes. So the correct answer to question is choice 4.
Development of ovarian follicles to the point of ovulation is stimulated primarily by
1. estrone.
2. progesterone.
3. luteinizing hormone.
4. follicle-stimulating hormone.
5. chorionic gonadotropic hormone.
FSH

We can get this just from the name of the hormone. Ovarian follicles are stimulated, not surprisingly, by follicle-stimulating hormone or FSH from the anterior pituitary. Following ovulation, the remaining corpus luteum will be stimulated by LH or luteinizing hormone, choice 3. Choice 1, estrone, is a female sex hormone produced by the ovary, as is progesterone, choice 2. They are involved primarily in preparing and maintaining the uterine lining for implantation. Chorionic gonadotrophic hormone is produced by the placenta in the pregnant woman. It helps maintain the uterine lining during pregnancy. However, only FSH or follicle-stimulating hormone, choice 3, will stimulate the ovarian follicles.
During an automobile accident, a child's posterior pituitary is severely damaged. Which of the following would most likely be seen?
A. Addison disease
B. Decreased skin pigmentation
C. Decreased thyroid function
D. Diabetes insipidus
E. Dwarfism
Diabetes insipidus

First, you need to know which hormones are produced in the posterior pituitary; then know the function of each hormone; and finally know which pathologic condition will follow when this "particular" hormone is absent. The posterior pituitary produces antidiuretic hormone (ADH, or vasopressin) and oxytocin. ADH is responsible for maintaining plasma osmolarity and preventing excessive water loss. When ADH is absent, diabetes insipidus can result. This condition is caused by a decrease in the renal reabsorption of water, an increase in serum osmolarity, and the generation of a dilute (hypotonic) urine. All the other conditions can result from a decrease in the secretions of the anterior pituitary. Addison disease (choice A) is caused by decreased production of corticosteroids, such as cortisol. The production is decreased secondary to a decrease in the secretion of ACTH. Decreased skin pigmentation (choice B) can occur when melanocyte-stimulating hormone is not secreted. When thyroid-stimulating hormone (TSH) is not secreted, you can expect to see a decreased thyroid function (choice C). Dwarfism (choice E) is a condition that can occur in young children when growth hormone production is decreased.
During extended periods of fasting, all of the following are able to utilize ketones and/or fatty acids as a fuel source EXCEPT
A. the brain
B. cardiac muscle
C. the liver
D. skeletal muscle
E. red blood cells
RBCs
Patterns of fuel utilization depend on the metabolic state (well-fed versus fasting) and on the individual tissue. Most cells in the body are able to utilize different fuels during the fasting state, except red blood cells. Red blood cells are only able to utilize glucose as a fuel source. Although glucose is the primary fuel for the brain (choice A), during periods of fasting, this organ can utilize ketone as an energy source. The primary source of fuel for cardiac muscle (choice B) is glucose and fatty acids. However, cardiac muscle can utilize ketones during periods of fasting. The liver (choice C) prefers to utilize glucose and amino acids; however, it can utilize glycogen, lactate, glycerol, fatty acids, and amino acids. Skeletal muscle (choice D) can utilize glucose, as well as fatty acids and ketones.
During periods of fasting or starvation, an increased production of ketone bodies leads to
A. Decreased blood pH
B. Diabetes
C. Increased blood pH
D. Increased protein breakdown
E. Skeletal muscle breakdown
Decreased blood pH

A ketone body is also referred to as a keto acid since it dissociates in solution releasing a hydrogen ion. As a result, the appearance of ketone bodies in circulation will decrease the blood pH, making it more acidic. This acidification process is referred to as ketoacidosis. Ketone bodies are produced as a result of lipid and amino acid breakdown during periods of fasting or starvation When excessive amounts of ketone bodies are formed, the pathological state known as ketoacidosis can occur. Ketoacidosis occurs most commonly in patients with diabetes (choice B). Note: Diabetes can lead to ketoacidosis, not vise versa. Ketoacidosis would not increase protein breakdown (choice D) or skeletal muscle production.
During periods of moderate exercise, one would expect to see all of the following EXCEPT
A. decreased carbon dioxide production
B. increased oxygen consumption
C. increased pulmonary blood flow
D. increased ventilation rate
E. no change in arterial pH
Decreased CO2 production

During periods of exercise, a number of physiologic changes occur that account for the increased oxygen demand in the body. During periods of moderate exercise, one would expect to find an increased, not decreased, production of carbon dioxide. There is an increased oxygen consumption (choice B), increased pulmonary blood flow (choice C), and increased ventilation rate (choice D). In addition, there is no change in arterial pH (choice E); however, one may see a decrease in arterial pH during periods of strenuous activity.
During red blood cell recycling each heme unit is stripped of its iron and immediately converted to:
A. bilirubin
B. biliverdin
C. ferritin
D. transferrin
E. urobilin
Biliverdin

An erythrocyte is exposed to severe mechanical stresses over the course of its life. A single round-trip of the circulatory system for an erythrocyte often takes less than 30 seconds. During this time period it is bounced off vessel walls and squeezed through tiny capillaries. With all this wear and tear and no repair mechanisms, the red blood cell has a relatively short life span. As the "old" red blood cells are broken down and recycled, each component of the hemoglobin has a different fate. For example, during this red blood cel recycling each heme unit is stripped of its iron and immediately converted to biliverdin. Biliverdin is a porphyrin derivative with a green color. The biliverdin is then converted into bilirubin (choice A) which is found in the general circulation. Liver cells absorb the bilirubin and use it to synthesize a complex molecule, conjugated bilirubin, which is excreted in the bile. Transferrin (choice D) is a plasma protein to which excessive iron is bound. Red blood cells developing in the bone marrow absorb amino acids and transferrins from circulation and use them to synthesize new hemoglobin molecules. Excessive transferrins are removed by the liver and bone marrow and the iron is then stored in two special protein-iron complexes, ferritin (choice C) and hemosiderin. Inside the large intestine, bacteria will convert the conjugated urobilin to urobilogen. When exposed to oxygen, the urobilinogen is converted to urobilin (choice E) which is the substance that gives urine its yellow color.
During which of the following phases of the cardiac cycle are all cardiac valves open?
1. Rapid filling
2. Early ejection
3. lsovolumic relaxation
4. lsovolumic contraction
5. None of these
none of the above. During atrial systole, a small amount of blood is pumped into the ventricles. The semilunar valves are closed and the AV valves are then opened to provide a funnel for the transfer of blood from atrium to ventricle. This continues until the ventricular pressure exceeds the atrial pressure and causes the AV valves to close during ventricular systole. Ventricular systole causes the semilunar valves to open when ventricular pressure exceeds pulmonary and aortic diastolic pressure, thus beginning the period of isotonic contraction during which blood is ejected into the outflowing trunk. When pulmonary and I aortic pressures exceed end systolic ventricular pressure, it causes the semilunar valves to close. So in summary there is no point at which all valves are open at the same time. Choice 1 is incorrect. During rapid filling. AV valves are open, semilunar valves are closed. Choice 2 is incorrect. During early ejection, semilunar valves are open but AV valves are closed. Choice 3, isovolumetric relaxation, semilunar valves are closed. Choice 4, isovolumetric contraction, AV valves are closed. So the correct answer to question again, is choice 5.
Elevated levels of calcium ions in the blood stimulates the secretion of which of the following?
A. Calcitonin
B. Growth hormone
C. Parathyroid hormone
D. Testosterone
E. Thyroid hormone
Calcitonin

...... is the hormone primarily responsible for the regulation of calcium ion concentrations in the blood. When blood levels of calcium ions become elevated, calcitonin is secreted to reduce calcium ion levels to normal limits. Growth hormone (choice B) stimulates cell growth and replication by accelerating the rate of protein synthesis. Parathyroid hormone (choice C) is responsible for increasing calcium ion concentrations in the body fluids; therefore, this hormone would not be secreted in an individual with hypercalcemia. Testosterone (choice D) promotes the production of functional sperm, maintains the secretory glands of the male reproductive tract and stimulates growth. The effects of thyroid hormones (choice E) are as follows: Elevation of oxygen consumption and the rate of energy consumption; increased heart rate; stimulation of activity of other endocrine glands; maintenance of sensitivity of respiratory centers as well as various other functions.
Emotional feelings are MOST closely related to which area of the brain?
A. Thalamus
B. Brain stem
C. Cerebellum
D. Hypothalamus
E. Limbic system
Limbic System


Emotional drives, motivation, and behavior are most closely associated with the limbic system of the brain (Choice E). It is located in the midbasal region of the brain.
Choice A thalamus, located in the lower region of the brain, contributes to the subconscious activities of the body such as control of blood pressure and respiration.
Choice B the brain stem consists of the medulla, pons, and mesencephalon. It is concerned with the motor and sensory functions for the face and head regions.
Choice C the cerebellum lies above the pons and medulla and is chiefly responsible for motor coordination and body equilibrium.
Choice D the hypothalamus is important in the control of autonomic body functions to maintain homeostasis.
Energy for ATP synthesis is derived from the electron transport system by which of the following processes?
1. Transamination
2. Aldolization
3. Reductive synthesis
4. Oxidative deamination
5. Oxidative phosphorylation
oxidative phosphorylation.

Oxidative phosphorylation is necessary for all cells that derive energy via aerobic oxidation. Mitochondria, the powerhouses of the cell, are the locations of the oxidative phosphorylation and electron transport enzymes. The mitochondria have two membranes, the inner membrane and the outer membrane. The inner membrane contains cytochromes b, c, c1, a and a3, and the FATP-ase associated with the mechanism of oxidative phosphorylation. Proof has been established that electron transport from NADH to oxygen is the main source of energy used for the completed phosphorylation of ADP, so the correct answer is #5, oxidative phosphorylation.
Establishing a tracheostomy results in
1. increased airway resistance.
2. increased anatomic dead space.
3. increased physiologic dead space.
4. decreased respiratory work.
5. decreased effective alveolar ventilation.
decreased respiratory work.

Anatomic dead space is that part of the conducting airway in which almost no gas exchange occurs. This includes the trachea, the main stem bronchi, and their subdivisions. This anatomic dead space is decreased by tracheostomy because of removal or bypass of some of the conducting airway. In doing so, the work of breathing which is defined as P V, pressure x the change in volume, is decreased because volume is decreased. Choice 1 is incorrect. Most of the turbulence to air flow is in the trachea and therefore bypassing the trachea results in decreased airway resistance. Choice 2 is incorrect. The trachea is part of the anatomic dead space. Tracheostomy actually decreases and not increases the anatomic dead space. Choice 3 is incorrect. Physiologic dead space = the anatomic dead space plus the volume of air that reaches the alveoli but does not come into contact with pulmonary capillary blood and therefore does not participate in gas exchange. Choice 5 is incorrect. Effective alveolar ventilation is ventilation that occurs at the gas exchange surfaces of the alveoli. A tracheostomy does not affect the effective alveolar ventilation. So the correct answer to question is choice 4.
Excluding the effect of drugs, apnea occurring after hyperventilation of an anesthetized patient results from
1. decreased demand for oxygen.
2. increased cerebral blood flow.
3. decreased carbon dioxide tension.
4. increased hydrogen ion concentration
5. decreased oxygenation of carotid bodies.
Decreased CO2 tension

The term "apnea" refers to respiratory arrest, a time period during which breathing ceases. Respiratory activity is stimulated by the C02 level of the blood as sensed by chemoreceptors in the medulla. It is also triggered by 02 levels as sensed by the aortic and carotid bodies. When a patient is hyperventilated, sufficient oxygen can be taken in, but excess C02 may be released from the blood. Low C02 level will then fail to stimulate the medullary chemoreceptors. Choice 1 is incorrect. Hyperventilation will not cause decreased oxygen demand. Choice 2 is also incorrect as hyperventilation does not increase cerebral blood flow. Choice 4 is incorrect. Hyperventilation will cause loss of C02 which causes acid loss or increased pH or respiratory alkalosis. Choice 5 is incorrect as well. Decreased oxygenation would not occur during hyperventilation and decreased oxygenation of thc carotid bodies should increase respiration, not cause apnea. So the correct answer to question is choice 3.
Following an antineoplastic treatment regimen, a patient's uric acid level increases dramatically. Therefore, this individual would derive the most benefit from an agent that inhibits the action of which of the following enzymes?
A. Alpha-glucosidase
B. Angiotensin-converting enzyme
C. Beta-lactamase
D. Cyclooxygenase
E. Xanthine oxidase
Xanthine oxidase

After selected antineoplastic therapies, patients can have a high level of urate secondary to the breakdown of nucleic acids. Therefore, they are often administered allopurinol to decrease plasma urate levels. Allopurinol prevents the formation of uric-acid by inhibiting the enzymkxanthine oxidase. Decreasing uric acid levels will help prevent the formation of kidney stones, as well as block the appearance of other deleterious effects of hyperuricemia. Acarbose is an agent used in the treatment of type 2 diabetes. This agent inhibits the activity of alpha-glucosidase (choice A) in the intestinal tract, thereby helping to prevent postprandial surges of hyperglycemia. Inhibition of angiotensin-converting enzymes (choice B) will result in a lowering of blood pressure and aid in the prevention of the ventricular remodelingn that occurs secondary to congestive heart failure. Beta-lactamase (choice C) inhibitors are combined with peniciliin antibiotics to help improve their activity against bacteria that produce the enzyme beta-lactamase. Nonsteroidal antiinflammatory drugs (NSAIDs) inhibit the activity of the enzyme cyclooxygenase (choice D), thereby decreasing the production of prostaglandins.
Following exercise, ventilation is in excess of the metabolic requirement. During this period, the level of lactic acid in blood
1. increases.
2. decreases.
3. remains unchanged.
decreases. Severe exercise will increase the demands on the muscle to a point where it must rely on glycolysis for the energy. Lactic acid, as a result, will be produced. This produces an acidotic state. Acidosis will increase ventilation, along with increasing the number of open capillaries, to increase surface area. An increase in the cardiac output causes an increase in the capillary blood flow. All of these factors tend to decrease the concentration of the lactic acid in the blood and return it a to its normal pre-exercise conditions. So again, the correct choice is 2.
Following the birth of an infant, which of the following hormones is responsible for increasing milk production in the mother?
A. Estrogen
B. Leutinizing hormone
C. Oxytocin
D. Progesterone
E. Prolactin
Prolactin

Prolactin is the hormone responsible for increasing the production of milk in the mammary glands of women after the birth of a child. Note: The role of prolactin in males is poorly understood although it is believed to have a role in the regulation of androgen production. Milk is continuously secreted into the alveoli of the breasts; however, it does not flow easily from the alveoli into the ductal system. The milk is inhibited from entering the ductal system to prn,v ent the continuous leaking of milk from the breasts when the mother is not feeding. Therefore, the milk must be "let down" from alveoli to the ductal system before the baby can obtain it. Oxytocin (choice C) stimulates the myoepithelial cells in the mammary glands to express milk from the breast when a baby suckles. Aside from their other actions, both estrogen (choice A) and progesterone (choice D) aid in the development of breast tissue; however, these hormones inhibit the production of milk in the mammary glands. Luteinizing Hormone (choice B) plays an important role in causing ovulation and secretion of female sex hormones from the ovaries.
For which of the following are the loops of Henle responsible?
1. The acidification of urine
2. The reabsorption of amino acids
3. The reabsorption of most of the glomerular filtrate
4. The formation of most of the ammonium ion excreted in the urine
5. The establishment of an osmotic gradient within the medulla of the kidney
The establishment of an osmotic gradient within the medulla of the kidney

The major function of the loop of Henle is to reabsorb sodium ions into the cells surrounding the tubule. This absorption sets up a high concentration of solute which is then used to help in the passive reabsorption of water through the collecting tubules. Choice 1 is incorrect. Acidification of the urine occurs by hydrogen ion secretion. This occurs throughout all tubules of the kidney. Choices 2 and 3 are both incorrect. Most reabsorption, for example, of glucose and amino acids occurs in the proximal convoluted tubule. For example, more than 99% of all filtered glucose is reabsorbed in the proximal convoluted tubule. Choice 4 is incorrect. Very little ammonia is excreted by the kidney as most is changed by the liver to urea. Any ammonia in urine is the result of deamination. As a review point, remember that the reabsorption of water is through the collecting tubules and that reabsorpuon is under the control of ADH or antidiuretic hormone from the posterior pituitary. Its action is to make the collecting tubule more permeable to water therefore more reabsorption occurs and we get a more concentrated urine. So the correct answer to question is choice 5.
Glomerular filtration rate may be determined by clearance of
1. urea.
2. water.
3. inulin.
4. glucose.
5. para-aminohippuric acid (PAH).
inulin
Glomerular filtration rate or GFR is a measure of the volume per time of filtrate that leaves the glomerulus and enters the Bowman's capsule. It is measured by a material that is freely filtered into the Bowman's capsule and not reabsorbed nor secreted by any tubules. Inulin, a fructose polymer, is usually used for this purpose. Choices 2 and 4 are incorrect because they are reabsorbed from the blood in the kidney tubules. Choice 5, PAH or para-aminohippurate acid, is often used to measure renal plasma flow and is a substance freely filtered by the glomerulus but actively secreted by tubular cells. Choice 1, urea, although readily filtered into the Bowman's capsule, is more than 50% reabsorbed and readily flows into and out of certain areas of the kidney. So our correct answer to question is choice 3.
Glycolysis begins when glucose reacts with which of the following enzymes?
A. Aldolase
B. Hexokinase
C. Phosphofructokinase
D. Phosphoglucose isomerase
E. Pyruvate kinase
Hexokinase

Glycolysis is defined as the sequence of reactions that converts glucose into pyruvate with the concomitant production of ATP. The process of glycolysis begins when glucose reacts with hexokinase to form glucose-6-phosphate. When this compound interacts with the enzyme phosphoglucose isomerase (choice D), the compound fructose-6-phosphate is formed. Following the formation of this compound, it is interacted with the enzyme phosphofructokinase (choice C) to form the compound fructose 1,6-biphosphate. When interacted with aldolase (choice A) glyceraldehyde 3-phosphate is formed. After a number of enzymatic reactions, the compound phosphoenolpyruvate is formed. When acted upon by pyruvate kinase (choice E), pyruvate is formed and the glycolytic pathway is completed.
Glycosaminoglycans function as important structural components of
1. glycogen.
2. nucleic acids.
3. hyaluronidase.
4. connective tissue.
connective tissue. Glycoprotein contains carbohydrate and protein, usually more protein than carbohydrate. There is another class of glycoprotein which contains 95% or more carbohydrate. To distinguish these compounds from other glycoproteins, they are referred to as proteoglycans and their carbohydrate chains as glycosaminoglycans or GAG. There are six classes of GAG molecules now recognized. They are made up largely of disaccharide repeating units. Both their electrical charge and 1 molecular structure aid in their biological role. Choice 1, glycogen, is incorrect. Glycogen consists of alpha 1,4 or alpha 1.6-linked glucose molecules. Choice 2 is incorrect. Nucleic acids form chromatin, the material which makes up chromosomes. Choice 3, hyaluronidase, is incorrect. Hyaluronidase functions as a lubricant and support element in connective tissue. So the correct answer to question is choice 4.
Human parotid saliva is believed to be hyposmolar because
1. water is reabsorbed by excretory ducts of the gland.
2. acinar cells transport water more readily than sodium ions.
3. striated duct cells produce excess water and retain potassium.
4. reabsorption of water by striated duct cells is less than reabsorption of sodium.
resorption of water by striated duct cells is less than resorption of sodium. To answer this question you must have an understanding of the epithelial components of the salivary gland and the overall morphology. The epithelial components of a salivary gland, histologically, include the acini, intercalated and striated ducts and the excretory duct cells. The acinus resembles a spherical arrangement of protein-producing cells with well-developed rough endoplasmic reticulum. The product is secreted via secretory vesicles and travels through the ductile lumen, passing by the cuboidal intercalated duct cells and striated cells. The striated cells are responsible for the alteration of the fluid product. Sodium ions, chloride ions and water are removed and transported back into the blood, and potassium ions are added to the product from the blood. Thus, we have an active process and the formation of the hypo-osmolar saliva is dependent on ATP and the functioning sodium-potassium pump. In formation of saliva the resorption of water via the striated duct cells is less than the resorption of sodium. From the striated duct the salivary product passes into the excretory duct, which carries saliva into the oral cavity. Again, the correct choice is #4, resorption of water by striated duct cells is less than the resorption of sodium.
Hypoactivity of the posterior pituitary gland leads to
A. dwarfism.
B. cretinism.
C. acromegaly.
D. diabetes insipidus.
Diabetes insipidus

The posterior pituitary gland secretes two important hormones, oxytocin and antidiuretic hormone. Antidiuretic hormones control and regulate the concentration of water in the body fluids. A hypoactivity of the posterior pituitary gland resulting in a deficiency of antidiuretic hormone will lead to diabetes insipidus (Choice D). In this condition. the kidneys excrete larger amounts of dilute urine causing dehydration and increased concentration of sodium chloride in the extracellular fluid.
Dwarfism (Choice A) is caused by deficiency of anterior pituitary secretion of GH (growth hormone) and not posterior pituitary secretion.
Cretinism (Choice B) is caused by extreme hypothyroidism during fetal life, infancy, and childhood.
Acromegaly (Choice C) is caused by an excessively active anterior pituitary gland, and excess GH in adulthood.
Hypothyroidism in an adult results in
1. myxedema.
2. thyrotoxicosis.
3. osteitis fibrosa cystica.
4. increased basal metabolic rate.
myxedema

Hypothyroidism refers to an underactive thyroid which produces insufficient thyroxin. Under this condition, basal metabolism rate is low, hair is sparse, skin is yellowish and dry, and cold is not well tolerated. In adults, hypothyroidism is often termed myxedema. It is the opposite of choice 4, hyperthyroidism, which causes increased metabolic rate. Choice 2. thyrotoxicosis, is another term for hyperthyroidism. Choice 3, osteitis fibrosa cystica, refers to a bone disease caused by hyperparathyroidism. In this disease, multiple bone cysts occur. So the correct answer to question is choice 1.
If a patient becomes excited and nervous before a tooth extraction and exhales large amounts of carbon dioxide, the most likely effect on the body would be
A. Lactic acidosis
B. Metabolic acidosis
C. Metabolic alkalosis
D. Respiratory acidosis
E. Respiratory alkalosis
Respiratory alkalosis

An important buffer system in the human body involves carbon dioxide and bicarbonate ions as seen below.
C02 + H20 <-> H2CO3 <-> H+ + HC03
Alterations in the carbon dioxide and/or bicarbonate levels will result in alterations in the blood pH. Respiratory alkalosis develops when increased respiratory activity, as seen with hyperventilation, lowers plasma PCO2 levels below normal levels. The resultant effect on blood pH will be a net increase. Conversely, respiratory acidosis (choice D) develops when decreased respiratory activity, as seen with hypoventilation, increases plasma PC02 levels above normal levels. The resultant effect on blood pH will be a net decrease. Lactic acidosis (choice A) may develop following severe exercise or prolonged tissue hypoxia. The most common causes of metabolic acidosis are lactic acidosis and ketoacidosis. Metabolic acidosis (choice B) occurs when the hydrogen ions liberated by these acids overwhelm the bicarbonate buffer system leading to a net decrease in blood pH. Metabolic alkalosis (choice C) occurs when bicarbonate concentrations become elevated, leading to a net increase of blood pH.
If a patient becomes hypotensive after the administration of a sedative before a dental procedure, which of the following reflexes should help to correct the patients low blood pressure?
A. Acquired reflex
B. Baroreceptor reflex
C. Chemoreceptor reflex
D. Hering-Breuer reflex
E. Protective reflex
Baroreceptor Reflex

A reflex is defined as a rapid automatic response to a stimulus. The baroreceptor reflexes affect systemic blood pressure and stimulate the respiratory rate when blood pressure declines. For example, aortic baroreceptors monitor blood pressure within the ascending aorta. The aortic reflex, which is a baroreceptor reflex, adjusts blood pressure in response to changes at this particular location. The goal of this reflex is to maintain adequate blood flow throughout the circulatory system. An acquired reflex (choice A) is a reflex that is considered to be very complex and is a learned motor pattern. An example of an acquired reflex would be when an experienced driver steps on the brakes of the car when an animal runs in front of the vehicle. The chemoreceptor reflexes (choice C) are stimulated by changes in pH, PC02 and P02. The Hering-Breuer reflex (choice D) is composed of two different reflexes; the inflation reflex and the deflation reflex. The inflation reflex prevents over expansion of the lungs during forceful breathing and the deflation reflex inhibits the expiratory center and stimulates the inspiratory center when the lungs are in danger of collapsing. Coughing and sneezing are protective reflexes (choice E) that operate on the exposure to chemical irritants, toxic vapors or mechanical stimulation of the respiratory system. Coughing is triggered by irritation of the larynx and sneezing is triggered by irritation of the wall of the nasal cavity.
If a person has a stroke that results in a depression of the activity in the pneumotaxic center of the pons, one would expect to see
A. Decreased apneustic center activity
B. Decreased tidal volume
C. Increased pulmonary ventilation
D. Increased respiratory rate
E. Longer inspiratory cycle
Longer inspiratory cycle

The apneustic center of the lower pons and the pneumotaxic center of the upper pons adjust the respiratory rate and depth of respiration in response to stimuli. In summary, the apneustic center promotes inhalation and the pneumotaxic center, which inhibits apneustic center activity, promotes exhalation. Therefore, if the activity of the pneumotaxic center is inhibited, there would be a much longer inspiratory cycle since the activity of the apneustic center cannot be inhibited. In other words, apneustic center activity would increase (choice A). Since only inspiration is promoted, the tidal volume in this person would be expected to increase (choice B). Since the respiratory rate (choice D) will decrease due to a lack of cyclic inhalations and exhalations, pulmonary ventilation would also be expected to decrease (choice C).
If a person suffered a stoke that left them partially paralyzed on one side of the body, one would expect to see an increased number of which of the following cell types in the area of the brain affected by the stroke?
A. Astrocyte
B. Ependymal cells
C. Microglia
D. Oligodendrocyte
E. Satellite cells
Microglia

In a stroke, the blood flow to a specific area of the brain is interrupted by some kind of blockage or vascular rupture. If the circulation to a given region is interrupted for more than a few seconds the neurons will be permanently damaged. The degree of functional impairment that an individual will suffer after a stroke is determined by the duration of circulatory interruption as well as the size of the region deprived of blood. After a stroke, the nerve cells will become damaged. Therefore, one would expect to see an increased number of microglia cells in the affected area. Microglia cells are responsible for removing cellular debris, wastes and pathogens by the process of phagocytosis. Astrocytes (choice A) are glial cells that maintain the integrity of the blood brain barrier, provide structural support and regulate ion, nutrient and dissolved gas concentrations. Ependymal cells (choice B) assist in the production, circulation and monitoring of cerebrospinal fluid. Oligodendrocytes (choice D) myelinate CNS axons and provide the structural framework for the axons in the CNS. Satellite cells (choice E) are the glial cells found in the peripheral nervous system and surround the neuron cell bodies in the ganglia.
If a person, who has lived at sea level all their life, takes a vacation to a mountain resort, which of the following physiological changes will most likely be seen in this individual after a few days?
A. Decreased blood pressure
B. Elevated hematocrit
C. Decreased PO2 in the alveoli
D. Decreased ventilation rate
E. Increased hemoglobin saturation
Elevated hematocrit

Both atmospheric pressure and the partial pressures of the component gasses, including oxygen, decline with increasing altitude. Individuals living at sea level normally function with an alveolar pressure around 80-90 mmHg. However, at higher elevations, the alveolar PO2 can fall to as low as 60 mmHg. Although the alveolar PO2 decreases, the body will make important physiological adjustments to compensate, such as increasing heart rate (leading to an increased blood pressure (choice A), elevated hematocrit (choice B) to account for the decreased hemoglobin saturation (choice E) as well as increasing the ventilation rate (choice D) to increase the flow of oxygen through the lungs.

(Hematocrit is the percentage of the concentration of RBCs in blood. Normally 45% for men and 40% for women.
If an agonist produces flexion, the primary action of the antagonist would be
A. Abduction
B. Adduction
C. Extension
D. Opposition
E. Protraction
Extension

Flexion is described as a movement in the anterior/posterior plane that decreases the angle between the articulating extremities. For example, flexion occurs at the biceps. If an agonist produces flexion, the primary action of the antagonist would be extension, which reverses the action of flexion. Extension occurs in the same plane but increases the angle between the articulating extremities. The triceps cause extension. Abduction (choice A) is the movement away from the longitudinal axis of the body. For example, swinging the right arm to the side. Conversely, adduction (choice B) is movement toward the longitudinal axis of the body. Opposition (choice D) is the movement of the thumb that allows it to grasp and hold various objects. Protraction (choice E) is described as the movement of a given part of the body anteriorly in a horizontal plane: the grasping of ones upper lip with their lower teeth is a protracting movement.
If an individual becomes hyponatremic, which of the following physiological responses would one expect to see?
A. Increased aldosterone levels
B. Increased antidiuretic hormone release
C. Increased thirst
D. Increased water retention
E. Stimulation of the osmoreceptors
Increased Aldosterone levels

Hyponatremia is a condition where sodium ion levels in the blood are lower than normal. There are many different mechanisms through which sodium levels can be regulated. One of the primary ways in which hyponatremia occurs is through the dilutional effect of increased blood volume secondary to increased fluid intake. When sodium levels are low, one would expect to see an increased blood level of aldosterone. Aldosterone causes sodium to be reabsorbed from the filtrate in exchange for potassium. Therefore, aldosterone increases sodium ion concentrations. All of the other answer choices (situations) would be expected to be seen in an individual with hypernatremia. In hypernatremia, osmoreceptors are stimulated (choice E) which then causes antidiuretic hormone (ADH) to be released (choice B) and an increased thirst for fluid (choice C). When ADH is released there is an increased water retention (choice D), leading to an increased blood volume and a subsequent lowering of sodium ion levels.
If an individual contracted a disease that destroyed a portion of the cells in the anterior gray horn of the lumbar region of the spinal cord, one would expect to experience which of the following?
A. Decreased ability to breathe
B. Decreased sensation in the hands
C. Decreased sensation in the legs
D. Problems with raising the arms
E. Problems with walking
Problems with walking

The cell bodies of the neurons in the gray matter of the spinal cord contain two types of nuclei: sensory nuclei and motor nuclei. The posterior portion of the gray horn contains somatic and visceral sensory nuclei. The anterior gray horn contains somatic motor nuclei. Therefore, if an individual damages the cells in the anterior gray horn of the spinal cord, one would be expected to have a decreased ability to perform associated motor functions. Since the lumbar plexus of the spinal cord is responsible for coordinating ones ability to walk, the individual in this question would most likely have problems walking. However, since the white matter was not damaged, the sensation in the legs would not be affected (choice C). One's ability to breathe would be affected by the neurons of the cervical plexus (choice A). The sensations of the hands (choice B) and ability to move the arm (choice D) are regulated by the nerves in the brachial plexus.
If an individual contracted travelers diarrhea and was unable to ingest carbohydrates, one would expect to see which of the following?
A. Decreased gluconeogenesis in the liver
B. Decreased levels of urea in the blood
C. Increased blood pH
D. Increased protein metabolism
E. Ketosis
Ketosis

If carbohydrates cannot be ingested for an extended period of time, the body will begin to convert both fat and protein stores into carbohydrates. Ketosis is a condition that occurs when the body increases the production of ketone bodies from the catabolism of the glucogenic and ketogenic amino acids. As mentioned before, when carbohydrate reserves have been exhausted, the body will catabolize amino acids (choice D) during the process of gluconeogenesis (choice A). Once the amino acids have been broken down, ammonia will be generated through this catabolic process. The ammonia created will be converted to urea (choice B). With the increased production of keto acids, one would expect to see a decreased blood pH (choice C).
If an individual damaged their fifth cranial nerve, they would be expected to have an inability to perform which of the following cranial reflexes?
A. Auditory reflex
B. Corneal reflex
C. Direct light reflex
D. Tympanic reflex
E. Vestibuloocular reflex
Corneal Reflex

The cranial reflex arc involves both sensory and motor fibers of the cranial nerves. The afferent division of cranial nerve V (trigeminal nerve) is responsible for providing the corneal reflex. The cranial reflex is a somatic reflex that results in the blinking of the eye when there is contact with the corneal surface. The vestibuloocular nerve (cranial nerve VIII) coordinates three somatic reflexes: Auditory reflex, tympanic reflex and vestibuloocular reflex. The auditory reflex (choice A) is stimulated by a loud noise; the eyes and/or head will move in the direction of the sound. The tympanic reflex (choice D) is also stimulated by a loud noise. This reflex causes a reduced movement of the auditory ossicles. The vestibuloocular reflex (choice E) is stimulated by the rotation of the head. With this reflex, there is an opposite movement of the eyes in the direction of the moving head to stabilize the filed of vision. The direct light reflex (choice C) results in the constriction of the ipsilateral pupil when light is striking the photoreceptors in the eye.
If an individual decides to go on a hunger strike and ingest only water on a daily basis, one would expect to see which of the following after approximately one week?
A. Decreased urine output
B. Increased androgen levels
C. Increased glucocorticoid levels
D. Increased insulin levels
E. Increased lipogenesis
Increased Glucocorticoid levels

When individuals do not ingest nutrients for a period of time, the body goes into a metabolic state where it must rely on internal energy reserves to continue meeting its own energy demands. Approximately 12 hours each day is spent in this metabolic state. The primary focus of this state is to mobilize energy reserves and maintain normal blood glucose levels. With respect to the individual on the "hunger strike" in this question, carbohydrate reserves will become depleted several hours after the last meal. Once these reserves are depleted, the body will then focus on converting lipids and proteins into carbohydrates in an effort to maintain "normal" blood glucose levels. Increased glucocorticoid levels will have the following metabolic effects: stimulation of amino acid and lipid breakdown by the liver; protein breakdown in skeletal muscles; and gluconeogenesis and glycogenolysis in the liver. In other words, if an individual decides to go on a hunger strike and ingest only water on a daily basis, one would expect to see increased glucocorticoid levels several hours after the ingestion of the last meal. As long as the individual on the hunger strike maintains normal water intake, urine output should remain constant (choice A). In "starvation," androgen levels would be expected to decrease (choice B) because these hormones increase amino acid usage in protein synthesis. Since insulin (choice D) is responsible for decreasing blood glucose levels, one would expect to see decreased insulin levels in this individual. Lipids are being broken down in this individual, not being stored (lipogenesis) (choice E).
If an individual had hepatitis, one would expect to see
A. Decreased clotting time
B. Hyperglycemia
C. Hypernatremia
D. Hypoalbuminemia
E. Hypoglycemia
hypoalbuminemia

The liver is responsible for both metabolic and hematological regulation as well as over 200 different functions. The liver is also responsible for plasma protein synthesis. The hepatocytes synthesize and release the majority of plasma proteins in the blood, including albumin. Albumin is responsible for contributing to the osmotic concentration of the blood as well as acting as a transport protein. Therefore, in individuals with hepatitis, one would expect to see hypoalbuminemia. Since the liver is responsible for the production of various clotting factors and the storage of vitamin K, one would expect to see increased clotting times (choice A). The pancreas is responsible for the production of glucagon and insulin, which have hyperglycemic (choice B) and hypoglycemic (choice E) effects, respectively. Hypernatremia (choice C) is a condition characterized by high sodium ion concentrations; this condition often occurs in individuals with renal failure.
If an individual ingested a large amount of medication that inhibited the liver from utilizing vitamin K, this individual would most likely experience which of the following?
A. Acidosis
B. Anemia
C. Hemorrhage
D. Seizure
E. Thrombocytopenia
Hemorrhage

Both vitamin K and calcium ions affect almost every aspect of the clotting process; the intrinsic, extrinsic and common clotting pathways. Adequate amounts of vitamin K must be present for the liver to be able to synthesize four clotting factors, which include prothrombin. Without vitamin K, the entire clotting system will be inactivated. Therefore, if an individual ingested a large amount of medication that inhibited the liver from utilizing vitamin K, this individual would most likely experience some degree of hemorrhage. Hemorrhage would most likely occur because the body would be unable to stop bleeding in a traumatic event. Warfarin sodium is a medication that inhibits the action of vitamin K. Acidosis (choice A) is a condition where the blood pH is less than 7.4. Anemia (choice B) is a condition marked by a reduction in the hemoglobin and/or hematocrit content of the blood. A seizure (choice D) can be defined as a condition characterized by a sudden onset of convulsions or other epileptic symptoms. Thrombocytopenia (choice E) is characterized by an abnormally low platelet count.
If an individual is dehydrated, one would expect to see increased production of the hormone necessary for preserving water. This hormone is produced in
A. Anterior pituitary
B. Hypothalamus
C. Pancreas
D. Parathyroid
E. Posterior pituitary
Hypothalamus

The hypothalamus produces antidiuretic hormone that leads to increased water reabsorption in the kidney. Therefore, if an individual is dehydrated, one would expect to see increased production of this hormone in an effort to prevent water loss. The hypothalamus also produces oxytocin, which is responsible for stimulating uterine contractions. Although both these hormones are produced in the hypothalamus, they are stored in the posterior pituitary (choice E). The anterior pituitary (choice A) produces various hormones, such as growth hormone, prolactin, adrenocorticotropin hormone, thyroid stimulating hormone, lutenizing hormone and follicle stimulating hormone, all of which do not affect blood calcium levels. The pancreas, (choice C), produces the hormones insulin and glucagon that are involved with the regulation of blood glucose levels. The parathyroid, (choice D), produces parathyroid hormone, which increases calcium blood levels in body fluids through the simulation of osteoclasts. These are large, motile multinucleated cells found on bone surfaces. When stimulated by parathyroid hormones, these cells increase blood calcium levels by the resorption of bone tissue.
If an otherwise healthy 25 year old female had her ovaries removed, one would expect to see all of the following EXCEPT
A. Amenorrhea
B. Increased blood levels of estrogen
C. Increased blood levels of follicle stimulating hormone
D. Increased blood levels of luteinizing hormone
E. Increased blood levels of gonadotropin releasing hormone
Estrogen levels increase

The ovaries are small organs located near the lateral walls of the pelvic cavity of females. These organs are responsible for the production of ova, secretion of estrogen and progestins, and the inhibition of other reproductive hormones. If the ovaries were removed, estrogen levels would decrease immediately. Estrogen is responsible for maintaining secondary sex characteristics, stimulation of the repair of the endometrium and inhibition of gonadotropin releasing hormone (GnRH) (choice E). Without estrogen, amenorrhea (choice A) will occur; this condition is characterized by a cessation of menstruation. Since estrogen inhibits the secretion of GnRH, and GnRH stimulates the secretion of both follicle stimulating hormone (FSH) (choice C) and luteinizing hormone (LH) (choice D) , one would expect to find increased blood levels of all three hormones in the absence of estrogen. Both follicle stimulating hormone and luteinizing hormone stimulate follicular development.
If it is dangerous for an infant to ingest a product containing aspartame, he most likely has which of the following genetic disorders?
A. Hyperornithemia
B. Hyperuricemia
C. Hypervalinemia
D. Phenylketonuria
E. Wilson disease
Phenylketonuria

The administration of any product that contains phenylalanine, such as aspartame, to an individual with any of the hyperphenylaninemias could be detrimental to his or her general health. The hyperphenylaninemias result from an impaired conversion of phenylalanine to tyrosine. The most common and clinicaliy important is phenylketonuria, which is characterized by an increased concentration of phenylalanine in blood, increased concentration of phenylalanine and its by-products (such as phenylpyruvate, phenylacetate, and phenyllactate) in urine, and mental retardation. Phenylketonuria is a condition caused by a deficiency of phenylalanine hydrolase. Although this is a rare condition, the U.S. Food and Drug Administration requires that the following warning be placed on all food products that contain aspartame: Caution phenylketonurics - Contains phenylalanine. Hyperornithemia (choice A) is an inherited disorder of amino acid metabolism that results from a defect of the enzyme ornithine decarboxylase. This condition is associated with mental retardation, neuropsychiatric dysfunction, and protein intolerance. Hyperyricemia (choice B) is a condition associated with higher than normal blood levels of uric acid. This condition is commonly known as gout. Hypervalinemia (choice C) is an inherited disorder of amino acid metabolism that results from a defect of the enzyme valine aminotransderase. This condition is associated with mental retardation, neuropsychiatric dysfunction, and protein intolerance. Wilson disease (choice E) is an autosomal-recessive disorder associated with an abnormality of the hepatic excretion of copper resulting in toxic accumulations of the metal in the brain, liver, and other organs.
If one individual is diagnosed with a parasitic infection and another individual is diagnosed as having an allergic reaction, one would expect to see increased blood levels of which of the following in both patients?
A. Basophils
B. Eosinophils
C. Erythrocytes
D. Megakaryocytes
E. Monocytes
The correct answer is B. Eosinophils attack foreign objects in the body that have already been coated with antibodies. They are phagocytic cells that will engulf antibody-marked bacteria, cellular debris and parasites. Since they are sensitive to circulating allergens (substances that trigger allergies), their numbers increase during allergic reactions. Basophils (choice A) migrate to the site of injury and cross the capillary endothelium to accumulate within the damaged tissues, where they discharge their granules. These granules include histamine and heparin. Erythrocytes (choice C), or mature red blood cells, are responsible for transporting oxygen throughout the body. Megakaryocytes (choice D) form platelets, which are responsible for helping blood to "clot" when injury occurs. Monocytes (choice E) are aggressive phagocytes that engulf foreign bodies and release chemicals that attract and stimulate neutrophils and other phagocytic cells.
If the conversion of T4 to T3 was inhibited, what physiological changes would one expect to see?
A. Decreased TSH levels
B. Increased body temperature
C. Increased heart rate
D. Increased thyroxine levels
E. Normal metabolic rate
Increased thyroxine levels

The thyroid gland produces large quantities of T4 (thyroxine); however, T3 (triiodothyronine) is the hormone responsible for producing the observed effects of the thyroid hormones. T4 is typically converted to T3 by enzymes in the peripheral tissues. Approximately 90% of the T3 that reaches its target cells is produced from the conversion of T4. Therefore, if the conversion of T4 to T3 were blocked, one would expect to see increased T4 (thyroxine) levels. With a decrease in the T3 levels in the blood. the body will compensate by increasing the secretion of thyroid stimulating hormone (choice A) in an effort to increase both T4 and T3 levels. Since T3 is primarily responsible for the physiological effects of thyroid hormones, one would expect to see decreased body temperature (choice B), decreased heart rate (choice C) and decreased metabolic rate (choice E).
If the digestion of a carbohydrate results in large amounts of the monosaccharides glucose and galactose, the parent compound was most likely
A. Cellulose
B. Glycogen
C. Lactose
D. Maltose
E. Sucrose
lactose

Prior to absorption, disaccharides and polysaccharides must be broken down into simple sugars by a variety of enzymes. For example, the enzyme lactase causes the release of a molecule of glucose and galactose from the hydrolysis of the disaccharide lactose. Cellulose (choice A) is a structural component found in many plants; the body cannot digest this compound at all. Glycogen (choice B) is a branched polysaccharide composed of interconnected glucose molecules; when this compound is broken down, many glucose molecules are formed. The enzyme maltase splits the bonds in a molecule of the disaccharide maltose (choice D) to yield two glucose molecules. Glucose and fryctose are formed when sucrose (choice E) is hydrolyzed by the enzyme sucrase.
If the thymus decreased the production of thymosins, one would expect to see a decrease in which of the following?
A. B-lymphocytes
B. Complement
C. Cytotoxic T cells
D. NK cells
E. Neutrophils
Cytotoxic T cells

Lymphopoiesis (or lymphocyte production) involves the bone marrow, thymus and peripheral lymphatic tissues. The bone marrow produces hemocytoblasts which eventually generate lymphocytic stem cells responsible for &e production of lymphocytes. The lymphocytic stem cells that migrate to the thymus to differentiate into T cells do so under the influence of "thymosins" Therefore, if the thymus decreased production of thymosins, there would be a decrease in the number of T cells in the body, including cytotoxic T cells. If the lymphocytic stem cells remain in the bone marrow, they will differentiate into immature B cells (choice A) and NK (natural killer)cells (choice D). Once matured, the B cells will move to the lymph nodes and the NK cells will roam throughout the body in search of abnormal cells. Complement (choice B) is a system of circulating proteins that assists antibodies in the destruction of pathogens through the breakdown of their cell walls. Complement is not affected by thymosin secretion. Neutrophils (choice E) or PMN's attack cells or pathogens that have been "marked" with antibodies or complement proteins. The production of these cells is stimulated by colony stimulating factors, such as GSF.
If the-action of 17-alpha-hydroxylase is inhibited, the formation of which of the following is most likely to be decreased?
A. Aldosterone
B. Cholesterol
C. Cortisol
D. Progesterone
E. Testosterone
Testosterone

When cholesterol (choice B) comes in contact with the enzyme cholesterol desmolase, the substance pregnenolone is formed. This compound is generally involved in two enzymatic processes. When pregnenolone is reacted with 17-alpha hydrolase, it will be converted to the compound 17-hydroxypregnenolone. After a series of enzymatic reactions, it will eventually be converted to testosterone. Therefore, if the action of 17-alpha-hydroxylase is inhibited, the formation of testosterone is most likely to be decreased. If pregnenolone is reacted with 3-beta-hydroxysteroid dehydrogenase, it will be converted to progesterone (choice D). When progesterone undergoes a series of enzymatic reactions, it can be converted to either aldosterone (choice-A) or cortisol (choice C).
If, through a traumatic event, the blood flow to the kidneys decreases, one would expect to see which of the following?
A. Decreased aldosterone secretion
B. Decreased water reabsorption
C. Dilation of the efferent arteriole
D. Increased renin secretion
E. Increased systemic vasodilation
Increased renin secretion

The glomerular filtration rate depends on the filtration pressure moving across the glomerular capillaries. When there is a decline in the renal blood pressure, the kidney will take several actions to increase this low blood pressure. Renin release increases when the glomerular pressure decreases or the osmolarity of f glucose the tubular fluid reaching the distal tubule decreases. Renin is a hormone released by the juxtaglomerular apparatus in the kidneys. This hormone converts the inactive protein angiotensinogen to angiotensin I. Once this occurs, a converting enzyme will convert angiotensin I to angiotensin II, which has the following physiological effects: Peripheral vasoconstriction, constriction of the efferent arteriole (choice C), stimulation of the reabsorption of sodium ions and water in the proximal tubule and triggering the release of two hormones (antidiuretic hormone and aldosterone (choice A). All of these factors serve to increase blood pressure. Decreasing water reabsorption (choice B) will further lower blood volume and blood pressure. Systemic vasodilation (choice E) will draw blood from the internal organs, including the kidneys, to the peripheral tissues.
In a comparison of the sympathetic and parasympathetic divisions of the autonomic nervous system, all of the following are true EXCEPT
A. Acetylcholine is secreted by the postganglionic fibers of both the sympathetic and parasympathetic nervous systems.
B. Epinephrine is secreted by some of the ganglionic neurons in the sympathetic nervous system.
C. The parasympathetic nervous system promotes energy storage and the sympathetic nervous system promotes usage of energy reserves.
D. The preganglionic neurons of the sympathetic nervous system are relatively short and the preganglionic neurons of the parasympathetic nervous system are relatively long.
E. The sympathetic division is primarily located between spinal segments T1 and L2 and the parasympathetic division is primarily located between spinal segments S2 and S4
Acetylcholine is secreted by the postganglionic fibers of both the sympathetic and parasympathetic nervous systems.


The correct answer is A. The sympathetic and parasympathetic divisions of the autonomic nervous system have distinctive characteristics. For example, acetylcholine is secreted by the postganglionic fibers of parasympathetic nervous system, whereas, norepinephrine is primarily secreted by the post ganglionic fibers of the sympathetic nervous system. Epinephrine is secreted by the ganglionic neurons in the sympathetic nervous system, that release this hormone directly into the circulatory system (choice B) The parasympathetic nervous system promotes energy storage (rest) and the sympathetic nervous system promotes usage of energy reserves. (fight or flight) (choice C). The preganglionic neurons of the sympathetic nervous system are relatively short and the preganglionic neurons of the parasympathetic nervous system are relatively long (choice D). With respect to general anatomy the sympathetic division is primarily located between spinal segments T1 and L2, and the parasympathetic division is primarily located between spinal segments S2 and S4 (choice E).
In addition to causing beriberi, thiamine deficiency can lead to the development of which of the following?
A. Carotenemia
B. Korsakoff syndrome
C. Pellagra
D. Pernicious anemia
E. Scurvy
Korsakoff syndrome

Beriberi is a condition caused by a deficiency of thiamine. It typically occurs in alcoholics because of impaired absorption. Cardiovascular complications secondary to beriberi include a high-output state, biventricular myocardial failure, and retention of sodium. Another complication of thiamine deficiency is Korsakoff syndrome. This condition is mostly associated with neurologic complications. Carotenemia (choice A) results from excessive intake of vitamin A and leads to a yellowing of the skin, especially on the palms and soles. A deficiency of niacin can lead to pellagra (choice C), which is a disease typically associated with dermatitis, dementia, and diarrhea. Pernicious anemia (choice D) is a condition caused by a deficiency of vitamin B12, or cyanocobalamin. Scurvy (choice E) is caused by a deficiency of vitamin Cand can lead to perifollicular hemorrhage.
Which of the following characterize the links between monomeric units of nucleic acids?
1. Ionic bonds
2. Glycosidic bonds
3. Phosphodiester bonds
4. Phosphotriester bonds
3. Phosphodiester bonds


The correct answer is choice 3. Monomeric units of DNA consist of a phosphate group, deoxyribose, sugar, and nitrogen base of adenine, guanine, cytosine or thymine. These monomeric units are linked together to form the DNA strand by phosphodiester bonds. In these bonds, the 3-prime carbon of one deoxyribose links with one oxygen of the phosphate group, while a 5-prime carbon of another deoxyribose links with another oxygen of the phosphate group. This is known as a phosphodiester bond, indicating two bonds to a phosphate group. So the correct answer to question is choice 3.
Which of the following enzymes converts trypsinogen to trypsin?
A. Enterokinase
B. Peptidase
C. Secretin
D. Pepsin
A. Enterokinase


The correct answer is Choice A. Trypsin is a powerful proteolytic enzyme present in pancreatic secretions. It aids in the digestion of proteins. When synthesized in the pancreatic cells, the proteolytic enzyme is in its inactive form trypsinogen. It is important for the enzyme to remain inactive as it will digest the pancreas itself. Trypsinogen is activated to trypsin only after it is released into the intestinal tract by enterokinase (Choice A). Enterokinase is secreted by the intestinal mucosa when chyme comes into contact with the mucosa.
Peptidase (Choice B) is an enzyme present in the microvilli of the small intestine capable of splitting large polypeptides into their components.
Secretin (Choice C) is a hormone secreted by duodenal and jejunal mucosa in response to high acid content in the small intestine. This hormone in turn stimulates pancreatic secretion of sodium bicarbonate.
Pepsin (Choice D) is the major digestive enzyme of the gastric juices. It is active in an acidic environment to digest proteins and collagen.
Which of the following enzymes is involved in the synthesis of fatty acids?
A. Acetyl CoA carboxylase
B. Glucose 6-phosphate dehydogenase
C. Glycogen synthase
D. Phosphorylase
E. Thiolase
A. Acetyl CoA carboxylase


The correct answer is A. The synthesis of lipids is known as lipogenesis. Pathways of lipid synthesis begin with acetyl-CoA; these molecules can be stmng together in the cytosol yielding fatty acids. Acetyl CoA carboxylase is the key enzyme involved in fatty acid synthesis. Glucose 6- phosphate dehydogenase (choice B) is involved in the pentose phosphate pathway. The two enzymes involved in the synthesis and degradation of glycogen are phosphorylase and glycogen synthase. Furthermore, glycogen synthesis and degradation are coordinated by a hormone-triggered cascade, which ensures that when one enzyme is active, the other enzyme is inactive. The enzyme involved in glycogen degradation is phosphorylase (choice D) and the enzyme involved in glycogen synthesis is glycogen synthase (choice C), as the name implies. Thiolase (choice E) is responsible for converting acetoacetyl CoA into acetyl CoA.
Which of the following enzymes is responsible for catalyzing the first step in fatty acid synthesis?
A. Acetyl-CoA carboxylase
B. Carnitine-acyl transferase
C. Carnitine translocase
D. Fatty acid synthase
E. Lecithin cholesterol acyl transferase
A. Acetyl-CoA carboxylase


The correct answer is A.
The enzyme responsible for catalyzing the first step in fatty acid synthesis is acetyl-CoA carboxylase. This enzyme requires the presence of both biotin and bicarbonate for converting acetyl-CoA to malonyl-CoA. Fatty acid synthase (choice D) is a multienzyme complex that catalyzes the remaining reactions for fatty acid synthesis. Carnitine-acyl transferase I (choice B) transfers the fatty acid from fatty acid acyl-CoA to carnitine to form acyl carnitine. Carnitine translocase (choice C) transports fatty acyl carnitine into the mitochondria and transports free carnitine back outside the mitochondria. Lecithin cholesterol acyl transferase (choice D) is a plasma enzyme that esterifies HDL cnolesterol.
Which of the following functions as part of the extracellular matrix?
A. Mucin
B. Heparin
C. Collagenase
D. Chondroitin sulfate
E. Dolichol phosphate
D. Chondroitin sulfate


The correct answer is Choice D. Of the following, only chondroitin sulfate (Choice D) functions as part of the extracellular matrix. It is a proteoglycan that acts as a filler between collagen fibers and cells. Chondroitin sulfate is also a principal component in the organic matrix of bone and cartilage.
Mucin (Choice A) is a mucus secretion produced by salivary glands that acts as a lubricant for surface protection.
Heparin (Choice B) is a powerful anticoagulant produced mainly by the basophilic mast cells located in the pericapillary connective tissues.
Collagenase (Choice C) is an enzyme that catalyzes the hydrolysis of collagen. It is not a functional part of normal extracellular matrix.
Dolichol phosphate (Choice E) is a very long chain lipid that acts as a carrier for oligosaccharide.
Which of the following groups includes only amino acids essential for humans?
A. Valine, serine, leucine
B. Leucine, lysine, glycine
C. Tyrosine, threonine, tryptophan
D. Phenylalanine, methionine, proline
E. Tryptophan, methionine, isoleucine
E. Tryptophan, methionine, isoleucine


The correct answer is Choice E. Amino acids are the basic building blocks of proteins. They are classified into two separate groups, the essential and the nonessential amino acids. The nonessential group includes amino acids that the body can synthesize. The essential amino acid group include ones that the body either cannot synthesize or are synthesized in amounts too small to meet the needs of the body. These amino acids must be supplied in the diet. The following is a list of essential amino acids:
1. Threonine
2. Lysine
3. Methionine
4. Arginine
5. Valine
6. Phenylalanine
7. Leucine
8. Tryptophan
9. Isoleucine
10. Histidine
Of the choices listed, Choice E (tryptophan, methionine, isoleucine) contains only essential amino acids.
Which of the following hormones are produced by the neurohypophysis and affect contraction of smooth muscle?
1. Vasopressin (ADH) and oxytocin
2. Thyrotropin and intermedin
3. Adrenotropin and somatotropin
4. Aldosterone and luteinizing hormone
1. Vasopressin (ADH) and oxytocin


The correct answer is choice 1. The neurohypophysis is another term for the posterior pituitary. The two major hormones produced by the posterior pituitary are ADH, also known as vasopressin, and oxytocin. ADH raises blood pressure and helps retain fluid by increasing permeability of collecting tubules in the kidney. Oxytocin causes uterine contraction and milk secretion. In choice 2, thyrotropin or TSH comes from the anterior pituitary and stimulates the thyroid gland; while intermedin or melanotropin is an anterior pituitary hormone that stimulates melanin production. And choice 3, adrenotropin or ACTH is another anterior pituitary hormone that stimulates the adrenal cortex, while somatotropin or growth hormone is another anterior pituitary hormone that stimulates overall body growth. And choice 4, aldosterone is an adrenal cortical hormone primarily responsible for sodium retention, while luteinizing hormone is an anterior pituitary hormone involved in triggering ovulation and formation of the corpus luteum. So the correct answer to question is choice 1.
Which of the following hormones conserves body protein, carbohydrate, and fat stores?
A. Cortisol
B. Glucagon
C. Insulin
D. Somatotropin
C. Insulin


The correct answer is Choice C. Of the hormones listed, only insulin (Choice C) conserves body protein, carbohydrates, and fat stores. It inhibits the breakdown of proteins in cells, causes carbohydrates to be stored as glycogen in liver muscles, and it stimulates fat storage in adipose tissue.
Cortisol, Choice A, is a potent glucocorticoid that stimulates gluconeogenesis by the liver. It also reduces cellular protein and promotes mobilization of fatty acids from adipose tissue. Glucagon is secreted by the alpha cells of the islets of Langerhans. Its primary function is to increase blood glucose concentration by stimulating breakdown of liver glycogen, and activation of adipose cell lipase making increased quantites of free fatty acids. These functions are opposite of insulin.
Somatotropin, Choice D, is also known as growth hormone. It promotes growth of almost all tissues of the body by conserving carbohydrates, enhancing body proteins, and increasing utilization of fat stores for energy.
Which of the following hormones plays a crucial role in the maintenance of normal body temperature?
A. Calcitonin
B. Insulin
C. Oxytocin
D. Parathyroid hormone
E. Thyroxine
E. Thyroxine


The correct answer is E.
Thyroid hormones, such as a thyroxine and triiodothyronine, affect almost every cell in the body because they readily cross cell membranes. The effects of thyroid hormones on peripheral tissues include: elevation of oxygen consumption and the rate of energy consumption, which leads to an elevation of body temperature; increased heart rate; stimulation of activity of other endocrine tissues; maintenance of sensitivity of respiratory centers as well as various other functions. Calcitonin (choice A) is primarily responsible for decreasing of calcium ions in body tissues. Insulin (choice B) facilitates the uptake of glucose by cells and stimulates lipid and glycogen formation and storage. Oxytocin (choice C) is a hormone that promotes uterine contractions and milk ejection. The parathyroid hormone (choice D) is responsible for increasing calcium ion concentrations in the body fluids.
Which of the following immunoglobulins is the first antibody secreted after the arrival of an antigen, and plays a role in the determination of ones blood type?
A. IgA
B. IgD
C. IgE
D. IgG
E. IgM
E. IgM


The correct answer is E. In the human body, there are five classes of antibodies or immunoglobulins: IgA, IgD, IgE, IgG, and IgM. Each immunoglobulin is classified based on variations in the structure of the heavy chain constant segment. IgM is the first antibody secreted after the arrival of an antigen. When the concentration of IgG increases in the affected area, IgM levels decrease. IgM antibodies, also known as agglutinins, are responsible for the agglutination of cross-matched blood and are so used to determine an individuals blood type. IgA (choice A) is primarily found in glandular secretions and functions primarily to attack pathogens before they gain access to internal tissues. IgD (choice B) is found on the surface of B cells. These immunoglobulins help to bind antigen molecules. IgE (choice C) attaches to the exposed surfaces of basophils and mast cells. When an antigen binds to the IgE molecule, the cell is stimulated to release histamine, which is the primary cause of allergic reactions. IgG (choice D) is the largest antibody found in the body. It is responsible for providing "resistance" against many different bacteria, viruses, and bacterial toxins.
Which of the following important nutrients is present in milk in low concentration?
1. Iron
2. Protein
3. Calcium
4. Phosphorus
5. Carbohydrate
1. Iron


The correct answer is choice 1, iron. Milk is high in protein, in particular, complete protein, calcium, phosphorus and carbohydrate, particularly lactose. However, it is deficient in iron and also deficient in niacin. So the correct answer to question is choice 1.
Which of the following is a classical decomposition reaction?
A. A + B -> AB
B. AB -> A + B
C. A + B <-> AB
D. AB <-> A + B
E. AB + CD <-> AD + CB
B. AB -> A + B


The correct answer is B. A decomposition reaction breaks a molecule into smaller fragments. The classical synthesis reaction is characterized by the symbols A + B -> AB. This shows two raw materials joining to form one molecule. Conversely, a decomposition reaction AB -> A + B (choice B) is the opposite of a synthetic reaction. Here, one molecule breaks down into two smaller components. Choices C (A + B <-> AB) and D (AB <-> A + B) are essentially the same type of reaction; an equilibrium reaction. In an equilibrium reaction there is a decomposition reaction followed by a synthesis reaction or vise versa. These reactions occur on a continual basis and are reversible. The reaction AB + CD <-> AD + CB is termed an exchange reaction because the two initial compounds (AB and CD) decompose and form two different compounds (AD and CB) usually by exchanging ions.
Which of the following is a principal action of insulin?
1. To mobilize lipid deposits
2. To enhance cell permeability to glucose
3. To decrease cell permeability to glucose
4. To conserve glucose by breaking down amino acids
2. To enhance cell permeability to glucose


The correct answer is choice 2. Insulin acts primarily to reduce blood glucose levels. It is made by beta cells in the pancreas. Its actions include increasing cell permeability to glucose in fat tissue and muscle; therefore, glucose is taken out of the blood by these tissues. It also increases fatty acid synthesis by fat tissue and glycogen synthesis by the liver. It also decreases the release of gluconeogenic amino acids by the liver and decreases protein breakdown. So choices 1, 3 and 4 are incorrect and opposite. Insulin will tend to cause formation, not degradation, of lipids; increase, not decrease, cell membrane permeability to glucose; and cause decreased, not increased, breakdown of amino acids. So the correct answer to question is choice 2.
Which of the following is a reflex mediated by the vagus?
1. Bile flow from the liver
2. Mucus secretion from the Brunner's glands
3. Cephalic phase of gastric secretion
4. Pancreatic secretion of bicarbonate
3. Cephalic phase of gastric secretion


The correct answer is choice 3. The vagus nerve functions primarily to promote digestion. It increases acid production in the stomach. Selective vagotomy is sometimes utilized as a treatment for chronic gastric ulcers. Choice 1, bile flow from the liver, is incorrect. The vagus nerve causes contraction of the gallbladder, not the liver, to increase bile flow. Choice 2, mucus secretion from the Brunner's glands, is incorrect. Mucus secretion in the duodenum is stimulated by the presence of gastrin, secretin and cholecystokinin. Choice 4, pancreatic secretion of bicarbonate, is incorrect. Pancreatic secretion of bicarbonate is under the control of secretin. So the correct answer to question is choice 3.
Which of the following is MOST likely to cause respiratory acidosis?
A. Moving from high altitude to sea level
B. Untreated diabetes mellitus
C. Hypoventilation
D. Kidney failure
E. Vomiting
C. Hypoventilation


The correct answer is Choice C. Respiratory acidosis is a condition caused by retention of C02 due to abnormal or inadequate pulmonary ventilation. This may be a result of pathologic conditions that damage the respiratory centers in the brain or that decrease the lung's ability to eliminate C02 such as obstruction or pneumonia.
Of the following choices, hypoventilation (Choice C) is the most likely cause of respiratory acidosis as discussed above. Inadequate pulmonary ventilation increases H2CO3 and hydrogen ion concentration in the blood resulting in acidosis.
Moving from high altitude to sea level (Choice A) will not cause respiratory acidosis. Respiratory alkalosis, however, may occur when a person ascends from sea level to high altitude.
Untreated diabetes mellitus (Choice B) may result in acidosis but it is caused by pathologic levels of insulin and not by an abnormality in respiration.
Kidney failure (Choice D) can lead to acidosis resulting from failure of the kidney to eliminate acidic products from the body. It is also not a cause of respiratory acidosis.
Which of the following is NOT a monosaccharide?
1. Amylose
2. Glucose
3. Fructose
4. Glyceraldehyde
5. Glucuronic acid
1. Amylose


The correct choice to question is #1, amylose. Amylose is an unbranched polyglycan of starch. It is not a monosaccharide. You should know that both glucose, C6H12O6 is a monosaccharide, and fructose is also a monosaccharide. Remember, a monosaccharide is a carbohydrate that cannot form any simpler sugar by simple hydrolysis. The pentoses and hexoses are monosaccharides. Both glyceraldehyde and glucuronic acid are also monosaccharides. So again, the correct choice to question is #1, amylose.
Which of the following isa present in plasma but absent in serum?
1. Albumin
2. Globulin
3. Lecithin
4. Fibrinogen
5. Prothrombin
4. Fibrinogen


The correct answer is choice 4. And we will shortly bring up a problem with answer choice 4, but let's review the choices anyway. Plasma is the fluid noncellular portion of the blood containing water, various ions, inorganic compounds, proteins and other organic compounds. If whole blood is allowed to clot and the clot is removed, the remaining liquid is known as serum. Serum has essentially the same composition as plasma, except that fibrinogen and various clotting factors -- in particular II, V and VIII -- have been removed. Plasma and serum do have similar composition in terms of albumin, globulin and lecithin. In what appears to be an oversight, prothrombin, also known as clotting Factor II, is removed from plasma when clotting occurs and should apparently be viewed as another correct answer to question. So anyway, the official answer is choice 4, but choice 5 is also physiologically correct.
Which of the following is responsible for protecting tooth enamel and binding toxic tannins?
A. High-proline protein
B. immunoglobulin A
C. Kallikrein
D. Lysozyme
E. Mucins
A. High-proline protein


The correct answer is A.
High-proline proteins are responsible for protecting tooth enamel nd binding toxic tannins. lmmunoglobulin A (choice B) helps defend against bacteria on mucosal surfaces. Kallikrein (choice C) is a proteolytic enzyme that cleaves kininogens to form bradykinin. Lysozymes (choice D) break down the cell walls of bacteria. Mucins (choice E) are glycoproteins that lubricate the food and protect buccal and esophageal mucosa from scratching.
Which of the following is responsible for secreting substances that have a basic pH?
A. Cholecystokinin
B. Gastrin
C. Hydrochloric acid
D. Intrinsic factor
E. Secretin
E. Secretin


The correct answer is E.
Secretin is synthesized and stored in the S cells of the upper intestine. It stimulates the secretion of bicarbonate-containing substances from the pancreas and inhibits gastric emptying and gastric acid production. Bicarbonate containing sutistances are "basic" by chemical nature. Cholecystokinin (choice A) is produced and stored in the cells of the duodenal and jejunal mucosa. The parietal cell secretes two substances: intrinsic factor and hydrochloric acid. The hormone gastrin (choice B), which is produced in the G cells of the duodenum, primarily functions to stimulate hydrochloric acid, histamine, and pepsinogen secretion, as well as increase gastric blood flow. Hydrochloric acid (choice C) functions primarily to denature proteins and activate digestive enzymes. Intrinsic factor (choice D) is the secretion of parietal cells that facilitates the absorption of vitamin B12 across the intestinal lining. Vitamin B12 is a member of the cobalamin family and serves as a cofactor for two reactions in humans: the methylation of homocysteine to methionine and conversion of methylmalonyl-CoA to succinyl-CoA. The most common disorder seen with vitamin B12 deficiency is pernicious anemia, which is associated with the development of a megaloblastic anemia.
Which of the following is the major building block, comprising greater than 50%, of all biologic membranes?
A. Cholesterol
B. Glycolipids
C. Integral membrane proteins
D. Peripheral membrane proteins
E. Phospholipids
E. Phospholipids


The correct answer is E.
Lipids are the primary constituents of all biologic membranes. Phospholipids are the major building block of all biologic membranes, comprising between 50% and 60% of the total membrane lipid. Although cholesterol (choice A) is a major component of plasma membranes, it comprises between 15% and 25% of the total membrane. Glycolipids (choice B) generally make up the remainder of the lipid portion of the cell membrane. Integral membrane proteins (choice C) are embedded in the lipid bilayer and cannot be removed without disrupting the membrane. Peripheral membrane proteins (choice D) are also found in the lipid-based membrane; however, these proteins can be removed.
Which of the following is the major contributor to colloid osmotic pressure?
1. Albumin
2. Alpha globulin
3. Beta globulin
4. Gamma globulin
1. Albumin


The correct choice is #1, albumin. Plasma osmotic pressure is a direct result of plasma proteins in the bloodstream. Plasma proteins such as albumin have osmotic activity because of their inability to permeate the membrane. They are the most abundant protein relative to globulins and fibrinogen: therefore, due to their abundance they are the major contributors to osmotic colloidal pressure. The effective colloidal osmotic pressure is 25 mmHg. This may decrease or increase in plasma as the concentrations of plasma proteins vary. Alpha globulin, beta globulin and gamma globulin are not in abundance relative to albumin, and therefore are not major contributors to the colloid osmotic pressure. Again, the correct choice is #1, albumin.
Which of the following is the region/nucleus of the hypothalamus responsible for controlling the feeding reflexes, such as swallowing and licking?
A. Mamillary bodies
B. Paraventricular nucleus
C. Preoptic area
D. Supraoptic nucleus
E. Tuberal area
A. Mamillary bodies


The correct answer is A. The hypothalamus contains a variety of important control and integrative centers as well as those associated with the limbic system. The mamillary bodies are the region of the hypothalamus responsible for controlling the feeding reflexes, such as swallowing and licking. The paraventricular nucleus (choice B) is responsible for secreting oxytocin. The preoptic area (choice C) is responsible for regulating body temperature. The supraoptic nucleus (choice D) secretes Antidiuretic hormone, which restricts water loss in the kidneys. The tuberal area (choice E) is the portion of the hypothalamus that releases hormones that control the endocrine cells of the anterior pituitary.
Which of the following minerals is responsible for the proper functioning of the enzyme carbonic anhydrase?
A. Copper
B. Magnesium
C. Manganese
D. Phosphorus
E. Zinc
E. Zinc


The correct answer is E. Minerals (and vitamins) are essential components of the diet because the body is unable to synthesize the majority of these compounds. Minerals are inorganic ions typically released through the dissociation of electrolytes. Zinc is a mineral responsible for the proper functioning of the enzyme carbonic anhydrase and acts as a cofactor of various other enzyme systems. When individuals are zinc deficient, the immune system may become compromised. Copper (choice A) acts as a cofactor for hemoglobin synthesis. Magnesium (choice B) is a cofactor of enzyme systems and is responsible for normal membrane functioning. It is also a constituent of bones and teeth. Manganese (choice C) is also a cofactor for various enzyme systems; however, it is not involved with the enzyme carbonic anhydrase. Phosphorus (choice D) is a constituent of bones, teeth, and ATP phosphorylated metabolic intermediates.
Which of the following must be digested before being in a form that can be absorbed by enterocytes?
A. Monoglycerides
B. Fatty acids
C. Fructose
D. Glycine
E. Maltose
E. Maltose


The correct answer is Choice E. Foods from which the human body obtains nutrition can be classified into carbohydrates, fat, and proteins. These substances generally cannot be absorbed by the intestinal mucosa without the process of digestion. Carbohydrates, for example, are mainly large disaccharides and polysaccharides that must be hydrolyzed before they can be utillized by the enterocytes of the small intestine. Maltose (Choice E), a disaccharide found in starch that must be split into its constituent monosaccharide before it can be absorbed by the enterocytes.
Monoglycerides (Choice A) and fatty acids (Choice B) are the digestive end products of fat. They are both readily (glucose) absorbable by enterocytes.
Fructose (Choice C) is one of the monosaccharides formed from digestion of cane sugar. It is already in a form that can be absorbed by enterocytes via facilitated diffusion. Glycine (Choice D), an amino acid, can also be absorbed directly by enterocytes without further digestion.
Which of the following occurs in a normal menstrual cycle?
1. Ovulation generally occurs 10 days from the beginning of the cycle.
2. The ovum generally disintegrates or becomes nonviable if it is not fertilized within four days.
3. The blood concentration of estradiol increases as the corpus luteum develops.
4. The blood level of follicle-stimulating hormone increases and the level of luteinizing hormone decreases at the time of ovulation.
2. The ovum generally disintegrates or becomes nonviable if it is not fertilized within four days.


The correct answer is choice 2. Nonfertilized ova will generally degenerate within about four days. Choice 1 is incorrect. Ovulation occurs approximately on the 14th day of the cycle, although the exact time will vary. Choice 3 is incorrect. Estradiol, or estrogen, secreted by the ovary will decrease as the corpus luteum develops. The corpus luteum formed after ovulation secretes large amounts of progesterone and much smaller amounts of estrogen than at the peak of the follicular stage. Choice 4 is incorrect and opposite. Slightly before ovulation, FSH levels decline and then slowly rise, while LH levels rise dramatically in a spike-like fashion. The great increase in LH, or luteinizing hormone, is largely responsible for ovulation. So the correct answer to question is choice 2.
Which of the following occurs last in the generation of an action potential?
A. Sodium ions enter the cell
B. The transmembrane potential goes from - 70 mV to + 30 mV
C. Voltage-regulated potassium channels open
D. Voltage-regulated sodium channels close
E. Voltage-regulated sodium channels open
C. Voltage-regulated potassium channels open


The correct answer is C. Action potentials are propagated changes in the transmembrane potential that spread across an entire excitable membrane . In a given neuron, an action potential usually begins at the initial segment of the axon and is conducted along the length of the axon. The steps in the generation of an action potential are as follows: A graded depolarization brings an area of excitable membrane to a threshold. The voltage regulated sodium channels open (choice E) which permits sodium ions to enter the cell (choice A). Once this occurs, the transmembrane potential goes from - 70 mV to + 30 mV (choice B). The voltage-regulated sodium channels then close (choice D) and the voltage regulated potassium channels open (choice C). In other words, the opening of the potassium channels is the last step that occurs based on the given answer choices. Once the potassium channels open, potassium ions move out of the cell and repolarization begins.
Which of the following portions of the cardiovascular system contains the greatest volume of blood?
A. Arterioles
B. Capillaries
C. Systemic veins
D. Chambers of the heart
E. Pulmonary vasculature
C. Systemic veins


The correct answer is Choice C. The greatest volume of blood by far is contained in the systemic veins (Choice C). It comprises 64% of blood in the systemic circulation. The veins serve as a major reservoir of blood for the body along with their important function as conduits for transport of blood from tissues back to the heart.
The arterioles (Choice A) and capillaries (Choice B) carry approximately 7% of the blood.
The chambers of the heart (Choice D) also contain 7% of the blood.
The pulmonary vasculature (Choice E) holds 9% of blood volume.
Which of the following portions of the nephron has a primary function of only reabsorbing water from filtrate?
A. Ascending loop of Henle
B. Collecting tubules
C. Descending loop of Henle
D. Distal convoluted tubule
E. Proximal convoluted tubule
B. Collecting tubules


The correct answer is B. The primary function of the nephron is to clear the plasma of unwanted substances as it passes through the kidney. Since plasma contains both wanted and unwanted substances, the nephron will selectively reabsorb these "wanted" substances, at selected portions of the nephron, back into the plasma and the remaining components are subsequently excreted into the urine. The descending loop of Henle (Choice C) and the ascending loop of Henle (choice A) are primarily responsible for the reabsorption of ions and assisting in the creation of the medullary concentration gradient. The collecting tubules (choice B) are primarily responsible for the resorption of water, under the control of the hormone ADH. The distal convoluted tubule (choice D) is the primary site for secretion of substances into the filtrate as well as some reabsorption of water and sodium ions. The primary site of nutrient reabsorption in the nephron is the proximal convoluted tubule (choice E).
Which of the following reflexes complements the withdrawal reflex by making compensatory adjustments on the opposite side of the body receiving the stimulus?
A. Crossed extensor reflex
B. Flexor reflex
C. Patellar reflex
D. Stretch reflex
E. Tendon reflex
A. Crossed extensor reflex


The correct answer is A. Reflexes are classified according to the site where the information is being processed, the nature of the resulting motor response or the complexity of the neural circuit involved. The crossed extensor reflex is classified as a contralateral reflex because of the motor response that occurs on the opposite side of the body from that receiving the stimulus. For example, if one were to step on a nail, the flexor reflex (a type of withdrawal reflex) would pull the foot away from the nail and the crossed extensor reflex would "stiffen" the other leg to support the body. As previously mentioned, the flexor reflex (choice B) will "flex" the muscles in the affected limb to pull away from the painful stimulus. The patellar reflex (choice C) is a type of stretch reflex (choice D) that results in a reflexive kick when properly stimulated. The stretch reflex provides automatic regulation of muscle length. The tendon reflex (choice E) prevents a muscle from exerting too much tension.
Which of the following represent(s) the matrix proteins of enamel?
A. Carboxylglutamic acid containing proteins
B. Type I collagen
C. Amelogenins
D. Proteoglycans
D. Elastin
C. Amelogenins


The correct answer is Choice C. As discussed, the human tooth enamel is made up of approximately 95-98% of inorganic matter by weight. The remaining structure is composed of an organic matrix and water. Protein such as amelogenin (Choice C) is formed in the matrix.
Choice A, carboxylglutamic acid containing protein, is not found in enamel.
Choice B, type 1 collagen, is found in skin, tendon, bone and cornea but does not represent matrix protein of enamel.
Choice D proteoglycans along with extracellular fluid make up the ground substances in bone.
Choice E elastin is also a connective tissue protein. It is the major component of elastic fibers that can stretch to several times their length. Large amounts of elastin are found in the walls of blood vessels and ligaments. It is not present in enamel.
Which of the following represents a soluble polysaccharide found in dental plaque and is formed from the fructose moiety of sucrose?
1. Levan
2. Dextran
3. Amylopectin
4. Hyaluronic acid
1. Levan


The correct answer is choice 1, levan. Levan is a polyfructose synthesized by plaque bacteria. Levan is a stimulator of B cells and individuals with periodontal disease exhibit greater response to this polysaccharide than do normal control individuals. Choice 2, dextrans is incorrect. Dextrans are water soluble, high molecular weight glucose polymers. Choice 3, amylopectins, is incorrect. Amylopectins are branched chain polyglucose molecules or glucans. Choice 4. hyaluronic acid, is incorrect. This is a component of the ground substance that makes up the intercellular matrix of the gingival connective tissue. So the correct answer to question is choice 1.
Which of the following represents the amount of dietary calcium normally absorbed from the gut of an adult man?
1. An amount less than 50%
2. An amount between 60-70%
3. An amount between 80-100%
4. An amount dependent upon the mucosal ferritin level
1. An amount less than 50%


The correct answer is choice 1, an amount less than 50%. Less than 50% of the dietary calcium is absorbed through the small intestine via active transport. This active transport is mediated by a carrier synthesized from vitamin D. Choices 2 and 3 are therefore incorrect. Choice 4 is incorrect. Ferritin is responsible for the transport of iron in the bloodstream and is not connected to the transport of calcium. So the correct answer to question is choice 1.
Which of the following represents the chemical substance that is the immediate source of energy for muscle contraction?
A. Glycogen
B. AcetylCoA
C. Lactic acid
D. Creatine phosphate
E. Adenosine triphosphate
E. Adenosine triphosphate


The correct answer is Choice E. When a muscle contracts, work is performed and energy is required for this process. The immediate source of energy is provided by the cleavage of adenosine triphophate (ATP) (Choice E) to form adenosine diphosphate (ADP). The concentration of ATP present in muscle fibers is limited and only sufficient to maintain full contraction for 1 to 2 seconds. As muscle contraction continues, creatine phosphate (Choice D) is used to rephosphorylate ADP to form new ATP. The amount of creatine phosphate, however, is also limited. It will carry the muscle contraction for another 3-5 seconds. Glycogen (Choice A) is then used to reconstitute both ATP and creatine phosphate.
So though both glycogen (Choice A) and creatine phosphate (Choice D) are important energy sources during muscle contraction, they are not the initial or immediate source of energy.
Acetyl CoA (Choice B) reacts in the citric acid cycle to release large amounts of energy to convert ADP to ATP in the mitochondria. It is not, however, used by the muscle during its contraction. Lactic acid (Choice C) is an end product in the breakdown of glycogen. It is also not a primary energy source.
Which of the following spinal tracts controls the involuntary regulation of eyes, head, neck and upper limb position in response to either a visuai or auditory stimulus?
A. Corticobulbar tract
B. Reticulospinal tract
C. Rubrospinal tract
D. Tectospinal tract
E. Vestibulospinal tract
D. Tectospinal tract


The correct answer is D. In the extrapyramidal spinal tract are the following: The tectospinal tract controls the involuntary regulation of eye, head, neck and upper limb position in response to either a visual or auditory stimulus. The reticulospinal tract (choice B) controls the involuntary regulation of reflex activity and autonomic function. The rubrospinal tract (choice C) controls the involuntary regulation of posture and muscle tone. The vestibulospinal tract (choice E) controls the involuntary regulation of balance and muscle tone. The pyramidal portion of the spinal tracts contain the corticobulbar (choice A), lateral corticospinal and anterior corticospinal tracts. All three tracts are involved in the voluntary motor control of the skeletal muscles.
Which of the following substances represents an unsaturated fatty acid?
A. Cholesterol
B. Palmitate
C. Stearate
D. Choline
E. Oleate
E. Oleate


The correct answer is Choice E. Fatty acids are long-chain hydrocarbon organic acids. An unsaturated fatty acid is one in which its carbon chain contains one or more double or triple bonds capable of absorbing additional hydrogen atoms. Oleate (Choice E) is an 18 carbon chain unsaturated fatty acid with one double bond in the middle of the chain.
Cholesterol (Choice A) is one of the most abundant lipids in the body. It does not contain fatty acids.
Palmitate (Choice B) is a 16-carbon chain fatty acid that is fully saturated.
Stearate (Choice C) is an l&carbon fatty acid fully saturated with hydrogen atoms.
Choline (Choice D) is a relatively small hydrophilic compound essential in the formation of some phospholipids such as a lecithin. It is not a fatty acid.
Which of the following tissues is primarily responsible for formation of urea?
1. Liver
2. Brain
3. Kidney
4. Muscle
1. Liver


The correct choice is #1, the liver. Nitrogen is excreted predominantly in the form of urea in urine. Urea is formed in the liver, brain and also the kidney, but mainly in the liver via the urea cycle. Let us review this very important cycle. C02 and NH3 + H20 + 2ATP via carbamoylphosphate synthetase form the unstable carbamoylphosphate. Carbamoylphosphate combines with ornithine, via ornithine carbamoyltransferase to form citrulline. Citrulline and aspartate and ATP form arginosuccinate, then fumarate and arginine, via arginosuccinate lyase. Arginine is then hydrolyzed via arginase to form the urea, NH2-CO-NH2 and H2. The net reaction of the urea cycle is C02 + 2NH3 + 3ATP + 3H20 -> urea + 2ADP + AMP + 4inorganic phosphates. So again, the correct choice is #1, the liver.
Which of the following vitamins can be supplied to humans by the normal action of intestinal flora?
1. A
2. D
3. E
4. K
5. None of the above
4. K


The correct choice is #4, vitamin K. Vitamin K can be supplied to humans by the normal action of intestinal flora. Vitamin K is a key factor in the synthesis of prothrombin. In the absence of vitamin K or in the presence of dicumarol or warfarin prothrombin is not produced. Prothrombin binds to calcium, and in the presence of these antagonists of vitamin K it does not bind to calcium. Prothrombin, via activated factor X is converted into thrombin. This is modified by factor V. When calcium binds to prothrombin it increases its affinity to the phospholipid membranes from platelets of injury. This binding then brings the prothrombin into proximity of factors Xa and V, which are also involved in the clotting cascade. Thrombin, in turn, converts fibrinogen to fibrin for the final step in the clotting cascade. So again, the correct choice to question is 4, vitamin K.
Which of the following vitamins is the LEAST likely to be involved in tooth development and substance that is the immediate source of calcification?
1. A
2. B1
3. C
4. D
2. B1


The correct answer is Choice B. Tooth development and calcification are similar to that of bone. They are affected by various factors of metabolism such as the availability of calcium and phosphate in the diet, and rate of secretion of thyroid, growth, and parathyroid hormones. Vitamins are also essential in the formation of teeth. Vitamin A (Choice A) is necessary for normal growth and proliferation of epithelial cells, vitamin C (Choice C) activates prolyl hydroxylase which promotes normal formation of collagen, and vitamin D (Choice D) increases calcium absorption and helps control calcium deposition in hard tissues. Choice B, Vitamin B1,is least likely to be involved in tooth development since it functions as a cocarboxylase specifically needed in the final metabolism of carbohydrate and amino acids.
Which type of muscle contraction occurs when there is a change in length of the muscle with a corresponding change in tension on that muscle?
A. Concentric
B. Dynamic
C. Eccentric
D. Isometric
E. Isotonic
B. Dynamic


The correct answer is B.
A dynamic contraction includes both concentric and eccentric types of contractions. In general, a dynamic contraction results in a change in length of the muscle, with a corresponding change in tension on that muscle. A concentric contraction (choice A) is a type of dynamic contraction in which the muscle fibers shorten and the tension on the muscle increases. An eccentric contraction (choice C) is a type of dynamic contraction in which the muscle fiber lengthens and the tension on the muscle increases. An isometric contraction (choice D) occurs when both ends of the muscle are fixed and no change in length occurs during the contraction, but the tension increases. An isotonic contraction (choice E) occurs when a muscle shortens against a fixed load while the tension on that muscle remains constant.
With respect to a normal kidney, which of the following situations would cause the glomerular filtration rate to increase?
A. Constricting the afferht arteriole
B. Decreasing blood flow to the kidneys
C. Decreasing plasma protein concentrations
D. Decreasing renin levels
E. Sympathetic stimulation of the kidney
C. Decreasing plasma protein concentrations


The correct answer is C. The glomerular filtration rate is the amount of filtrate produced in the kidneys each minute. The average glomerular filtration rate is around 125 mL of filtrate per minute. There are many different factors that can affect the glomerular filtration rate. For example, when the blood volume is increased, due to increased water intake, the normal concentration of plasma proteins in the blood will be decreased. This relative decrease can be attributed to the dilutional effect of increased fluid volume. Therefore, when plasma protein concentrations are decreased, the kidneys will increase the glomerular filtration rate (GFR) to increase the production of urine. Constricting the afferent arteriole (choice A) will directly decrease the GFR as less blood enters the nephron. Constriction of the afferent arteriole often occurs when there is a fall in blood pressure or when there is sympathetic stimulation of the kidney (choice E). When blood flow to the kidneys decreases (choice B), the GFR will naturally decrease because there is not as much fluid being presented to the kidneys per unit of time. Decreased renin levels (choice D), leads to a decrease in the conversion of angiotensin I to angiotensin II. Since angiotensin II is a potent constrictor of the efferent arteriole, decreasing renin levels will decrease the GFR.
With respect to catabolism of amino acids, deamination results in the production of
A. keto acid
B. acetyl-CoA
C. ammonia
D. carbon dioxide
E. urea
C. ammonia


The correct answer is C. The first step in the catabolism of an amino acid is the removal of the amino group. The process requires a coenzyme derivative (vitamin B6). The amino group can be removed by deamination or transamination. Deamination is performed in the preparation of an amino acid for breakdown in the TCA cycle. In deamination, an amino group is removed and ammonia is generated. In the process of transamination, the amino group of a given amino acid is transformed to a keto acid (choice A). A keto acid resembles an amino acid except that the second carbon is bound to an oxygen rather than an amino group. Acetyl-CoA (choice B) is produced by the breakdown of pyruvic acid and the addition of an acetyl-group to these byproducts. Note: Acetyl-CoA is produced only in aerobic glycolysis. In the absence of oxygen, the pyruvic acid is converted to lactic acid. Carbon dioxide (choice D) is produced in the TCA cycle. The ammonia produced during deamination is toxic; therefore, the liver metabolizes the ammonia molecule to urea (choice E), which is a relatively non-toxic, water-soluble compound that is excreted in the urine.
With respect to the coagulation phase of homeostasis, the intrinsic pathway involves
A. Clotting factors VII, IX, XI and XII
B. Prostacyclin
C. Thrombin and fibrin
D. Thromboxane A2 and platelet factors
E. Tissue factor and tissue thromboplastin
A. Clotting factors VII, IX, XI and XII


The correct answer is A. The coagulation phase of homeostasis has three primary pathways for stimulation of blood clotting: The intrinsic pathway, the extrinsic pathway, and the common pathway. The intrinsic pathway begins with the activation of proenzymes exposed to collagen fiber located at the site of the injury and involves the clotting factors VII, IX, XI and XII (choice A). This pathway, after a series of linked reactions, forms a complex called platelet thromboplastin. The common pathway begins when the thromboplastin from both the intrinsic and extrinsic pathways appears in the plasma. After a series of reactions, prompted by these compounds, thrombin completes the coagulation process by converting fibrinogen to fibrin (choice C). The platelet phase, begins within seconds after the injury, and causes platelet adhesion to the exposed surfaces of the site of injury. Once platelets arrive at the site of injury, they begin releasing thromboxane A2 and platelet factors (choice D) which stimulate platelet aggregation and vascular spasms. The platelet phase is regulated by prostacyclin (choice B), which inhibits platelet aggregation. The extrinsic pathway is part of the coagulation phase of homeostasis and begins about 30 seconds after the traumatic event. The extrinsic pathway begins with the release of tissue factor, which combines with calcium and another procoagulant, to form tissue thromboplastin (choice E).
With respect to the physiological movement of a body part, abduction is best described as
A. the movement away from the longitudinal axis of the body
B. the movement of a structure in a superior direction
C. the movement in the anterior/posterior plane that decreases the angle between the articulating extremities.
D. the movement of the thumb which allows it to grasp and hold various objects
E. the movement of a given part of the body anteriorly in a horizontal plane
A. the movement away from the longitudinal axis of the body


The correct answer is A. Abduction is the physiological movement away from the longitudinal axis of the body. For example, swinging the right arm to the side. Elevation occurs when a structure moves in a superior direction (choice B). Conversely, depression occurs when a structure moves in a inferior direction; the process of chewing is a repetitive cycle of depressing and elevating the mandible. Flexion (choice C) is described as a movement in the anteriorlposterior plane that decreases the angle between the articulating extremities: for example, flexion of the biceps. Opposition (choice D) is the movement of the thumb that allows it to grasp and hold various objects. Protraction (choice E) is described as the movement of a given part of the body anteriorly in a horizontal plane; the grasping of ones upper lip with their lower teeth is a protracting movement.
With respect to types of muscle contractions, treppe is a situation where
A. A muscle is producing peak tension during rapid cycles of contraction and relaxation
B. A muscle is stimulated so frequently that the relaxation phase is completely eliminated.
C. A muscle is stimulated repeatedly for several seconds with a constant stimulus; the amount of tension produced then gradually increases to a maximum.
D. A second stimulus arrives before the relaxation phase has ended and a second and a more powerful contraction occurs.
E. A smooth but steady increase in the muscle tension is produced by increasing the number of active motor units.
C. A muscle is stimulated repeatedly for several seconds with a constant stimulus; the amount of tension produced then gradually increases to a maximum.


The correct answer is C. During a muscle contraction, there is no mechanism to regulate the amount of tension produced in a given muscle protraction; therefore the muscle fiber is either producing tension or is in a relaxed state. This characteristic is known as the all-or-none principle. Therefore, the amount of tension produced by a skeletal muscle as a whole is determined by both the frequency of stimulation and the number of muscles stimulated. With these principles in mind, the following principles can be considered: Treppe is situation where a muscle is stimulated repeatedly for several seconds with a constant stimulus; the amount of tension produced then gradually increases to a maximum (choice C). When a muscle is producing peak tension during rapid cycles of contraction and relaxation relazation, it is known as tetanus. Complete tetanus occurs when a muscle is stimulated so frequently that the relaxation phase is completely eliminated (choice B). Wave summation occurs when a second stimulus arrives before the relaxation phase has ended and a second, more powerful, contraction occurs (choice D). Recruitment occurs when there is a smooth but steady increase in the muscle tension that is produced by increasing the number of active motor units (choice E). The muscle is producing peak tension during rapid cycles of contraction and relaxation.
Within physiologic limits, activity of the respiratory center is decreased directly by
1. decreased pH.
2. increased pH.
3. increased hemoglobin concentration.
4. increased plasma carbon dioxide partial pressure.
2. increased pH.


The correct answer is choice 2. The respiratory center is located in the medulla. Increased C02 concentration in blood causes excess H2CO3 or carbonic acid formation which lowers pH near the respiratory center and increases ventilation. The opposite is true for decreased ventilation. Low C02 causes less acidity and higher pH, decreasing the activity of the respiratory center. Choice 3 is incorrect. The hemoglobin content of blood is fairly stable and does not change fast enough to be the controlling influence on the respiratory center. Choice 4 is incorrect. As mentioned previously, increased PC02 or partial pressure of carbon dioxide, will cause lower pH and will increase, not decrease, ventilation. So the correct answer to question is choice 2.